OB- HESI

Lakukan tugas rumah & ujian kamu dengan baik sekarang menggunakan Quizwiz!

Reciprocal attachment behavior refers to: A. The positive feedback an infant exhibits toward parents during the attachment process B. Behavior during the sensitive period when the infant is in the quiet alert stage C. Unidirectional behavior exhibited by the infant, initiated and enhanced by eye contact Incorrect D. Behavior by the infant during the sensitive period to elicit feelings of falling in love from the parents

A

Rho immune globulin will be ordered postpartum if which situation occurs? a. Mother Rh-, baby Rh+ b. Mother Rh-, baby Rh- c. Mother Rh+, baby Rh+ d. Mother Rh+, baby Rh-

A

Which condition is considered a medical emergency that requires immediate treatment? a.Inversion of the uterus b.Hypotonic uterus c.ITP d.Uterine atony

ANS: A Inversion of the uterus is likely to lead to hypovolemic shock and therefore is considered a medical emergency. Although hypotonic uterus, ITP, and uterine atony are serious conditions, they are not necessarily medical emergencies that require immediate treatment.

Which nutrients recommended dietary allowance (RDA) is higher during lactation than during pregnancy? a. Energy (kcal) b. Iron c. Vitamin A d. Folic acid

ANS: A Nutrient needs for energyprotein, calcium, iodine, zinc, B vitamins, and vitamin Cremain higher during lactation than during pregnancy. The need for iron is not higher during lactation than during pregnancy. A lactating woman does not have a greater requirement for vitamin A than a nonpregnant woman. Folic acid requirements are the highest during the first trimester of pregnancy.

Screening at 24 weeks of gestation reveals that a pregnant woman has gestational diabetes mellitus (GDM). In planning her care, the nurse and the client mutually agree that an expected outcome is to prevent injury to the fetus as a result of GDM. This fetus is at the greatest risk for which condition? a. Macrosomia b. Congenital anomalies of the central nervous system c. Preterm birth d. Low birth weight

ANS: A Poor glycemic control later in pregnancy increases the rate of fetal macrosomia. Poor glycemic control during the preconception time frame and into the early weeks of the pregnancy is associated with congenital anomalies. Preterm labor or birth is more likely to occur with severe diabetes and is the greatest risk in women with pregestational diabetes. Increased weight, or macrosomia, is the greatest risk factor for this fetus.

According to research, which risk factor for PPD is likely to have the greatest effect on the client postpartum? a. Prenatal depression b. Single-mother status c. Low socioeconomic status d. Unplanned or unwanted pregnancy

ANS: A Prenatal depression has been found to be a major risk factor for PPD. Single-mother status and low socioeconomic status are both small-relationship predictors for PPD. Although an unwanted pregnancy may contribute to the risk for PPD, it does not pose as great an effect as prenatal depression

What important, immediate postoperative care practice should the nurse remember when caring for a woman who has had a mastectomy? a.The blood pressure (BP) cuff should not be applied to the affected arm. b.Venipuncture for blood work should be performed on the affected arm. c.The affected arm should be used for intravenous (IV) therapy. d.The affected arm should be held down close to the woman's side.

ANS: A The affected arm should not be used for BP readings, IV therapy, or venipuncture. The affected arm should be elevated with pillows above the level of the right atrium.

Ovarian function and hormone production decline during which transitional phase? a. Climacteric b. Menarche c. Menopause d. Puberty

ANS: A The climacteric phase is a transitional period during which ovarian function and hormone production decline. Menarche is the term that denotes the first menstruation. Menopause refers only to the last menstrual period.Puberty is a broad term that denotes the entire transitional period between childhood and sexual maturity.

Which nonpharmacologic contraceptive method has a failure rate of less than 25%? a. Standard days variation b. Periodic abstinence c. Postovulation d. Coitus interruptus

ANS: A The standard days variation on the calendar method has a failure rate of 12% and is a variation of the calendar rhythm method with a fixed number of days for fertility in each cycle. The periodic abstinence method has a failure rate of 25% or higher. The postovulation method has a failure rate of 25% or higher. The coitus interruptus method has a failure rate of 27% or higher.

What is a maternal indication for the use of vacuum-assisted birth? a.Wide pelvic outlet b.Maternal exhaustion c.History of rapid deliveries d.Failure to progress past station 0

ANS: B A mother who is exhausted may be unable to assist with the expulsion of the fetus. The client with a wide pelvic outlet will likely not require vacuum extraction. With a rapid delivery, vacuum extraction is not necessary. A station of 0 is too high for a vacuum-assisted birth

The client being cared for has severe preeclampsia and is receiving a magnesium sulfate infusion. Which new finding would give the nurse cause for concern? a. Sleepy, sedated affect b. Respiratory rate of 10 breaths per minute c. DTRs of 2 d. Absent ankle clonus

ANS: B A respiratory rate of 10 breaths per minute indicates the client is experiencing respiratory depression from magnesium toxicity. Because magnesium sulfate is a CNS depressant, the client will most likely become sedated when the infusion is initiated. DTRs of 2 and absent ankle clonus are normal findings.

Which sign of a potential complication is the most important for the nurse to share with the client? a. Constipation b. Alteration in the pattern of fetal movement c. Heart palpitations d. Edema in the ankles and feet at the end of the day

ANS: B An alteration in the pattern or amount of fetal movement may indicate fetal jeopardy. Constipation, heart palpitations, and ankle and foot edema are normal discomforts of pregnancy that occur in the second and third trimesters.

In contrast to placenta previa, what is the most prevalent clinical manifestation of abruptio placentae? a.Bleeding b.Intense abdominal pain c.Uterine activity d.Cramping

ANS: B Pain is absent with placenta previa and may be agonizing with abruptio placentae. Bleeding may be present in varying degrees for both placental conditions. Uterine activity and cramping may be present with both placental conditions.

The client is instructed to place her thumb and forefinger on the areola and gently press inward. What is the purpose of this exercise? a. To check the sensitivity of the nipples b. To determine whether the nipple is everted or inverted c. To calculate the adipose buildup in the abdomen d. To see whether the fetus has become inactive

ANS: B Sometimes known as the pinch test, this exercise is used to determine whether the nipple is everted or inverted. Nipples must be everted to allow breastfeeding. The pinch does not determine the level of sensitivity of the nipples, nor is it not used to determine the level of adipose tissue in the abdomen. Fetal activity is not determined by using the pinch test.

Which benefit regarding FAMs makes it an appealing choice for some women? a. Adherence to strict recordkeeping b. Absence of chemicals and hormones c. Decreased involvement and intimacy of partner d. Increased spontaneity of coitus

ANS: B The absence of chemicals or hormones to alter the natural menstrual flow is extremely important to some women. The strict recordkeeping with FAMs may be difficult and creates a potential risk for failure. These methods require increased involvement by the partner; however, they also reduce the spontaneity of coitus.

Which body part both protects the pelvic structures and accommodates the growing fetus during pregnancy? a. Perineum b. Bony pelvis c. Vaginal vestibule d. Fourchette

ANS: B The bony pelvis protects and accommodates the growing fetus. The perineum covers the pelvic structures. The vaginal vestibule contains openings to the urethra and vagina. The fourchette is formed by the labia minor.

The unique muscle fibers that constitute the uterine myometrium make it ideally suited for what? a. Menstruation b. Birth process c. Ovulation d. Fertilization

ANS: B The myometrium is made up of layers of smooth muscle that extend in three directions. These muscles assist in the birth process by expelling the fetus, ligating blood vessels after birth, and controlling the opening of the cervical os.

Which clinical finding is a major use of ultrasonography in the first trimester? a. Amniotic fluid volume b. Presence of maternal abnormalities c. Placental location and maturity d. Cervical length

ANS: B Ultrasonography can detect certain uterine abnormalities such as bicornuate uterus, fibroids, and ovarian cysts. Amniotic fluid volume, placental location and maturity, and cervical length are not available via ultrasonography until the second or third trimester.

What is the most likely cause for variable FHR decelerations? a.Altered fetal cerebral blood flow b.Umbilical cord compression c.Uteroplacental insufficiency d.Fetal hypoxemia

ANS: B Variable FHR decelerations can occur at any time during the uterine contracting phase and are caused by compression of the umbilical cord. Altered fetal cerebral blood flow results in early decelerations in the FHR. Uteroplacental insufficiency results in late decelerations in the FHR. Fetal hypoxemia initially results in tachycardia and then bradycardia if hypoxia continues.

Which condition is the least likely cause of amenorrhea in a 17yearold client? a. Anatomic abnormalities b. Type 1 diabetes mellitus c. Obesity d. Pregnancy

ANS: C A moderately obese adolescent (20% to 30% above ideal weight) may have early onset menstruation. Girls who regularly exercise before menarche can have delayed onset of menstruation to age 18 years. Anatomic abnormalities are a possible cause of amenorrhea. Type 1 diabetes mellitus is a possible cause of amenorrhea. Pregnancy is the most common cause of amenorrhea.

Nurses with an understanding of cultural differences regarding likely reactions to pain may be better able to help their clients. Which clients may initially appear very stoic but then become quite vocal as labor progresses until late in labor, when they become more vocal and request pain relief? a.Chinese b.Arab or Middle Eastern c.Hispanic d.African-American

ANS: C Hispanic women may be stoic early in labor but more vocal and ready for medications later. Chinese women may not show reactions to pain. Medical interventions must be offered more than once. Arab or Middle Eastern women may be vocal in response to labor pain from the start; they may prefer pain medications. African-American women may openly express pain; the use of medications for pain is more likely to vary with the individual.

The indirect Coombs test is a screening tool for Rh incompatibility. If the titer is greater than ______, amniocentesis may be a necessary next step. a. 1:2 b. 1:4 c. 1:8 d. 1:12

ANS: C If the maternal titer for Rh antibodies is greater 1:8, then an amniocentesis is indicated to determine the level of bilirubin in the amniotic fluid. This testing will determine the severity of fetal hemolytic anemia.

Having a genetic mutation may create an 85% chance of developing breast cancer in a woman's lifetime. Which condition does not increase a client's risk for breast cancer? a.BRCA1 or BRCA2 gene mutation b.Li-Fraumeni syndrome c.Paget disease d.Cowden syndrome

ANS: C Paget disease originates in the nipple and causes nipple carcinoma and exhibits bleeding, oozing, and crusting of the nipple. BRCA1 or BRCA2, Li-Fraumeni syndrome, and Cowden syndrome are all genetic mutations that have different family pedigrees and increase the risk of breast cancer.

While obtaining a diet history, the nurse might be told that the expectant mother has cravings for ice chips, cornstarch, and baking soda. Which nutritional problem does this behavior indicate? a. Preeclampsia b. Pyrosis c. Pica d. Purging

ANS: C The consumption of foods low in nutritional value or of nonfood substances (e.g., dirt, laundry starch) is called pica. Preeclampsia is a vasospastic disease process encountered after 20 weeks of gestation. Characteristics of preeclampsia include increasing hypertension, proteinuria, and hemoconcentration. Pyrosis is a burning sensation in the epigastric region, otherwise known as heartburn. Purging refers to self-induced vomiting after consuming large quantities of food.

What is the primary role of the nonpregnant partner during pregnancy? a. To provide financial support b. To protect the pregnant woman from old wives tales c. To support and nurture the pregnant woman d. To make sure the pregnant woman keeps prenatal appointments

ANS: C The partners primary role in pregnancy is to nurture the pregnant woman and respond to her feelings of vulnerability. Although financial support is important, it is not the partners primary role in pregnancy. Protecting the pregnant woman from old wives tales is not the partners role. The womans partner can encourage the client to keep all appointments; however, this is not the most important role during the pregnancy.

Which statement related to the condition of endometriosis is most accurate? a. Endometriosis is characterized by the presence and growth of endometrial tissue inside the uterus. b. It is found more often in African-American women than in Caucasian or Asian women. c. Endometriosis may worsen with repeated cycles or remain asymptomatic and disappear after menopause. d. It is unlikely to affect sexual intercourse or fertility.

ANS: C With endometriosis, the endometrial tissue is outside the uterus. Endometriosis is found equally in Caucasian and African-American women and is slightly more prevalent in Asian women. Symptoms vary among women, ranging from nonexistent to incapacitating. The condition is seven times more prevalent in women who have a firstdegree relative with endometriosis. Women can experience painful intercourse and impaired fertility with endometriosis.

Which client should the nurse refer for further testing? a.Left breast slightly smaller than right breast b.Eversion (elevation) of both nipples c.Faintly visible bilateral symmetry of venous network d.Small dimple located in the upper outer quadrant of the right breast

ANS: D A small dimple is an abnormal finding and should be further evaluated. Nipple retraction and a dimpling or pitting of the skin is suggestive of a locally advanced, aggressive form of breast cancer. In many women, one breast is smaller than the other, and eversion of both nipples is a normal finding. Faintly visible venous network is also a normal finding.

Which statement is the most complete and accurate description of medical abortions? a. Medical abortions are performed only for maternal health. b. They can be achieved through surgical procedures or with drugs. c. Medical abortions are mostly performed in the second trimester. d. They can be either elective or therapeutic.

ANS: D Medical abortions can be either elective (the womans choice) or therapeutic (for reasons of maternal or fetal health) and are performed through the use of medications rather than surgical procedures. Medical abortions are usually performed in the first trimester.

The nurse performs a vaginal examination to assess a client's labor progress. Which action should the nurse take next? a. Perform an examination at least once every hour during the active phase of labor. b. Perform the examination with the woman in the supine position. c. Wear two clean gloves for each examination. d. Discuss the findings with the woman and her partner.

ANS: D The nurse should discuss the findings of the vaginal examination with the woman and her partner, as well as report the findings to the primary care provider. A vaginal examination should be performed only when indicated by the status of the woman and her fetus. The woman should be positioned so as to avoid supine hypotension. The examiner should wear a sterile glove while performing a vaginal examination for a laboring woman.

In which situation would the nurse be called on to stimulate the fetal scalp? a.As part of fetal scalp blood sampling b.In response to tocolysis c.In preparation for fetal oxygen saturation monitoring d.To elicit an acceleration in the FHR

ANS: D The scalp can be stimulated using digital pressure during a vaginal examination. Fetal scalp blood sampling involves swabbing the scalp with disinfectant before a sample is collected. The nurse stimulates the fetal scalp to elicit an acceleration of the FHR. Tocolysis is relaxation of the uterus. Fetal oxygen saturation monitoring involves the insertion of a sensor.

A woman who is 14 weeks pregnant tells the nurse that she always had a glass of wine with dinner before she became pregnant. She has abstained during her first trimester and would like to know if it is safe for her to have a drink with dinner now. Which guidance should the nurse provide? a. Since youre in your second trimester, theres no problem with having one drink with dinner. b. One drink every night is too much. One drink three times a week should be fine. c. Since youre in your second trimester, you can drink as much as you like. d. Because no one knows how much or how little alcohol it takes to cause fetal problems, the best course is to abstain throughout your pregnancy.

ANS: D The statement Because no one knows how much or how little alcohol it takes to cause fetal problems, the best course is to abstain throughout your pregnancy is accurate. A safe level of alcohol consumption during pregnancy has not yet been established. Although the consumption of occasional alcoholic beverages may not be harmful to the mother or her developing fetus, complete abstinence is strongly advised.

A pregnant woman is at 38 weeks of gestation. She wants to know whether there are any signs that "labor is getting close to starting." Which finding is an indication that labor may begin soon? a. Weight gain of 1.5 to 2 kg (3 to 4 lb) b. Increase in fundal height c. Urinary retention d. Surge of energy

ANS: D Women speak of having a burst of energy before labor. The woman may lose 0.5 to 1.5 kg, as a result of water loss caused by electrolyte shifts that, in turn, are caused by changes in the estrogen and progesterone levels. When the fetus descends into the true pelvis (called lightening), the fundal height may decrease. Urinary frequency may return before labor.

During which stage of role attainment do the parents become acquainted with their baby and combine parenting activities with cues from the infant? a. Anticipatory b. Formal c. Informal d. Personal

B

If the fundus is palpated on the right side of the abdomen above the expected level, the nurse should suspect that the patient has a. Been lying on her right side too long b. A distended bladder c. Stretched ligaments that are unable to support the uterus d. A normal involution

B

Many women given up smoking during pregnancy to protect the health of the fetus. The majority of women resumed smoking within the first 6 months postpartum. Factors that increase the likelihood of relapse include (select all that apply): a. Living with a smoker b. Returning to work c. Weight concerns d. Successful breastfeeding e. Failure to breastfeed

A, C, E

What is the correct definition of a spontaneous termination of a pregnancy (abortion)? a.Pregnancy is less than 20 weeks. b.Fetus weighs less than 1000 g. c.Products of conception are passed intact. d.No evidence exists of intrauterine infection.

ANS: A An abortion is the termination of pregnancy before the age of viability (20 weeks). The weight of the fetus is not considered because some older fetuses may have a low birth weight. A spontaneous abortion may be complete or incomplete and may be caused by many problems, one being intrauterine infection.

Childbirth may result in injuries to the vagina and uterus. Pelvic floor exercises also known as Kegel exercises will help to strengthen the perineal muscles and encourage healing. The nurse knows that the patient understands the correct process for completing these conditioning exercises when she reports: a. "I contract my thighs, buttocks, and abdomen." b. "I do 10 of these exercises every day." c. "I stand while practicing this new exercise routine." d. "I pretend that I am trying to stop the flow of urine midstream."

D

Dysfunctional uterine bleeding (DUB) is defined as excessive uterine bleeding without a demonstrable cause. Which statement regarding this condition is most accurate? a. DUB is most commonly caused by anovulation. b. DUB most often occurs in middle age. c. The diagnosis of DUB should be the first consideration for abnormal menstrual bleeding. d. Steroids are the most effective medical treatment for DUB.

ANS: A Anovulation may occur because of hypothalamic dysfunction or polycystic ovary syndrome. DUB most often occurs when the menstrual cycle is being established or when it draws to a close at menopause. A diagnosis of DUB is made only after all other causes of abnormal menstrual bleeding have been ruled out. The most effective medical treatment is oral or intravenous estrogen.

A woman is 16 weeks pregnant and has elected to terminate her pregnancy. Which is the most common technique used for the termination of a pregnancy in the second trimester? a. Dilation and evacuation (D&E) b. Methotrexate administration c. Prostaglandin administration d. Vacuum aspiration

ANS: A D&E can be performed at any point up to 20 weeks of gestation. It is more commonly performed between 13 and 16 weeks of gestation. Methotrexate is a cytotoxic drug that causes early abortion by preventing fetal cell division. Prostaglandins are also used for early abortion and work by dilating the cervix and initiating uterine wall contractions. Vacuum aspiration is used for abortions in the first trimester.

Which statement best describes Kegel exercises? a. Kegel exercises were developed to control or reduce incontinent urine loss. b. Kegel exercises are the best exercises for a pregnant woman because they are so pleasurable. c. Kegel exercises help manage stress. d. Kegel exercises are ineffective without sufficient calcium in the diet.

ANS: A Kegel exercises help control the urge to urinate. Although these exercises may be fun for some, the most important factor is the control they provide over incontinence. Kegel exercises help manage urination, not stress. Calcium in the diet is important but not related to Kegel exercises.

Which vitamins or minerals may lead to congenital malformations of the fetus if taken in excess by the mother? a. Zinc b. Vitamin D c. Folic acid d. Vitamin A

ANS: D If taken in excess, vitamin A causes a number of problems. An analog of vitamin A appears in prescribed acne medications, which must not be taken during pregnancy. Zinc, vitamin D, and folic acid are all vital to good maternity and fetal health and are highly unlikely to be consumed in excess.

The perinatal nurse realizes that an FHR that is tachycardic, bradycardic, has late decelerations, or loss of variability is nonreassuring and is associated with which condition? a.Hypotension b.Cord compression c.Maternal drug use d.Hypoxemia

ANS: D Nonreassuring FHR patterns are associated with fetal hypoxemia. Fetal bradycardia may be associated with maternal hypotension. Variable FHR decelerations are associated with cord compression. Maternal drug use is associated with fetal tachycardia.

Which component of the physical examination are Leopold's maneuvers unable to determine? a. Gender of the fetus b. Number of fetuses c. Fetal lie and attitude d. Degree of the presenting part's descent into the pelvis

ANS: A Leopold's maneuvers help identify the number of fetuses, the fetal lie and attitude, and the degree of descent of the presenting part into the pelvis. The gender of the fetus cannot be determined by performing Leopold's maneuvers

What is the most common reproductive tract cancer associated with pregnancy? a.Cervical b.Uterine c.Ovarian d.Fallopian tube

ANS: A The incidence of cervical cancer concurrent with pregnancy is reported to be 1 in 2000 pregnancies making it the most common reproductive tract cancer associated with pregnancy. Uterine cancer is rarely diagnosed during pregnancy. Ovarian cancer is the second most frequent cancer diagnosis in pregnancy. At an incidence rate of approximately 1%, fallopian tube cancer remains a rare occurrence. The peak incidence of tubal cancer is between the ages of 50 and 55 years; for this cancer to be concurrent with pregnancy is only a remote possibility.

Which nursing intervention should be immediately performed after the forceps-assisted birth of an infant? a.Assessing the infant for signs of trauma b.Administering prophylactic antibiotic agents to the infant c.Applying a cold pack to the infant's scalp d.Measuring the circumference of the infant's head

ANS: A The infant should be assessed for bruising or abrasions at the site of application, facial palsy, and subdural hematoma. Prophylactic antibiotics are not necessary with a forceps delivery. A cold pack would place the infant at risk for cold stress and is contraindicated. Measuring the circumference of the head is part of the initial nursing assessment.

Which major neonatal complication is carefully monitored after the birth of the infant of a diabetic mother? a. Hypoglycemia b. Hypercalcemia c. Hypobilirubinemia d. Hypoinsulinemia

ANS: A The neonate is at highest risk for hypoglycemia because fetal insulin production is accelerated during pregnancy to metabolize excessive glucose from the mother. At birth, the maternal glucose supply stops and the neonatal insulin exceeds the available glucose, thus leading to hypoglycemia. Hypocalcemia is associated with preterm birth, birth trauma, and asphyxia, all common problems of the infant of a diabetic mother. Excess erythrocytes are broken down after birth, and large amounts of bilirubin are released into the neonates circulation, with resulting hyperbilirubinemia. Because fetal insulin production is accelerated during pregnancy, hyperinsulinemia develops in the neonate.

A perimenopausal client has arrived for her annual gynecologic examination. Which preexisting condition would be extremely important for the nurse to identify during a discussion regarding the risks and benefits of hormone therapy? a. Breast cancer b. Vaginal and urinary tract atrophy c. Osteoporosis d. Arteriosclerosis

ANS: A Women with a high risk for breast cancer should be counseled against using estrogen replacement therapy (ERT). Estrogen prevents the atrophy of vaginal and urinary tract tissue and protects against the development of osteoporosis. Estrogen also has a favorable effect on circulating lipids, reducing low-density lipoprotein (LDL) and total cholesterol levels and increasing high-density lipoprotein (HDL) levels. It also has a direct antiatherosclerotic effect on the arteries.

The musculoskeletal system adapts to the changes that occur throughout the pregnancy. Which musculoskeletal alteration should the client expect? a. Her center of gravity will shift backward. b. She will have increased lordosis. c. She will have increased abdominal muscle tone. d. She will notice decreased mobility of her pelvic joints.

ANS: B An increase in the normal lumbosacral curve (lordosis) develops, and a compensatory curvature in the cervicodorsal region develops to help her maintain balance. The center of gravity shifts forward. She will have decreased abdominal muscle tone and will notice increased mobility of her pelvic joints

The nurse who is caring for a woman hospitalized for hyperemesis gravidarum would expect the initial treatment to involve what? a. Corticosteroids to reduce inflammation b. Intravenous (IV) therapy to correct fluid and electrolyte imbalances c. Antiemetic medication, such as pyridoxine, to control nausea and vomiting d. Enteral nutrition to correct nutritional deficits

ANS: B Initially, the woman who is unable to down clear liquids by mouth requires IV therapy to correct fluid and electrolyte imbalances. Corticosteroids have been successfully used to treat refractory hyperemesis gravidarum, but they are not the expected initial treatment for this disorder. Pyridoxine is vitamin B6, not an antiemetic medication. Promethazine, a common antiemetic, may be prescribed. In severe cases of hyperemesis gravidarum, enteral nutrition via a feeding tube may be necessary to correct maternal nutritional deprivation but is not the initial treatment for this client.

The nurse should be aware of what important information regarding systemic analgesics administered during labor? a.Systemic analgesics cross the maternal blood-brain barrier as easily as they do the fetal blood-brain barrier. b.Effects on the fetus and newborn can include decreased alertness and delayed sucking. c.Intramuscular (IM) administration is preferred over IV administration. d.IV patient-controlled analgesia (PCA) results in increased use of an analgesic.

ANS: B The effects of analgesics depend on the specific drug administered, the dosage, and the timing. Systemic analgesics cross the fetal blood-brain barrier more readily than the maternal blood-brain barrier. IV administration is preferred over IM administration because the drug acts faster and more predictably. PCA results in a decrease in the use of an analgesic.

What would a steady trickle of bright red blood from the vagina in the presence of a firm fundus suggest to the nurse? a.Uterine atony b.Lacerations of the genital tract c.Perineal hematoma d.Infection of the uterus

ANS: B Undetected lacerations will bleed slowly and continuously. Bleeding from lacerations is uncontrolled by uterine contraction. The fundus is not firm in the presence of uterine atony. A hematoma would develop internally. Swelling and discoloration would be noticeable; however, bright bleeding would not be. With an infection of the uterus, an odor to the lochia and systemic symptoms such as fever and malaise would be present.

Which statement best describes the rationale for the physiologic anemia that occurs during pregnancy? a. Physiologic anemia involves an inadequate intake of iron. b. Dilution of hemoglobin concentration occurs in pregnancy with physiologic anemia. c. Fetus establishes the iron stores. d. Decreased production of erythrocytes occur.

ANS: B When blood volume expansion is more pronounced and occurs earlier than the increase in red blood cells, the woman has physiologic anemia, which is the result of the dilution of hemoglobin concentration rather than inadequate hemoglobin. An inadequate intake of iron may lead to true anemia. The production of erythrocytes increases during pregnancy.

A pregnant womans diet consists almost entirely of whole grain breads and cereals, fruits, and vegetables. Which dietary requirement is the nurse most concerned about? a. Calcium b. Protein c. Vitamin B12 d. Folic acid

ANS: C A pregnant womans diet is consistent with that followed by a strict vegetarian (vegan). Vegans consume only plant products. Because vitamin B12 is found in foods of animal origin, this diet is deficient in vitamin B12. Depending on the womans food choices, a pregnant womans diet may be adequate in calcium. Protein needs can be sufficiently met by a vegetarian diet. The nurse should be more concerned with the womans intake of vitamin B12 attributable to her dietary restrictions. Folic acid needs can be met by enriched bread products.

Which client is most at risk for fibroadenoma of the breast? a.38-year-old woman b.50-year-old woman c.16-year-old girl d.27-year-old woman

ANS: C Although it may occur at any age, fibroadenoma is most common in the teenage years. Ductal ectasia and intraductal papilloma become more common as a woman approaches menopause. Fibrocystic breast changes are more common during the reproductive years.

If consistently and correctly used, which of the barrier methods of contraception has the lowest failure rate? a. Spermicides b. Female condoms c. Male condoms d. Diaphragms

ANS: C For typical users, the failure rate for male condoms may approach 18%. Spermicide failure rates are approximately 28%. The failure rate for female condoms is approximately 21%. The failure rate for diaphragms with spermicides is 12%.

A woman has a breast mass that is not well delineated and is nonpalpable, immobile, and nontender. Which condition is this client experiencing? a.Fibroadenoma b.Lipoma c.Intraductal papilloma d.Mammary duct ectasia

ANS: C Intraductal papilloma is the only benign breast mass that is nonpalpable. Fibroadenoma is well delineated, palpable, and movable. Lipoma is palpable and movable. Mammary duct ectasia is not well delineated and is immobile, but it is palpable and painful.

Which is the most accurate description of PPD without psychotic features? a. Postpartum baby blues requiring the woman to visit with a counselor or psychologist b. Condition that is more common among older Caucasian women because they have higher expectations c. Distinguishable by pervasive sadness along with mood swings d. Condition that disappears without outside help

ANS: C PPD is characterized by an intense pervasive sadness along with labile mood swings and is more persistent than postpartum baby blues. PPD, even without psychotic features, is more serious and persistent than postpartum baby blues. PPD is more common among younger mothers and African-American mothers. Most women need professional help to get through PPD, including pharmacologic intervention.

At a routine prenatal visit, the nurse explains the development of the fetus to her client. At approximately ____ weeks of gestation, lecithin is forming on the alveolar surfaces, the eyelids open, and the fetus measures approximately 27 cm crown to rump and weighs approximately 1110 g. The client is how many weeks of gestation at todays visit? a. 20 b. 24 c. 28 d. 30

ANS: C These milestones in human development occur at 28 weeks of gestation. These milestones have not occurred by 20 or 24 weeks of gestation but have been reached before 30 weeks of gestation.

The nurse has evaluated a client with preeclampsia by assessing DTRs. The result is a grade of 3+. Which DTR response most accurately describes this score? a. Sluggish or diminished b. Brisk, hyperactive, with intermittent or transient clonus c. Active or expected response d. More brisk than expected, slightly hyperactive

ANS: D DTRs reflect the balance between the cerebral cortex and the spinal cord. They are evaluated at baseline and to detect changes. A slightly hyperactive and brisk response indicates a grade 3+ response.

A woman with preeclampsia has a seizure. What is the nurses highest priority during a seizure? a. To insert an oral airway b. To suction the mouth to prevent aspiration c. To administer oxygen by mask d. To stay with the client and call for help

ANS: D If a client becomes eclamptic, then the nurse should stay with the client and call for help. Nursing actions during a convulsion are directed toward ensuring a patent airway and client safety. Insertion of an oral airway during seizure activity is no longer the standard of care. The nurse should attempt to keep the airway patent by turning the clients head to the side to prevent aspiration. Once the seizure has ended, it may be necessary to suction the clients mouth. Oxygen is administered after the convulsion has ended.

Which alteration in cyclic bleeding best describes bleeding that occurs at any time other than menses? a. Oligomenorrhea b. Menorrhagia c. Leiomyoma d. Metrorrhagia

ANS: D Metrorrhagia (intermenstrual bleeding) refers to any episode or degree of bleeding that occurs between periods. It may be caused by contraceptives that contain progesterone or by intrauterine devices (IUDs).Oligomenorrhea is infrequent or scanty menstruation. Menorrhagia is excessive menstruation. Leiomyoma is a common cause of excessive bleeding.

Which action would be inappropriate for the nurse to perform before beginning the health history interview? a. Smile and ask the client whether she has any special concerns. b. Speak in a relaxed manner with an even, nonjudgmental tone. c. Make the client comfortable. d. Tell the client her questions are irrelevant.

ANS: D The woman should be assured that all of her questions are relevant and important. Beginning any client interaction with a smile is important and assists in putting the client at ease. If the nurse speaks in a relaxed manner, then the client will likely be more relaxed during the interview. The clients comfort should always be ensured before beginning the interview.

Which presumptive sign or symptom of pregnancy would a client experience who is approximately 10 weeks of gestation? a. Amenorrhea b. Positive pregnancy test c. Chadwick sign d. Hegar sign

ANS: A Amenorrhea is a presumptive sign of pregnancy. Presumptive signs of pregnancy are those felt by the woman. A positive pregnancy test and the presence of the Chadwick and Hegar signs are all probable signs of pregnancy.

A woman in labor passes some thick meconium as her amniotic fluid ruptures. The client asks the nurse where the baby makes the meconium. What is the correct response by the nurse? a. Fetal intestines b. Fetal kidneys c. Amniotic fluid d. Placenta

ANS: A As the fetus nears term, fetal waste products accumulate in the intestines as dark green-to-black, tarry meconium. Meconium is not produced by the fetal kidneys nor should it be present in the amniotic fluid, which may be an indication of fetal compromise. The placenta does not produce meconium.

A client is experiencing back labor and complains of intense pain in her lower back. Which measure would best support this woman in labor? a.Counterpressure against the sacrum b.Pant-blow (breaths and puffs) breathing techniques c.Effleurage d.Conscious relaxation or guided imagery

ANS: A Counterpressure is steady pressure applied by a support person to the sacral area with the fist or heel of the hand. This technique helps the woman cope with the sensations of internal pressure and pain in the lower back. The pain management techniques of pant-blow, effleurage, and conscious relaxation or guided imagery are usually helpful for contractions per the gate-control theory

Which condition is the most common malignancy of the reproductive system? a.Endometrial cancer b.Cervical cancer c.Ovarian cancer d.Vulvar and vaginal cancer

ANS: A Endometrial cancer occurs most frequently in Caucasian women and after menopause. Certain viral infections and sexually transmitted diseases (STIs) create risks for cervical cancer. Ovarian cancer is the most malignant reproductive system cancer and accounts for the most deaths. Cancers of the vulva and vagina are relatively rare.

A nurse providing couplet care should understand the issue of nipple confusion. In which situation might this condition occur? a. Breastfeeding babies receive supplementary bottle feedings. b. Baby is too abruptly weaned. c. Pacifiers are used before breastfeeding is established. d. Twins are breastfed together.

a. Breastfeeding babies receive supplementary bottle feedings.

The nurse is explaining the benefits associated with breastfeeding to a new mother. Which statement by the nurse would provide conflicting information to the client? a. Women who breastfeed have a decreased risk of breast cancer. b. Breastfeeding is an effective method of birth control. c. Breastfeeding increases bone density. d. Breastfeeding may enhance postpartum weight loss.

b. Breastfeeding is an effective method of birth control.

With regard to the long-term consequences of infant feeding practices, the nurse should instruct the obese client that the best strategy to decrease the risk for childhood obesity for her infant is: a. An on-demand feeding schedule. b. Breastfeeding. c. Lower-calorie infant formula. d. Smaller, more frequent feedings.

b. Breastfeeding.

Which statement regarding the nutrient needs of breastfed infants is correct? a. Breastfed infants need extra water in hot climates. b. During the first 3 months, breastfed infants consume more energy than formula-fed infants. c. Breastfeeding infants should receive oral vitamin D drops daily during at least the first 2 months. d. Vitamin K injections at birth are not necessary for breastfed infants.

c. Breastfeeding infants should receive oral vitamin D drops daily during at least the first 2 months.

In providing support to a new mother who must return to full-time employment 6 weeks after a vaginal delivery, the nurse should: a. Allow her to express her positive and negative feelings freely. b. Reassure her that she'll get used to leaving her baby. c. Discuss child care arrangements with her. d. Allow her to solve the problem on her own.

A

The mother-baby nurse is able to recognize reciprocal attachment behavior. This refers to: a. The positive feedback an infant exhibits toward parents during the attachment process b. Behavior during the sensitive period when the infant is in the quiet alert stage c. Unidirectional behavior exhibited by the infant, initiated and enhanced by eye contact d. Behavior by the infant during the sensitive period to elicit feelings of "falling in love" from the parents

A

Which finding 12 hours after birth requires further assessment? a. The fundus is palpable two fingerbreadths above the umbilicus. b. The fundus is palpable at the level of the umbilicus. c. The fundus is palpable one fingerbreadth below the umbilicus. d. The fundus is palpable two fingerbreadths below the umbilicus.

A

Which woman is most likely to have severe afterbirth pains and request a narcotic analgesic? a. Gravida 5, para 5 b. Woman who is bottle-feeding her first child c. Primipara who delivered a 7-lb boy d. Woman who wishes to breastfeed as soon as her baby is out of the neonatal intensive care unit

A

A woman is undergoing a nipple-stimulated CST. She is having contractions that occur every 3 minutes. The fetal heart rate (FHR) has a baseline heart rate of approximately 120 beats per minute without any decelerations. What is the correct interpretation of this test? a. Negative b. Positive c. Satisfactory d. Unsatisfactory

ANS: A Adequate uterine activity necessary for a CST consists of three contractions in a 10-minute time frame. If no decelerations are observed in the FHR pattern with the contractions, then the findings are considered to be negative. A positive CST indicates the presence of repetitive late FHR decelerations. The terms satisfactory or unsatisfactory are not applicable.

Which alteration in the FHR pattern would indicate the potential need for an amnioinfusion? a.Variable decelerations b.Late decelerations c.Fetal bradycardia d.Fetal tachycardia

ANS: A Amnioinfusion is used during labor to either dilute meconium-stained amniotic fluid or supplement the amount of amniotic fluid to reduce the severity of variable FHR decelerations caused by cord compression. Late decelerations are unresponsive to amnioinfusion. Amnioinfusion is not appropriate for the treatment of fetal bradycardia and has no bearing on fetal tachycardia.

Which information regarding amniotic fluid is important for the nurse to understand? a. Amniotic fluid serves as a source of oral fluid and a repository for waste from the fetus. b. Volume of the amniotic fluid remains approximately the same throughout the term of a healthy pregnancy. c. The study of fetal cells in amniotic fluid yields little information. d. A volume of more than 2 L of amniotic fluid is associated with fetal renal abnormalities.

ANS: A Amniotic fluid serves as a source of oral fluid, serves as a repository for waste from the fetus, cushions the fetus, and helps maintain a constant body temperature. The volume of amniotic fluid constantly changes. The study of amniotic fluid yields information regarding the sex of the fetus and the number of chromosomes. Too much amniotic fluid (hydramnios) is associated with gastrointestinal and other abnormalities.

Which women should undergo prenatal testing for the human immunodeficiency virus (HIV)? a. All women, regardless of risk factors b. Women who have had more than one sexual partner c. Women who have had a sexually transmitted infection (STI) d. Woman who are monogamous with one partner

ANS: A An HIV test is recommended for all women, regardless of risk factors. The incidence of perinatal transmission from an HIV-positive mother to her fetus ranges from 25% to 35%. Women who test positive for HIV can then be treated.

A client asks her nurse, My doctor told me that he is concerned with the grade of my placenta because I am overdue. What does that mean? What is the nurses best response? a. Your placenta changes as your pregnancy progresses, and it is given a score that indicates how well it is functioning. b. Your placenta isnt working properly, and your baby is in danger. c. We need to perform an amniocentesis to detect if you have any placental damage. d. Dont worry about it. Everything is fine

ANS: A An explanation of what is meant by the grade of my placenta is the most appropriate response. If the client desires further information, the nurse can explain that calcium deposits are significant in postterm pregnancies, and ultrasonography can also be used to determine placental aging. Although stating that the clients placenta is not working properly and that the baby is in danger may be a valid response, it does not reflect therapeutic communication techniques and is likely to alarm the client. An ultrasound, not amniocentesis, is the method of assessment used to determine placental maturation. Telling the client not to worry is not appropriate and discredits her concerns.

The client is being induced in response to worsening preeclampsia. She is also receiving magnesium sulfate. It appears that her labor has not become active, despite several hours of oxytocin administration. She asks the nurse, Why is this taking so long? What is the nurses most appropriate response? a. The magnesium is relaxing your uterus and competing with the oxytocin. It may increase the duration of your labor. b. I dont know why it is taking so long. c. The length of labor varies for different women. d. Your baby is just being stubborn.

ANS: A Because magnesium sulfate is a tocolytic agent, its use may increase the duration of labor. The amount of oxytocin needed to stimulate labor may be more than that needed for the woman who is not receiving magnesium sulfate. The nurse should explain to the client the effects of magnesium sulfate on the duration of labor. Although the length of labor varies for different women, the most likely reason this womans labor is protracted is the tocolytic effects of magnesium sulfate. The behavior of the fetus has no bearing on the length of labor.

Which statement indicates that a client requires additional instruction regarding BSE? a. Yellow discharge from my nipple is normal if Im having my period. b. I should check my breasts at the same time each month, after my period. c. I should also feel in my armpit area while performing my breast examination. d. I should check each breast in a set way, such as in a circular motion.

ANS: A Discharge from the nipples requires further examination from a health care provider. The breasts should be checked at the same time each month. The armpit should also be examined. A circular motion is the best method during which to ascertain any changes in the breast tissue.

What is the most likely cause for early decelerations in the fetal heart rate (FHR) pattern? a.Altered fetal cerebral blood flow b.Umbilical cord compression c.Uteroplacental insufficiency d.Spontaneous rupture of membranes

ANS: A Early decelerations are the fetus' response to fetal head compression; these are considered benign, and interventions are not necessary. Variable decelerations are associated with umbilical cord compression. Late decelerations are associated with uteroplacental insufficiency. Spontaneous rupture of membranes has no bearing on the FHR unless the umbilical cord prolapses, which would result in variable or prolonged bradycardia.

Which statement regarding emergency contraception is correct? a. Emergency contraception requires that the first dose be taken within 72 hours of unprotected intercourse. b. Emergency contraception may be taken right after ovulation. c. Emergency contraception has an effectiveness rate in preventing pregnancy of approximately 50%. d. Emergency contraception is commonly associated with the side effect of menorrhagia.

ANS: A Emergency contraception should be taken as soon as possible or within 72 hours of unprotected intercourse to prevent pregnancy. If taken before ovulation, follicular development is inhibited, which prevents ovulation. The risk of pregnancy is reduced by as much as 75%. The most common side effect of postcoital contraception is nausea.

Which clinical finding or intervention might be considered the rationale for fetal tachycardia to occur? a.Maternal fever b.Umbilical cord prolapse c.Regional anesthesia d.Magnesium sulfate administration

ANS: A Fetal tachycardia can be considered an early sign of fetal hypoxemia and may also result from maternal or fetal infection. Umbilical cord prolapse, regional anesthesia, and the administration of magnesium sulfate will each more likely result in fetal bradycardia, not tachycardia.

Which guidance might the nurse provide for a client with severe morning sickness? a. Trying lemonade and potato chips b. Drinking plenty of fluids early in the day c. Immediately brushing her teeth after eating d. Never snacking before bedtime

ANS: A Interestingly, some women can tolerate tart or salty foods when they are nauseated. Lemonade and potato chips are an ideal combination. The woman should avoid drinking too much when nausea is most likely, but she should increase her fluid levels later in the day when she feels better. The woman should avoid brushing her teeth immediately after eating. A small snack of cereal and milk or yogurt before bedtime may help the stomach in the morning.

Despite warnings, prenatal exposure to alcohol continues to far exceed exposure to illicit drugs. Which condition is rarely associated with fetal alcohol syndrome (FAS)? a. Respiratory conditions b. Intellectual impairment c. Neural development disorder d. Alcohol-related birth defects (ARBDs)

ANS: A Respiratory difficulties are not attributed to exposure to alcohol in utero. Other abnormalities related to FAS include mental retardation, neurodevelopment disorders, and ARBDs.

A client complains of severe abdominal and pelvic pain around the time of menstruation. This pain has become progressively worse over the last 5 years. She also complains of pain during intercourse and has tried unsuccessfully to become pregnant for the past 18 months. To which condition are these symptoms most likely related? a. Endometriosis b. PMS c. Primary dysmenorrhea d. Secondary dysmenorrhea

ANS: A Symptoms of endometriosis can change over time and may not reflect the extent of the disease. Major symptoms include dysmenorrhea and deep pelvic dyspareunia (painful intercourse). Impaired fertility may result from adhesions caused by endometriosis. Although endometriosis may be associated with secondary dysmenorrhea, it is not a cause of primary dysmenorrhea or PMS. In addition, this woman is complaining of dyspareunia and infertility, which are associated with endometriosis, not with PMS or primary or secondary dysmenorrhea.

Which information is an important consideration when comparing the CST with the NST? a. The NST has no known contraindications. b. The CST has fewer false-positive results when compared with the NST. c. The CST is more sensitive in detecting fetal compromise, as opposed to the NST. d. The CST is slightly more expensive than the NST.

ANS: A The CST has several contraindications. The NST has a high rate of false-positive results and is less sensitive than the CST but relatively inexpensive.

The various systems and organs of the fetus develop at different stages. Which statement is most accurate? a. Cardiovascular system is the first organ system to function in the developing human. b. Hematopoiesis originating in the yolk sac begins in the liver at 10 weeks of gestation. c. Body changes from straight to C-shape occurs at 8 weeks of gestation. d. Gastrointestinal system is mature at 32 weeks of gestation.

ANS: A The heart is developmentally complete by the end of the embryonic stage. Hematopoiesis begins in the liver during the sixth week. The body becomes C-shaped at 21 weeks of gestation. The gastrointestinal system is complete at 36 weeks of gestation.

A pregnant womans BPP score is 8. She asks the nurse to explain the results. How should the nurse respond at this time? a. The test results are within normal limits. b. Immediate delivery by cesarean birth is being considered. c. Further testing will be performed to determine the meaning of this score. d. An obstetric specialist will evaluate the results of this profile and, within the next week, will inform you of your options regarding delivery.

ANS: A The normal biophysical score ranges from 8 to 10 points if the amniotic fluid volume is adequate. A normal score allows conservative treatment of high-risk clients. Delivery can be delayed if fetal well-being is indicated. Scores less than 4 should be investigated, and delivery could be initiated sooner than planned. The results of the BPP are usually available immediately after the procedure is performed. Since this score is within normal range, no further testing is required at this time.

What are the complications and risks associated with cesarean births? (Select all that apply.) a.Pulmonary edema b.Wound dehiscence c.Hemorrhage d.Urinary tract infections e.Fetal injuries

ANS: A, B, C, D, E Pulmonary edema, wound dehiscence, hemorrhage, urinary tract infections, and fetal injuries are possible complications and risks associated with cesarean births.

At least five factors affect the process of labor and birth. These are easily remembered as the five Ps. Which factors are included in this process? (Select all that apply.) a. Passenger b. Passageway c. Powers d. Pressure e. Psychologic response

ANS: A, B, C, E The five Ps are passenger (fetus and placenta), passageway (birth canal), powers (contractions), position of the mother, and psychologic response. Pressure is not one of the five Ps.

Cellulitis with or without abscess formation is a fairly common condition. The nurse is providing education for a client whose presentation to the emergency department includes an infection of the breast. Which information should the nurse share with this client? (Select all that apply.) a.Nipple piercing may be the cause of a recent infection. b.Treatment for cellulitis will include antibiotics. c.Streptococcus aureus is the most common pathogen. d.Obesity, smoking, and diabetes are risk factors. e.Breast is pale in color and cool to the touch.

ANS: A, B, D The at-risk population for breast infection shares characteristics such as large breasts, obesity, previous surgeries, sebaceous cysts, smoking, diabetes, and recent nipple piercing. The most common pathogen isStaphylococcus aureus. Presentation of cellulitis of the breast includes pain, reddening, and warmth to the touch; treatment includes antibiotics and/or aspiration.

The client is undergoing treatment for ovarian cancer. Which common nutritional problems are related to gynecologic cancers and the treatment thereof? (Select all that apply.) a.Stomatitis b.Constipation c.Increased appetite d.Diarrhea e.Nausea and vomiting

ANS: A, B, D, E Altered taste, stomatitis, constipation, anorexia, diarrhea, and nausea and vomiting are all possible nutritional complications related to gynecologic cancers and their treatment. The nurse must assess accordingly and adapt the client's plan of care. To ensure recovery, these women should consume a diet high in iron and protein, drink plenty of fluids, and eat foods high in vitamins C, B, and K.

A client has requested information regarding alternatives to hormonal therapy for menopausal symptoms. Which current information should the nurse provide to the client? (Select all that apply.) a. Soy b. Vitamin C c. Vitamin K d. Vitamin E e. Vitamin A

ANS: A, D Both soy and vitamin E have been reported to help alleviate menopausal symptoms, and both are readily available in food sources. Vitamin E can be also be taken as a supplement. Vitamins C, K, and A have no apparent effect on menopausal symptoms.

Which statement concerning the third stage of labor is correct? a. The placenta eventually detaches itself from a flaccid uterus. b. An expectant or active approach to managing this stage of labor reduces the risk of complications. c. It is important that the dark, roughened maternal surface of the placenta appears before the shiny fetal surface. d. The major risk for women during the third stage is a rapid heart rate.

ANS: B Active management facilitates placental separation and expulsion, reducing the risk of complications. The placenta cannot detach itself from a flaccid (relaxed) uterus. Which surface of the placenta comes out first is not clinically important. The major risk for women during the third stage of labor is postpartum hemorrhaging.

A woman who is 16 weeks pregnant asks the nurse, Is it possible to tell by ultrasound if the baby is a boy or girl yet? What is the best answer? a. A babys sex is determined as soon as conception occurs. b. The baby has developed enough to enable us to determine the sex by examining the genitals through an ultrasound scan. c. Boys and girls look alike until approximately 20 weeks after conception, and then they begin to look different. d. It might be possible to determine your babys sex, but the external organs look very similar right now.

ANS: B Although gender is determined at conception, the external genitalia of males and females look similar through the ninth week. By the twelfth week, the external genitalia are distinguishable as male or female.

A perinatal nurse is giving discharge instructions to a woman, status postsuction, and curettage secondary to a hydatidiform mole. The woman asks why she must take oral contraceptives for the next 12 months. What is the bestresponse by the nurse? a."If you get pregnant within 1 year, the chance of a successful pregnancy is very small. Therefore, if you desire a future pregnancy, it would be better for you to use the most reliable method of contraception available." b."The major risk to you after a molar pregnancy is a type of cancer that can be diagnosed only by measuring the same hormone that your body produces during pregnancy. If you were to get pregnant, then it would make the diagnosis of this cancer more difficult." c."If you can avoid a pregnancy for the next year, the chance of developing a second molar pregnancy is rare. Therefore, to improve your chance of a successful pregnancy, not getting pregnant at this time is best." d."Oral contraceptives are the only form of birth control that will prevent a recurrence of a molar pregnancy."

ANS: B Beta-human chorionic gonadotropin (beta-hCG) hormone levels are drawn for 1 year to ensure that the mole is completely gone. The chance of developing choriocarcinoma after the development of a hydatidiform mole is increased. Therefore, the goal is to achieve a zero human chorionic gonadotropin (hCG) level. If the woman were to become pregnant, then it may obscure the presence of the potentially carcinogenic cells. Women should be instructed to use birth control for 1 year after treatment for a hydatidiform mole. The rationale for avoiding pregnancy for 1 year is to ensure that carcinogenic cells are not present. Any contraceptive method except an intrauterine device (IUD) is acceptable.

A client with maternal phenylketonuria (PKU) has come to the obstetrical clinic to begin prenatal care. Why would this preexisting condition result in the need for closer monitoring during pregnancy? a. PKU is a recognized cause of preterm labor. b. The fetus may develop neurologic problems. c. A pregnant woman is more likely to die without strict dietary control. d. Women with PKU are usually mentally handicapped and should not reproduce.

ANS: B Children born to women with untreated PKU are more likely to be born with mental retardation, microcephaly, congenital heart disease, and low birth weight. Maternal PKU has no effect on labor. Women without dietary control of PKU are more likely to miscarry or bear a child with congenital anomalies. Screening for undiagnosed maternal PKU at the first prenatal visit may be warranted, especially in individuals with a family history of the disorder, with low intelligence of an uncertain cause, or who have given birth to microcephalic infants.

A married couple is discussing alternatives for pregnancy prevention and has asked about fertility awareness methods (FAMs). Which response by the nurse is most appropriate? a. Theyre not very effective, and it is very likely that youll get pregnant. b. FAMs can be effective for many couples; however, they require motivation. c. These methods have a few advantages and several health risks. d. You would be much safer going on the pill and not having to worry

ANS: B FAMs are effective with proper vigilance about ovulatory changes in the body and with adherence to coitus intervals. FAMs are effective if correctly used by a woman with a regular menstrual cycle. The typical failure rate for all FAMs is 24% during the first year of use. FAMs have no associated health risks. The use of birth control has associated health risks. In addition, taking a pill daily requires compliance on the clients part.

During an inpatient psychiatric hospitalization, what is the most important nursing intervention? a. Contacting the clients significant other b. Supervising and guiding visits with her infant c. Allowing no contact with anyone who annoys her d. Having the infant with the mother at all times

ANS: B In the hospital setting, the reintroduction of the infant to the mother can and should occur at the mothers own pace. A schedule is set that increases the number of hours the mother cares for her infant over several days, culminating in the infant staying overnight in the mothers room. These supervised and guided visits allow the mother to experience meeting the infants needs and giving up sleep for the infant. Reintroducing the mother to her infant while in a supervised setting is essential. Another important task for a mother under psychiatric care is to reestablish positive interactions with others.

Which finding in the urinalysis of a pregnant woman is considered a variation of normal? a. Proteinuria b. Glycosuria c. Bacteria in the urine d. Ketones in the urine

ANS: B Small amounts of glucose may indicate physiologic spilling. The presence of protein could indicate kidney disease or preeclampsia. Urinary tract infections are associated with bacteria in the urine. An increase in ketones indicates that the patient is exercising too strenuously or has an inadequate fluid and food intake

A 48-year-old woman has just had a hysterectomy for endometrial cancer. Which statement alerts the nurse that further teaching is needed? a."I can't wait to go on the cruise that I have planned for this summer." b."I know that the surgery saved my life, but I will miss having sexual intercourse with my husband." c."I have asked my daughter to come and stay with me next week after I am discharged from the hospital." d."Well, I don't have to worry about getting pregnant anymore."

ANS: B Stating that she will miss having sexual intercourse with her husband indicates that further teaching is needed for this client regarding sexual activities after a hysterectomy. Although intercourse may be initially uncomfortable, the use of water-soluble lubricants, relaxation exercises, and changes in position may be helpful. Expressing plans for a vacation is a positive psychologic state with plans for the future. Stating that her daughter will stay with her indicates the client understands that she may need assistance during her acute recovery period. Stating that she no longer needs to worry about getting pregnant indicates knowledge related to the reproductive cycle and a positive outlook.

A 26-year-old pregnant woman, gravida 2, para 1-0-0-1, is 28 weeks pregnant when she experiences bright red, painless vaginal bleeding. On her arrival at the hospital, which diagnostic procedure will the client most likely have performed? a.Amniocentesis for fetal lung maturity b.Transvaginal ultrasound for placental location c.Contraction stress test (CST) d.Internal fetal monitoring

ANS: B The presence of painless bleeding should always alert the health care team to the possibility of placenta previa, which can be confirmed through ultrasonography. Amniocentesis is not performed on a woman who is experiencing bleeding. In the event of an imminent delivery, the fetus is presumed to have immature lungs at this gestational age, and the mother is given corticosteroids to aid in fetal lung maturity. A CST is not performed at a preterm gestational age. Furthermore, bleeding is a contraindication to a CST. Internal fetal monitoring is also contraindicated in the presence of bleeding.

Through a vaginal examination, the nurse determines that a woman is 4 cm dilated. The external fetal monitor shows uterine contractions every to 4 minutes. The nurse reports this as what stage of labor? a. First stage, latent phase b. First stage, active phase c. First stage, transition phase d. Second stage, latent phase

ANS: B This maternal progress indicates that the woman is in the active phase of the first stage of labor. During the latent phase of the first stage of labor, the expected maternal progress is 0 to 3 cm dilation with contractions every 5 to 30 minutes. During the transition phase of the first stage of labor, the expected maternal progress is 8 to 10 cm dilation with contractions every 2 to 3 minutes. During the latent phase of the second stage of labor, the woman is completely dilated and experiences a restful period of "laboring down."

What information is important for the nurse to include in planning for the care of a woman who has had a vaginal hysterectomy? a.Expect to be fully recovered in 4 to 6 weeks. b.Expect no changes in her hormone levels. c.Expect surgical menopause. d.Take tub baths to aid in healing.

ANS: B Unless the ovaries were also removed, hormonal levels should not change. Menses will cease, but the hypothalamus-pituitary-ovarian axis remains intact. The woman should expect to have vaginal discharge for 4 to 6 weeks. Full recovery varies from woman to woman, depending on risk factors and individual healing. Surgical menopause occurs only if the ovaries are also removed. The client should avoid tub baths, intercourse, and douching until after the follow-up examination.

Leiomyomas (also known as fibroid tumors) are benign tumors arising from the muscle tissue of the uterus. Which information related to these tumors is accurate? (Select all that apply.) a.Are rapid growing b.Are more common in African-American women c.Are more common in women who have never been pregnant d.Obesity is a risk factor with leiomyomas e.Become malignant if left untreated

ANS: B, C, D The exact cause of leiomyomas remains unknown, although genetic factors may be involved in their development. Most are found in the body of uterus and are classified according to their location on the uterine wall. They are benign, slow growing, and often spontaneously shrink after menopause.

What are the two primary functions of the ovary? (Select all that apply.) a. Normal female development b. Ovulation c. Sexual response d. Hormone production e. Sex hormone release

ANS: B, D The two functions of the ovaries are ovulation and hormone production. The presence of ovaries does not guarantee normal female development. The ovaries produce estrogen, progesterone, and androgen. Ovulation is the release of a mature ovum from the ovary. Sexual response is a feedback mechanism involving the hypothalamus, anterior pituitary gland, and ovaries.

Which nursing assessment indicates that a woman who is in second-stage labor is almost ready to give birth? a. Fetal head is felt at 0 station during vaginal examination. b. Bloody mucous discharge increases. c. Vulva bulges and encircles the fetal head. d. Membranes rupture during a contraction.

ANS: C A bulging vulva that encircles the fetal head describes crowning, which occurs shortly before birth. Birth of the head occurs when the station is +4. A 0 station indicates engagement. Bloody show occurs throughout the labor process and is not an indication of an imminent birth. ROM can occur at any time during the labor process and does not indicate an imminent birth.

What is the primary purpose for the use of tocolytic therapy to suppress uterine activity? a.Drugs can be efficaciously administered up to the designated beginning of term at 37 weeks gestation. b.Tocolytic therapy has no important maternal (as opposed to fetal) contraindications. c.The most important function of tocolytic therapy is to provide the opportunity to administer antenatal glucocorticoids. d.If the client develops pulmonary edema while receiving tocolytic therapy, then intravenous (IV) fluids should be given.

ANS: C Buying time for antenatal glucocorticoids to accelerate fetal lung development may be the best reason to use tocolytic therapy. Once the pregnancy has reached 34 weeks, however, the risks of tocolytic therapy outweigh the benefits. Important maternal contraindications to tocolytic therapy exist. Tocolytic-induced edema can be caused by IV fluids.

What condition indicates concealed hemorrhage when the client experiences abruptio placentae? a.Decrease in abdominal pain b.Bradycardia c.Hard, boardlike abdomen d.Decrease in fundal height

ANS: C Concealed hemorrhage occurs when the edges of the placenta do not separate. The formation of a hematoma behind the placenta and subsequent infiltration of the blood into the uterine muscle results in a very firm, boardlike abdomen. Abdominal pain may increase. The client will have shock symptoms that include tachycardia. As bleeding occurs, the fundal height increases.

Which description of the phases of the second stage of labor is most accurate? a. Latent phase: Feeling sleepy; fetal station 2+ to 4+; duration of 30 to 45 minutes b. Active phase: Overwhelmingly strong contractions; Ferguson reflex activated; duration of 5 to 15 minutes c. Descent phase: Significant increase in contractions; Ferguson reflex activated; average duration varies d. Transitional phase: Woman "laboring down"; fetal station 0; duration of 15 minutes

ANS: C The descent phase begins with a significant increase in contractions; the Ferguson reflex is activated, and the duration varies, depending on a number of factors. The latent phase is the lull or "laboring down" period at the beginning of the second stage and lasts 10 to 30 minutes on average. The second stage of labor has no active phase. The transition phase is the final phase in the second stage of labor; contractions are strong and painful.

If a clients normal prepregnancy diet contains 45 g of protein daily, how many more grams of protein should she consume per day during pregnancy? a. 5 b. 10 c. 25 d. 30

ANS: C The recommended intake of protein for the pregnant woman is 70 g. Therefore, additional protein intakes of 5, 10, or 15 g would be inadequate to meet protein needs during pregnancy. A protein intake of 30 g is more than would be necessary and would add extra calories.

Which consideration is essential for the nurse to understand regarding follow-up prenatal care visits? a. The interview portions become more intensive as the visits become more frequent over the course of the pregnancy. b. Monthly visits are scheduled for the first trimester, every 2 weeks for the second trimester, and weekly for the third trimester. c. During the abdominal examination, the nurse should be alert for supine hypotension. d. For pregnant women, a systolic BP of 130 mm Hg and a diastolic BP of 80 mm Hg is sufficient to be considered hypertensive.

ANS: C The woman lies on her back during the abdominal examination, possibly compressing the vena cava and aorta, which can cause a decrease in BP and a feeling of faintness. The interview portion of the follow-up examinations is less extensive than in the initial prenatal visits, during which so much new information must be gathered. Monthly visits are routinely scheduled for the first and second trimesters; visits increase to every 2 weeks at week 28 and to once a week at week 36. For pregnant women, hypertension is defined as a systolic BP of 140 mm Hg or higher and a diastolic BP of 90 mm Hg or higher.

A 25-year-old gravida 3, para 2 client gave birth to a 9-pound, 7-ounce boy, 4 hours ago after augmentation of labor with oxytocin (Pitocin). She presses her call light, and asks for her nurse right away, stating "I'm bleeding a lot." What is the most likely cause of postpartum hemorrhaging in this client? a. Retained placental fragments b. Unrepaired vaginal lacerations c. Uterine atony d. Puerperal infection

ANS: C This woman gave birth to a macrosomic infant after oxytocin augmentation. Combined with these risk factors, uterine atony is the most likely cause of bleeding 4 hours after delivery. Although retained placental fragments may cause postpartum hemorrhaging, it is typically detected within the first hour after delivery of the placenta and is not the most likely cause of the hemorrhaging in this woman. Although unrepaired vaginal lacerations may also cause bleeding, it typically occurs in the period immediately after birth. Puerperal infection can cause subinvolution and subsequent bleeding that is, however, typically detected 24 hours postpartum.

A woman arrives at the clinic for a pregnancy test. Her last menstrual period (LMP) was February 14, 2015. What is the clients expected date of birth (EDB)? a. September 17, 2015 b. November 7, 2015 c. November 21, 2015 d. December 17, 2015

ANS: C Using the Ngeles rule, the EDB is calculated by subtracting 3 months from the month of the LMP and adding 7 days + 1 year to the day of the LMP. Therefore, with an LMP of February 14, 2015, her due date is November 21, 2015. September 17, 2015, is too short a period to complete a normal pregnancy. Using the Ngeles rule, an EDB of November 7, 2015, is 2 weeks early. December 17, 2015, is almost a month past the correct EDB.

The nurse providing care for a high-risk laboring woman is alert for late FHR decelerations. Which clinical finding might be the cause for these late decelerations? a.Altered cerebral blood flow b.Umbilical cord compression c.Uteroplacental insufficiency d.Meconium fluid

ANS: C Uteroplacental insufficiency results in late FHR decelerations. Altered fetal cerebral blood flow results in early FHR decelerations. Umbilical cord compression results in variable FHR decelerations. Meconium-stained fluid may or may not produce changes in the FHR, depending on the gestational age of the fetus and whether other causative factors associated with fetal distress are present.

Which definition of an acceleration in the fetal heart rate (FHR) is accurate? a.FHR accelerations are indications of fetal well-being when they are periodic. b.FHR accelerations are greater and longer in preterm gestations. c.FHR accelerations are usually observed with breech presentations when they are episodic. d.An acceleration in the FHR presents a visually apparent and abrupt peak.

ANS: D Acceleration of the FHR is defined as a visually apparent abrupt (only to peak 30 seconds) increase in the FHR above the baseline rate. Periodic accelerations occur with uterine contractions and are usually observed with breech presentations. Episodic accelerations occur during fetal movement and are indications of fetal well-being. Preterm accelerations peak at 10 beats per minute above the baseline and last for at least 10 seconds.

The major source of nutrients in the diet of a pregnant woman should be composed of what? a. Simple sugars b. Fats c. Fiber d. Complex carbohydrates

ANS: D Complex carbohydrates supply the pregnant woman with vitamins, minerals, and fiber. The most common simple carbohydrate is table sugar, which is a source of energy but does not provide any nutrients. Fats provide 9 kcal in each gram, in contrast to carbohydrates and proteins, which provide only 4 kcal in each gram. Fiber is primarily supplied by complex carbohydrates

A woman who is 8 months pregnant asks the nurse, Does my baby have any antibodies to fight infection? What is the most appropriate response by the nurse? a. Your baby has all the immunoglobulins necessary: immunoglobulin G (IgG), immunoglobulin M (IgM), and immunoglobulin A (IgA). b. Your baby wont receive any antibodies until he is born and you breastfeed him. c. Your baby does not have any antibodies to fight infection. d. Your baby has IgG and IgM.

ANS: D During the third trimester, IgG is the only immunoglobulin that crosses the placenta; it provides passive acquired immunity to specific bacterial toxins. However, the fetus produces IgM by the end of the first trimester. IgA immunoglobulins are not produced by the baby. Therefore, by the third trimester, the fetus has both IgG and IgM. Breastfeeding supplies the newborn infant with IgA.

A nurse is providing education to a support group of women newly diagnosed with breast cancer. It is important for the nurse to discuss which factor related to breast cancer with the group? a.Genetic mutations account for 50% of women who will develop breast cancer. b.Breast cancer is the leading cause of cancer death in women. c.In the United States, 1 in 10 women will develop breast cancer in her lifetime. d.The exact cause of breast cancer remains unknown.

ANS: D The exact cause of breast cancer is unknown. Between 15% and 20% of these cancers are related to genetic mutations. Breast cancer is the second leading cause of cancer death in woman ages 45 to 55 years. One in eight women in the United States will develop breast cancer in her lifetime.

A man calls the nurse's station stating that his wife, who delivered 2 days ago, is happy one minute and crying the next. The man says, "She was never like this before the baby was born." The nurse's initial response should be to a. Tell him to ignore the mood swings, as they will go away. b. Reassure him that this behavior is normal. c. Advise him to get immediate psychological help for her. d. Instruct him in the signs, symptoms, and duration of postpartum blues.

B

When caring for a newly delivered woman, the nurse is aware that the best measure to prevent abdominal distention after a cesarean birth is a. Rectal suppositories b. Early and frequent ambulation c. Tightening and relaxing abdominal muscles d. Carbonated beverages

B

A postpartum woman overhears the nurse tell the obstetrics clinician that she has a positive Homans sign and asks what it means. The nurse's best response is: a."You have pitting edema in your ankles." b."You have deep tendon reflexes rated 2+." c."You have calf pain when the nurse flexes your foot." d."You have a 'fleshy' odor to your vaginal drainage."

C

What documentation on a woman's chart on postpartum day 14 indicates a normal involution process? a. Moderate bright red lochial flow b. Breasts firm and tender c. Fundus below the symphysis and not palpable d. Episiotomy slightly red and puffy

C

Which maternal event is abnormal in the early postpartum period? a. Diuresis and diaphoresis b. Flatulence and constipation c. Extreme hunger and thirst d. Lochial color changes from rubra to alba

D

When is a prophylactic cerclage for an incompetent cervix usually placed (in weeks of gestation)? a.12 to 14 b.6 to 8 c.23 to 24 d.After 24

ANS: A A prophylactic cerclage is usually placed at 12 to 14 weeks of gestation. The cerclage is electively removed when the woman reaches 37 weeks of gestation or when her labor begins. Six to 8 weeks of gestation is too early to place the cerclage. Cerclage placement is offered if the cervical length falls to less than 20 to 25 mm before 23 to 24 weeks. Although no consensus has been reached, 24 weeks is used as the upper gestational age limit for cerclage placement

The nurse should be cognizant of which physiologic effect of pain? a.Predominant pain of the first stage of labor is visceral pain that is located in the lower portion of the abdomen. b.Referred pain is the extreme discomfort experienced between contractions. c.Somatic pain of the second stage of labor is more generalized and related to fatigue. d.Pain during the third stage is a somewhat milder version of the pain experienced during the second stage.

ANS: A Predominant pain comes from cervical changes, the distention of the lower uterine segment, and uterine ischemia. Referred pain occurs when the pain that originates in the uterus radiates to the abdominal wall, lumbosacral area of the back, iliac crests, and gluteal area. Second-stage labor pain is intense, sharp, burning, and localized. Third-stage labor pain is similar to that of the first stage.

What type of cultural concern is the most likely deterrent to many women seeking prenatal care? a. Religion b. Modesty c. Ignorance d. Belief that physicians are evil

ANS: B A concern for modesty is a deterrent to many women seeking prenatal care. For some women, exposing body parts, especially to a man, is considered a major violation of their modesty. Many cultural variations are found in prenatal care. Even if the prenatal care described is familiar to a woman, some practices may conflict with the beliefs and practices of a subculture group to which she belongs.

What is the correct terminology for the nerve block that provides anesthesia to the lower vagina and perineum? a.Epidural b.Pudendal c.Local d.Spinal block

ANS: B A pudendal block anesthetizes the lower vagina and perineum to provide anesthesia for an episiotomy and the use of low forceps, if needed. An epidural provides anesthesia for the uterus, perineum, and legs. A local provides anesthesia for the perineum at the site of the episiotomy. A spinal block provides anesthesia for the uterus, perineum, and down the legs.

A new mother with a thyroid disorder has come for a lactation follow-up appointment. Which thyroid disorder is a contraindication for breastfeeding? a. Hyperthyroidism b. PKU c. Hypothyroidism d. Thyroid storm

ANS: B PKU is a cause of mental retardation in infants; mothers with PKU pass on phenylalanine and therefore should elect not to breastfeed. A woman with either hyperthyroidism or hypothyroidism would have no particular reason not to breastfeed. A thyroid storm is a complication of hyperthyroidism and is not a contraindication to breastfeeding.

The best measure to prevent abdominal distention following a cesarean birth is A. Rectal suppositories B. Early and frequent ambulation C. Tightening and relaxing abdominal muscles D. Carbonated beverages

B

The postpartum woman who continually repeats the story of her labor, delivery, and recovery experiences is a. Providing others with her knowledge of events b. Making the birth experience "real" c. Taking hold of the events leading to her labor and delivery d.Accepting her response to labor and delivery

B

Thirty minutes after delivery, the nurse can expect to palpate the fundus A. At the umbilicus B. Midway between the symphysis pubis and umbilicus C. At the symphysis pubis D. Above the umbilicus

B

A nurse is discussing the signs and symptoms of mastitis with a mother who is breastfeeding. Which findings should the nurse include in the discussion? (Select all that apply.) a. Breast tenderness b. Warmth in the breast c. Area of redness on the breast often resembling the shape of a pie wedge d. Small white blister on the tip of the nipple e. Fever and flulike symptoms

a. Breast tenderness b. Warmth in the breast c. Area of redness on the breast often resembling the shape of a pie wedge e. Fever and flulike symptoms

What is the most common medical complication of pregnancy? a. Hypertension b. Hyperemesis gravidarum c. Hemorrhagic complications d. Infections

ANS: A Preeclampsia and eclampsia are two noted deadly forms of hypertension. A large percentage of pregnant women will have nausea and vomiting, but a relatively few will have the severe form called hyperemesis gravidarum. Hemorrhagic complications are the second most common medical complication of pregnancy; hypertension is the most common. Infection is a risk factor for preeclampsia.

Women who have participated in childbirth education classes often bring a birth plan with them to the hospital. Which items might this plan include? (Select all that apply.) a. Presence of companions b. Clothing to be worn c. Care and handling of the newborn d. Medical interventions e. Date of delivery

ANS: A, B, C, D The presence of companions, clothing to be worn, care and handling of the newborn, medical interventions, and environmental modifications all might be included in the couple's birth plan. Other items include the presence of nonessential medical personnel (students), labor activities such as the tub or ambulation, preferred comfort and relaxation methods, and any cultural or religious requirements. The expected date of delivery would not be part of a birth plan unless the client is scheduled for an elective cesarean birth.

The nurse should be aware that a pessary is most effective in the treatment of which disorder? a.Cystocele b.Uterine prolapse c.Rectocele d.Stress urinary incontinence

ANS: B A fitted pessary may be inserted into the vagina to support the uterus and hold it in the correct position. Usually a pessary is used for only a short time and is not used for the client with a cystocele. A rectocele cannot be corrected by the use of a pessary. A pessary is not likely the most effective treatment for stress incontinence.

During labor a fetus displays an average FHR of 135 beats per minute over a 10-minute period. Which statement best describes the status of this fetus? a.Bradycardia b.Normal baseline heart rate c.Tachycardia d.Hypoxia

ANS: B The baseline FHR is measured over 10 minutes; a normal range is 110 to 160 beats per minute. Bradycardia is a FHR less than 110 beats per minute for 10 minutes or longer. Tachycardia is a FHR higher than 160 beats per minutes for 10 minutes or longer. Hypoxia is an inadequate supply of oxygen; no indication of hypoxia exists with a baseline FHR in the normal range.

Cardiac output increases from 30% to 50% by the 32nd week of pregnancy. What is the rationale for this change? a. To compensate for the decreased renal plasma flow b. To provide adequate perfusion of the placenta c. To eliminate metabolic wastes of the mother d. To prevent maternal and fetal dehydration

ANS: B The primary function of increased vascular volume is to transport oxygen and nutrients to the fetus via the placenta. Renal plasma flow increases during pregnancy. Assisting with pulling metabolic wastes from the fetus for maternal excretion is one purpose of the increased vascular volume.

Which statement regarding multifetal pregnancy is incorrect? a. The expectant mother often develops anemia because the fetuses have a greater demand for iron. b. Twin pregnancies come to term with the same frequency as single pregnancies. c. The mother should be counseled to increase her nutritional intake and gain more weight. d. Backache and varicose veins often are more pronounced with a multifetal pregnancy.

ANS: B Twin pregnancies often end in prematurity. Serious efforts should be made to bring the pregnancy to term. A woman with a multifetal pregnancy often develops anemia, suffers more or worse backache, and needs to gain more weight. Counseling is needed to help her adjust to these conditions.

Which preexisting factor is known to increase the risk of GDM? a. Underweight before pregnancy b. Maternal age younger than 25 years c. Previous birth of large infant d. Previous diagnosis of type 2 diabetes mellitus

ANS: C A previous birth of a large infant suggests GDM. Obesity (body mass index [BMI] of 30 or greater) creates a higher risk for gestational diabetes. A woman younger than 25 years is not generally at risk for GDM. The person with type 2 diabetes mellitus already has diabetes and thus will continue to have it after pregnancy. Insulin may be required during pregnancy because oral hypoglycemia drugs are contraindicated during pregnancy.

A pregnant woman at 25 weeks of gestation tells the nurse that she dropped a pan last week and her baby jumped at the noise. Which response by the nurse is most accurate? a. That must have been a coincidence; babies cant respond like that. b. The fetus is demonstrating the aural reflex. c. Babies respond to sound starting at approximately 24 weeks of gestation. d. Let me know if it happens again; we need to report that to your midwife.

ANS: C Babies respond to external sound starting at approximately 24 weeks of gestation. Acoustic stimulations can evoke a fetal heart rate response. There is no such thing as an aural reflex. The last statement is inappropriate and may cause undue psychologic alarm to the client.

Which statement concerning cyclic perimenstrual pain and discomfort (CPPD) is accurate? a. Premenstrual dysphoric disorder (PMDD) is a milder form of PMS and more common in young women. b. Secondary dysmenorrhea is more intense and more medically significant than primary dysmenorrhea. c. PMS is a complex, poorly understood condition that may include any of a hundred symptoms. d. The causes of PMS have been well established.

ANS: C PMS may manifest itself with one or more of a hundred physical and psychologic symptoms. PDD is a more severe variant of PMS. Secondary dysmenorrhea is characterized by more muted pain than the pain reported in primary dysmenorrhea; however, the medical treatment is close to the same. The cause of PMS is unknown and may be, in fact, a collection of different problems.

Which statement most accurately describes the HELLP syndrome? a. Mild form of preeclampsia b. Diagnosed by a nurse alert to its symptoms c. Characterized by hemolysis, elevated liver enzymes, and low platelets d. Associated with preterm labor but not perinatal mortality

ANS: C The acronym HELLP stands for hemolysis (H), elevated liver (EL) enzymes, and low platelets (LP). The HELLP syndrome is a variant of severe preeclampsia and is difficult to identify because the symptoms are not often obvious. The HELLP syndrome must be diagnosed in the laboratory. Preterm labor is greatly increased; therefore, so is perinatal mortality.

A labor and delivery nurse should be cognizant of which information regarding how the fetus moves through the birth canal? a. Fetal attitude describes the angle at which the fetus exits the uterus. b. Of the two primary fetal lies, the horizontal lie is that in which the long axis of the fetus is parallel to the long axis of the mother. c. Normal attitude of the fetus is called general flexion. d. Transverse lie is preferred for vaginal birth.

ANS: C The normal attitude of the fetus is called general flexion. The fetal attitude is the relationship of the fetal body parts to each one another. The horizontal lie is perpendicular to the mother; in the longitudinal (or vertical) lie, the long axes of the fetus and the mother are parallel. Vaginal birth cannot occur if the fetus stays in a transverse lie.

A woman with gestational diabetes has had little or no experience reading and interpreting glucose levels. The client shows the nurse her readings for the past few days. Which reading signals the nurse that the client may require an adjustment of insulin or carbohydrates? a. 75 mg/dl before lunch. This is low; better eat now. b. 115 mg/dl 1 hour after lunch. This is a little high; maybe eat a little less next time. c. 115 mg/dl 2 hours after lunch. This is too high; it is time for insulin. d. 50 mg/dl just after waking up from a nap. This is too low; maybe eat a snack before going to sleep.

ANS: D 50 mg/dl after waking from a nap is too low. During hours of sleep, glucose levels should not be less than 60 mg/dl. Snacks before sleeping can be helpful. The premeal acceptable range is 60 to 99 mg/dl. The readings 1 hour after a meal should be less than 129 mg/dl. Two hours after eating, the readings should be less than 120 mg/dl.

What is the importance of obtaining informed consent for a number of contraceptive methods? a. Contraception is an invasive procedure that requires hospitalization. b. The method may require a surgical procedure to insert a device. c. The contraception method chosen may be unreliable. d. The method chosen has potentially dangerous side effects.

ANS: D Being aware of the potential side effects is important for couples who are making an informed decision about the use of contraceptives. The only contraceptive method that is a surgical procedure and requires hospitalization is sterilization. Some methods have greater efficacy than others, and this efficacy should be included in the teaching.

When would an internal version be indicated to manipulate the fetus into a vertex position? a.Fetus from a breech to a cephalic presentation before labor begins b.Fetus from a transverse lie to a longitudinal lie before a cesarean birth c.Second twin from an oblique lie to a transverse lie before labor begins d.Second twin from a transverse lie to a breech presentation during a vaginal birth

ANS: D Internal version is used only during a vaginal birth to manipulate the second twin into a presentation that allows it to be vaginally born. For internal version to occur, the cervix needs to be completely dilated.

How does the nurse document a NST during which two or more FHR accelerations of 15 beats per minute or more occur with fetal movement in a 20-minute period? a. Nonreactive b. Positive c. Negative d. Reactive

ANS: D The NST is reactive (normal) when two or more FHR accelerations of at least 15 beats per minute (each with a duration of at least 15 seconds) occur in a 20-minute period. A nonreactive result means that the heart rate did not accelerate during fetal movement. A positive result is not used with NST. CST uses positive as a result term. A negative result is not used with NST. CST uses negative as a result term.

In evaluating the effectiveness of magnesium sulfate for the treatment of preterm labor, which finding alerts the nurse to possible side effects? a.Urine output of 160 ml in 4 hours b.DTRs 2+ and no clonus c.Respiratory rate (RR) of 16 breaths per minute d.Serum magnesium level of 10 mg/dl

ANS: D The therapeutic range for magnesium sulfate management is 4 to 7.5 mg/dl. A serum magnesium level of 10 mg/dl could lead to signs and symptoms of magnesium toxicity, including oliguria and respiratory distress. Urine output of 160 ml in 4 hours, DTRs of 2+, and a RR of 16 breaths per minute are all normal findings.

A pregnant patient asks when the dark line on her abdomen (linea nigra) will go away. The nurse knows the pigmentation will decrease after delivery because of a. Increased estrogen b. Increased progesterone c. Decreased melanocyte-stimulating hormone d. Decreased human placental lactogen

C

A client has been prescribed adjuvant tamoxifen therapy. What common side effect might she experience? a.Weight gain, hot flashes, and blood clots b.Vomiting, weight loss, and hair loss c.Nausea, vomiting, and diarrhea d.Hot flashes, weight gain, and headaches

ANS: A Common side effects of tamoxifen therapy include hot flashes, weight gain, and blood clots. Weight loss, hair loss, diarrhea, and headaches are not common side effects of tamoxifen.

Why might it be more difficult to diagnose appendicitis during pregnancy? a. The appendix is displaced upward and laterally, high and to the right. b. The appendix is displaced upward and laterally, high and to the left. c. The appendix is deep at the McBurneys point. d. The appendix is displaced downward and laterally, low and to the right.

ANS: A The appendix is displaced high and to the right, not to the left. It is displaced beyond the McBurneys point and is not displaced in a downward direction.

3. The nurse is caring for a client in early labor. Membranes ruptured approximately 2 hours earlier. This client is at increased risk for which complication? a. Intrauterine infection b. Hemorrhage c. Precipitous labor d. Supine hypotension

ANS: A When the membranes rupture, microorganisms from the vagina can ascend into the amniotic sac, causing chorioamnionitis and placentitis. ROM is not associated with fetal or maternal bleeding. Although ROM may increase the intensity of the contractions and facilitate active labor, it does not result in precipitous labor. ROM has no correlation with supine hypotension.

Which alternative approaches to relaxation have proven successful when working with the client in labor? (Select all that apply.) a.Aromatherapy b.Massage c.Hypnosis d.Cesarean birth e.Biofeedback

ANS: A, B, C, E Approaches to relaxation can include neuromuscular relaxation, aromatherapy, music, massage, imagery, hypnosis, or touch relaxation. Cesarean birth is a method of delivery, not a method of relaxation.

Which statement concerning neurologic and sensory development in the fetus is correct? a. Brain waves have been recorded on an electroencephalogram as early as the end of the first trimester (12 weeks of gestation). b. Fetuses respond to sound by 24 weeks of gestation and can be soothed by the sound of the mothers voice. c. Eyes are first receptive to light at 34 to 36 weeks of gestation. d. At term, the fetal brain is at least one third the size of an adult brain.

ANS: B Hearing develops early and is fully developed at birth. Brain waves have been recorded at week 8. Eyes are receptive to light at 28 weeks of gestation. The fetal brain is approximately one fourth the size of an adult brain.

A male client asks the nurse why it is better to purchase condoms that are not lubricated with nonoxynol-9 (a common spermicide). Which response by the nurse is the most accurate? a. The lubricant prevents vaginal irritation. b. Nonoxynol-9 does not provide protection against STIs as originally thought; it has also been linked to an increase in the transmission of the HIV and can cause genital lesions. c. The additional lubrication improves sex. d. Nonoxynol-9 improves penile sensitivity.

ANS: B Nonoxynol-9 does not provide protection against STIs as originally thought; it has also been linked to an increase in the transmission of the HIV and can cause genital lesions. Nonoxynol-9 may cause vaginal irritation, has no effect on the quality of sexual activity, and has no effect on penile sensitivity.

The nurse is providing health education to a pregnant client regarding the cardiovascular system. Which information is correct and important to share? a. A pregnant woman experiencing disturbed cardiac rhythm, such as sinus arrhythmia, requires close medical and obstetric observation no matter how healthy she may appear otherwise. b. Changes in heart size and position and increases in blood volume create auditory changes from 20 weeks of gestation to term. c. Palpitations are twice as likely to occur in twin gestations. d. All of the above changes will likely occur.

ANS: B These auscultatory changes should be discernible after 20 weeks of gestation. A healthy woman with no underlying heart disease does not need any therapy. The maternal heart rate increases in the third trimester, but palpitations may not necessarily occur, let alone double. Auditory changes are discernible at 20 weeks of gestation.

Autoimmune disorders often occur during pregnancy because a large percentage of women with an autoimmune disorder are of childbearing age. Which disorders fall into the category of collagen vascular disease? (Select all that apply.) a. Multiple sclerosis b. SLE c. Antiphospholipid syndrome d. Rheumatoid arthritis e. Myasthenia gravis

ANS: B, C, D, E Multiple sclerosis is not an autoimmune disorder. This patchy demyelination of the spinal cord may be a viral disorder. Autoimmune disorders (collagen vascular disease) make up a large group of conditions that disrupt the function of the immune system of the body. These disorders include those listed, as well as systemic sclerosis.

Which statement is not an expected outcome for the client who attends a reputable childbirth preparation program? a.Childbirth preparation programs increase the woman's sense of control. b.Childbirth preparation programs prepare a support person to help during labor. c.Childbirth preparation programs guarantee a pain-free childbirth. d.Childbirth preparation programs teach distraction techniques.

ANS: C All methods try to increase a woman's sense of control, prepare a support person, and train the woman in physical conditioning, which includes breathing techniques. These programs cannot, and reputable ones do not, promise a pain-free childbirth. Increasing a woman's sense of control is the goal of all childbirth preparation methods. Preparing a support person to help in labor is a vitally important component of any childbirth education program. The coach may learn how to touch a woman's body to detect tense and contracted muscles. The woman then learns how to relax in response to the gentle stroking by the coach. Distraction techniques are a form of care that are effective to some degree in relieving labor pain and are taught in many childbirth programs. These distractions include imagery, feedback relaxation, and attention-focusing behaviors.

At 35 weeks of pregnancy, a woman experiences preterm labor. Although tocolytic medications are administered and she is placed on bed rest, she continues to experience regular uterine contractions and her cervix is beginning to dilate and efface. What is an important test for fetal well-being at this time? a. PUBS b. Ultrasound for fetal size c. Amniocentesis for fetal lung maturity d. NST

ANS: C Amniocentesis is performed to assess fetal lung maturity in the event of a preterm birth. The fluid is examined to determine the lecithin to sphingomyelin (L/S) ratio. Indications for PUBS include prenatal diagnosis or inherited blood disorders, karyotyping of malformed fetuses, detection of fetal infection, determination of the acid-base status of the fetus with IUGR, and assessment and treatment of isoimmunization and thrombocytopenia in the fetus. Determination of fetal size by ultrasound is typically performed during the second trimester and is not indicated in this scenario. An NST measures the fetal response to fetal movement in a noncontracting mother.

In which clinical situation would the nurse most likely anticipate a fetal bradycardia? a.Intraamniotic infection b.Fetal anemia c.Prolonged umbilical cord compression d.Tocolytic treatment using terbutaline

ANS: C Fetal bradycardia can be considered a later sign of fetal hypoxia and is known to occur before fetal death. Bradycardia can result from placental transfer of drugs, prolonged compression of the umbilical cord, maternal hypothermia, and maternal hypotension. Intraamniotic infection, fetal anemia, and tocolytic treatment using terbutaline would most likely result in fetal tachycardia.

Postpartal overdistention of the bladder and urinary retention can lead to which complication? a. Postpartum hemorrhage and eclampsia b. Fever and increased blood pressure c. Postpartum hemorrhage and urinary tract infection d. Urinary tract infection and uterine rupture

C

A newly delivered mother who intends to breastfeed tells her nurse, "I am so relieved that this pregnancy is over so that I can start smoking again." The nurse encourages the client to refrain from smoking. However, this new mother is insistent that she will resume smoking. How will the nurse adapt her health teaching with this new information? a. Smoking has little-to-no effect on milk production. b. No relationship exists between smoking and the time of feedings. c. The effects of secondhand smoke on infants are less significant than for adults. d. The mother should always smoke in another room.

d. The mother should always smoke in another room.

A woman has preinvasive cancer of the cervix. Which modality would the nurse discuss as an available option for a client with this condition? a.Cryosurgery b.Colposcopy c.Hysterectomy d.Internal radiation

ANS: A Cryosurgery, laser surgery, and loop electrosurgical excision procedure (LEEP) are several techniques used to treat preinvasive lesions. Colposcopy is the examination of the cervix with a stereoscopic binocular microscope that magnifies the view of the cervix. This examination would have already been performed as part of the diagnosis of preinvasive cancer of the cervix. A hysterectomy is performed if the cancer has extended beyond the cervix. Women with positive pelvic nodes (indicating invasive cancer) usually receive whole pelvis irradiation.

The nurse should be aware of which information related to a woman's intake and output during labor? a. Traditionally, restricting the laboring woman to clear liquids and ice chips is being challenged because regional anesthesia is used more often than general anesthesia. b. Intravenous (IV) fluids are usually necessary to ensure that the laboring woman stays hydrated. c. Routine use of an enema empties the rectum and is very helpful for producing a clean, clear delivery. d. When a nulliparous woman experiences the urge to defecate, it often means birth will quickly follow.

ANS: A Women are awake with regional anesthesia and are able to protect their own airway, which reduces the worry over aspiration. Routine IV fluids during labor are unlikely to be beneficial and may be harmful. The routine use of an enema is, at best, ineffective and may be harmful. Having the urge to defecate followed by the birth of her fetus is true for a multiparous woman but not for a nulliparous woman

Transvaginal ultrasonography is often performed during the first trimester. While preparing a 6-week gestational client for this procedure, she expresses concerns over the necessity for this test. The nurse should explain that this diagnostic test may be indicated for which situations? (Select all that apply.) a. Multifetal gestation b. Obesity c. Fetal abnormalities d. Amniotic fluid volume e. Ectopic pregnancy

ANS: A, B, C, E Transvaginal ultrasound is useful in women who are obese whose thick abdominal layers cannot be penetrated with traditional abdominal ultrasound. This procedure is also used to identify multifetal gestation, ectopic pregnancy, estimating gestational age, confirming fetal viability, and identifying fetal abnormalities. Amniotic fluid volume is assessed during the second and third trimester; conventional ultrasound would be used.

A woman is in for a routine prenatal checkup. The nurse is assessing her urine for glycosuria and proteinuria. Which findings are considered normal? (Select all that apply.) a. Dipstick assessment of trace to +1 b. <300 mg/24 hours c. Dipstick assessment of +2 glucose d. >300 mg/24 hours e. Albumin < 30 mg/24 hours

ANS: A, B, E Small amounts of protein in the urine are acceptable during pregnancy. The presence of protein in greater amounts may indicate renal problems. A dipstick assessment of +2 and proteinuria >300 mg/24 hours, and albuminuria greater than 30 mg/24 hours are excessive and should be further evaluated.

Which information regarding substance abuse is important for the nurse to understand? a. Although cigarette smoking causes a number of health problems, it has little direct effect on maternity-related health. b. Women, ages 21 to 34 years, have the highest rates of specific alcohol-related problems. c. Coffee is a stimulant that can interrupt body functions and has been related to birth defects. d. Prescription psychotherapeutic drugs taken by the mother do not affect the fetus; otherwise, they would not have been prescribed.

ANS: B Although a very small percentage of childbearing women have alcohol-related problems, alcohol abuse during pregnancy has been associated with a number of negative outcomes. Cigarette smoking impairs fertility and is a cause of low-birth-weight infants. Caffeine consumption has not been related to birth defects. Psychotherapeutic drugs have some effect on the fetus, and that risk must be weighed against their benefit to the mother.

What is the primary nursing responsibility when caring for a client who is experiencing an obstetric hemorrhage associated with uterine atony? a.Establishing venous access b.Performing fundal massage c.Preparing the woman for surgical intervention d.Catheterizing the bladder

ANS: B The initial management of excessive postpartum bleeding is a firm massage of the uterine fundus. Although establishing venous access may be a necessary intervention, fundal massage is the initial intervention. The woman may need surgical intervention to treat her postpartum hemorrhage, but the initial nursing intervention is to assess the uterus. After uterine massage, the nurse may want to catheterize the client to eliminate any bladder distention that may be preventing the uterus from properly contracting.

Which clinical finding indicates that the client has reached the second stage of labor? a. Amniotic membranes rupture. b. Cervix cannot be felt during a vaginal examination. c. Woman experiences a strong urge to bear down. d. Presenting part of the fetus is below the ischial spines.

ANS: C During the descent phase of the second stage of labor, the woman may experience an increase in the urge to bear down. The ROM has no significance in determining the stage of labor. The second stage of labor begins with full cervical dilation. Many women may have an urge to bear down when the presenting fetal part is below the level of the ischial spines. This urge can occur during the first stage of labor, as early as with 5 cm dilation.

28. The first 1 to 2 hours after birth is sometimes referred to as what? a. Bonding period b. Third stage of labor c. Fourth stage of labor d. Early postpartum period

ANS: C The first 2 hours of the birth are a critical time for the mother and her baby and is often called the fourth stage of labor. Maternal organs undergo their initial readjustment to a nonpregnant state. The third stage of labor lasts from the birth of the baby to the expulsion of the placenta. Bonding will occur over a much longer period, although it may be initiated during the fourth stage of labor.

Which statement correctly describes the effects of various pain factors? a.Higher prostaglandin levels arising from dysmenorrhea can blunt the pain of childbirth. b.Upright positions in labor increase the pain factor because they cause greater fatigue. c.Women who move around trying different positions experience more pain. d.Levels of pain-mitigating beta-endorphins are higher during a spontaneous, natural childbirth.

ANS: D Higher endorphin levels help women tolerate pain and reduce anxiety and irritability. Higher prostaglandin levels correspond to more severe labor pains. Upright positions in labor usually result in improved comfort and less pain. Moving freely to find more comfortable positions is important for reducing pain and muscle tension.

What is the most dangerous effect on the fetus of a mother who smokes cigarettes while pregnant? a. Genetic changes and anomalies b. Extensive CNS damage c. Fetal addiction to the substance inhaled d. Intrauterine growth restriction

ANS: D The major consequences of smoking tobacco during pregnancy are low-birth-weight infants, prematurity, and increased perinatal loss. Cigarettes will not normally cause genetic changes or extensive CNS damage. Addiction to tobacco is not a usual concern related to the neonate.

The nurse caring for the postpartum woman understands that breast engorgement is caused by: a. Overproduction of colostrum b. Accumulation of milk in the lactiferous ducts and glands c. Hyperplasia of mammary tissue d. Congestion of veins and lymphatics

D

Which condition, not uncommon in pregnancy, is likely to require careful medical assessment during the puerperium? a. Varicosities of the legs b. Carpal tunnel syndrome c. Periodic numbness and tingling of the fingers d. Headaches

D

Individual irregularities in the ovarian (menstrual) cycle are most often caused by what? a. Variations in the follicular (preovulatory) phase b. Intact hypothalamic-pituitary feedback mechanism c. Functioning corpus luteum d. Prolonged ischemic phase

ANS: A Almost all variations in the length of the ovarian cycle are the result of variations in the length of the follicular phase. An intact hypothalamic-pituitary feedback mechanism would be regular, not irregular. The luteal phase begins after ovulation. The corpus luteum is dependent on the ovulatory phase and fertilization. During the ischemic phase, the blood supply to the functional endometrium is blocked, and necrosis develops. The functional layer separates from the basal layer, and menstrual bleeding begins.

A client has arrived for her first prenatal appointment. She asked the nurse to explain exactly how long the pregnancy will be. What is the nurses best response? a. Normal pregnancy is 10 lunar months. b. Pregnancy is made up of four trimesters. c. Pregnancy is considered term at 36 weeks. d. Estimated date of delivery (EDD) is 40 completed weeks.

ANS: A Pregnancy spans 9 calendar months; but, health care providers prefer to use the lunar month of 28 days or 4 weeks. Pregnancy consists of three trimesters, each approximately 13 weeks long. A pregnancy is considered term at 37 completed weeks; however, EDD is based upon 40 weeks of gestation.

Pregnant adolescents are at greater risk for decreased BMI and fad dieting with which condition? a. Obesity b. Gestational diabetes c. Low-birth-weight babies d. High-birth-weight babies

ANS: C Adolescents tend to have lower BMIs. In addition, the fetus and the still-growing mother appear to compete for nutrients. These factors, along with inadequate weight gain, lend themselves to a higher incidence of low-birth-weight babies. Obesity is associated with a higher-than-normal BMI. Unless the teenager has type 1 diabetes, an adolescent with a low BMI is less likely to develop gestational diabetes. High-birth-weight or large-for-gestational age (LGA) babies are most often associated with gestational diabetes.

Which client would be an ideal candidate for injectable progestins such as Depo-Provera (DMPA) as a contraceptive choice? a. The ideal candidate for DMPA wants menstrual regularity and predictability. b. The client has a history of thrombotic problems or breast cancer. c. The ideal candidate has difficulty remembering to take oral contraceptives daily. d. The client is homeless or mobile and rarely receives health care.

ANS: C Advantages of DMPA include its contraceptive effectiveness, compared with the effectiveness of combined oral contraceptives, and the requirement of only four injections a year. The disadvantages of injectable progestins are prolonged amenorrhea and uterine bleeding. The use of injectable progestin carries an increased risk of venous thrombosis and thromboembolism. To be effective, DMPA injections must be administered every 11 to 13 weeks. Access to health care is necessary to prevent pregnancy or potential complications.

To manage her diabetes appropriately and to ensure a good fetal outcome, how would the pregnant woman with diabetes alter her diet? a. Eat six small equal meals per day. b. Reduce the carbohydrates in her diet. c. Eat her meals and snacks on a fixed schedule. d. Increase her consumption of protein.

ANS: C Having a fixed meal schedule will provide the woman and the fetus with a steady blood sugar level, provide a good balance with insulin administration, and help prevent complications. Having a fixed meal schedule is more important than the equal division of food intake. Approximately 45% of the food eaten should be in the form of carbohydrates.

A woman's position is an important component of the labor progress. Which guidance is important for the nurse to provide to the laboring client? a. The supine position, which is commonly used in the United States, increases blood flow. b. The laboring client positioned on her hands and knees ("all fours" position) is hard on the woman's back. c. Frequent changes in position help relieve fatigue and increase the comfort of the laboring client. d. In a sitting or squatting position, abdominal muscles of the laboring client will have to work harder.

ANS: C Frequent position changes relieve fatigue, increase comfort, and improve circulation. Blood flow can be compromised in the supine position; any upright position benefits cardiac output. The "all fours" position is used to relieve backache in certain situations. In a sitting or squatting position, the abdominal muscles work in greater harmony with uterine contractions.

Which action is the first priority for the nurse who is assessing the influence of culture on a clients diet? a. Evaluate the clients weight gain during pregnancy. b. Assess the socioeconomic status of the client. c. Discuss the four food groups with the client. d. Identify the food preferences and methods of food preparation common to the clients culture.

ANS: D Understanding the clients food preferences and how she prepares food will assist the nurse in determining whether the clients culture is adversely affecting her nutritional intake. An evaluation of a clients weight gain during pregnancy should be included for all clients, not only for clients from different cultural backgrounds. The socioeconomic status of the client may alter the nutritional intake but not the cultural influence. Teaching the food groups to the client should come after assessing her food preferences.

To assess fundal contraction 6 hours after cesarean delivery, the nurse should a. Palpate forcefully through the abdominal dressing. b. Gently palpate, applying the same technique used for vaginal deliveries. c. Place hands on both sides of the abdomen and press downward. d. Rely on assessment of lochial flow rather than palpating the fundus.

B

To promote bonding and attachment immediately after delivery, the nurse should: a. Allow the mother quiet time with her infant. b. Assist the mother in assuming an en face position with her newborn. c. Teach the mother about the concepts of bonding and attachment. d. Assist the mother in feeding her baby.

B

If the patient's white blood cell (WBC) count is 25,000/mm3 on her second postpartum day, the nurse should a. Tell the physician immediately. b. Have the laboratory draw blood for reanalysis. c. Recognize that this is an acceptable range at this point postpartum. d. Begin antibiotic therapy immediately.

C

On observing a woman on her first postpartum day sitting in bed while her newborn lies awake in the bassinet, the nurse should: a. Realize that this situation is perfectly acceptable. b. Offer to hand the baby to the woman. c. Hand the baby to the woman. d. Explain "taking in" to the woman.

C

The best way for the nurse to promote and support the maternal-infant bonding process is to a. Help the mother identify her positive feelings toward the newborn. b. Encourage the mother to provide all newborn care. c. Assist the family with rooming-in. d. Return the newborn to the nursery during sleep periods.

C

Two days ago, a woman gave birth to a full-term infant. Last night, she awakened several times to urinate and noted that her gown and bedding were wet from profuse diaphoresis. One mechanism for the diaphoresis and diuresis that this woman is experiencing during the early postpartum period is: a. Elevated temperature caused by postpartum infection b. Increased basal metabolic rate after giving birth c. Loss of increased blood volume associated with pregnancy d. Increased venous pressure in the lower extremities

C

A nurse is observing a family. The mother is holding the baby she delivered less than 24 hours ago. Her husband is watching his wife and asking questions about newborn care. The 4-year-old brother is punching his mother on the back. The nurse should: a. Report the incident to the social services department. b. Advise the parents that the toddler needs to be reprimanded. c. Report to oncoming staff that the mother is probably not a good disciplinarian. d. Realize that this is a normal family adjusting to family change.

D

Because a full bladder prevents the uterus from contracting normally, nurses intervene to help the woman empty her bladder spontaneously as soon as possible. If all else fails, the last thing the nurse might try is: a. Pouring water from a squeeze bottle over the woman's perineum b. Providing hot tea c. Asking the physician to prescribe analgesics d. Inserting a sterile catheter

D

If rubella vaccine is indicated for a postpartum patient, instructions to the patient should include: a. Drinking plenty of fluids to prevent fever b. No specific instructions c. Recommending that she stop breastfeeding for 24 hours after injection d. Explaining the risks of becoming pregnant within 1 month after injection

D

A woman weighs herself the day after delivery and is upset because she just lost 14 pounds. The nurse's best response is based on knowledge that A. Prepregnancy weight should be reached at about 6 months B. Weight loss will increase after 48 hours C. Weight loss will occur only with diet and exercise D. Prepregnancy weight should be reached at about 2 weeks

A

During which phase of maternal adjustment will the mother relinquish the baby of her fantasies and accept the real baby? a. Letting go b. Taking hold c. Taking in d. Taking on

A

A woman is 2 days postpartum. Her current lab work shows a WBC level of 17,000/mm3. The nurse's next action should be: A. Nothing—this is in the acceptable range B. To notify the physician C. To ask for a retest D. To force fluids

A

The BMI for a woman who is 51 kg before pregnancy and 1.57 m tall is _________.

ANS: 20.7 A commonly used method of evaluating the appropriateness of weight for height is the BMI, which is calculated by the following formula. BMI = Weight in kg divided by the height in meters squared BMI = 51 kg (1.57 m)2 51 2.47 = 20.69

What is the correct placement of the tocotransducer for effective EFM? a.Over the uterine fundus b.On the fetal scalp c.Inside the uterus d.Over the mother's lower abdomen

ANS: A The tocotransducer monitors uterine activity and should be placed over the fundus, where the most intensive uterine contractions occur. The tocotransducer is for external use.

A woman exhibits symptoms that may lead to a possible diagnosis of polycystic ovary syndrome (PCOS). While completing the initial assessment of the client, which clinical finding would the nurse not anticipate? a.Anorexia b.Hirsutism c.Irregular menses d.Infertility

ANS: A These clients often are obese rather than anorexic with weight loss. Approximately 40% of these women also display glucose intolerance and hyperinsulinemia. Excessive hair growth is often present in women with PCOS. This client is likely to have irregular menses or even amenorrhea. Infertility as a result of decreased levels of follicle-stimulating hormone is common with this syndrome.

The nurse expects which maternal cardiovascular finding during labor? a. Increased cardiac output b. Decreased pulse rate c. Decreased white blood cell (WBC) count d. Decreased blood pressure

ANS: A During each contraction, 400 ml of blood is emptied from the uterus into the maternal vascular system, which increases cardiac output by approximately 10% to 15% during the first stage of labor and by approximately 30% to 50% in the second stage of labor. The heart rate increases slightly during labor. The WBC count can increase during labor. During the first stage of labor, uterine contractions cause systolic readings to increase by approximately 10 mm Hg. During the second stage, contractions may cause systolic pressures to increase by 30 mm Hg and diastolic readings to increase by 25 mm Hg.

Which condition would require prophylaxis to prevent subacute bacterial endocarditis (SBE) both antepartum and intrapartum? a. Valvular heart disease b. Congestive heart disease c. Arrhythmias d. Postmyocardial infarction

ANS: A Prophylaxis for intrapartum endocarditis and pulmonary infection may be provided for women who have mitral valve prolapse. Prophylaxis for intrapartum endocarditis is not indicated for a client with congestive heart disease, underlying arrhythmias, or postmyocardial infarction.

Which assessments are included in the fetal BPP? (Select all that apply.) a. Fetal movement b. Fetal tone c. Fetal heart rate d. AFI e. Placental grade

ANS: A, B, C, D Fetal movement, tone, heart rate, and AFI are all assessed in a BPP. The placental grade is determined by ultrasound and is not included in the criteria of assessment factors for a BPP.

What is the nurse's understanding of the appropriate role of primary and secondary powers? a. Primary powers are responsible for the effacement and dilation of the cervix. b. Effacement is generally well ahead of dilation in women giving birth for the first time; they are closer together in subsequent pregnancies. c. Scarring of the cervix caused by a previous infection or surgery may make the delivery a bit more painful, but it should not slow or inhibit dilation. d. Pushing in the second stage of labor is more effective if the woman can breathe deeply and control some of her involuntary needs to push, as the nurse directs.

ANS: A The primary powers are responsible for dilation and effacement; secondary powers are concerned with expulsion of the fetus. Effacement is generally well ahead of dilation in first-time pregnancies; they are closer together in subsequent pregnancies. Scarring of the cervix may slow dilation. Pushing is more effective and less fatiguing when the woman begins to push only after she has the urge to do so.

Emergency conditions during labor that would require immediate nursing intervention can arise with startling speed. Which situations are examples of such an emergency? (Select all that apply.) a. Nonreassuring or abnormal FHR pattern b. Inadequate uterine relaxation c. Vaginal bleeding d. Prolonged second stage e. Prolapse of the cord

ANS: A, B, C, E A nonreassuring or abnormal FHR pattern, inadequate uterine relaxation, vaginal bleeding, infection, and cord prolapse all constitute an emergency during labor that requires immediate nursing intervention. A prolonged second stage of labor after the upper limits for duration is reached. This is 3 hours for nulliparous women and 2 hours for multiparous women.

Pregnancy is a hypercoagulable state in which women are at a fivefold to sixfold increased risk for thromboembolic disease. The tendency for blood to clot is greater, attributable to an increase in various clotting factors. Which of these come into play during pregnancy? (Select all that apply.) a. Factor VII b. Factor VIII c. Factor IX d. Factor XIII e. Fibrinogen

ANS: A, B, C, E Factors VII, VIII, IX, X, and fibrinogen increase in pregnancy. Factors that inhibit coagulation decrease. Fibrinolytic activity (dissolving of a clot) is depressed during pregnancy and the early postpartum period to protect the women from postpartum hemorrhage.

Which medications are used to manage PPH? (Select all that apply.) a.Oxytocin b.Methergine c.Terbutaline d.Hemabate e.Magnesium sulfate

ANS: A, B, D Oxytocin, Methergine, and Hemabate are medications used to manage PPH. Terbutaline and magnesium sulfate are tocolytic medications that are used to relax the uterus, which would cause or worsen PPH.

Because of its size and rigidity, the fetal head has a major effect on the birth process. Which bones comprise the structure of the fetal skull? (Select all that apply.) a. Parietal b. Temporal c. Fontanel d. Occipital e. Femoral

ANS: A, B, D The fetal skull has two parietal bones, two temporal bones, an occipital bone, and a frontal bone. The fontanels are membrane-filled spaces.

Nurses are in an ideal position to educate clients who experience PMDD. What self-help activities have been documented as helpful in alleviating the symptoms of PMDD? (Select all that apply.) a. Regular exercise b. Improved nutrition c. Daily glass of wine d. Smoking cessation e. Oil of evening primrose

ANS: A, B, D, E Regular exercise, improved nutrition, smoking cessation, and oil of evening primrose are accurate modalities that may provide significant symptom relief in 1 to 2 months. If no improvement is realized after these changes have been made, then the client may need to begin pharmacologic therapy. Women should decrease their alcohol and caffeinated beverage consumption if they suffer from PMDD.

Which statements regarding menstruation (periodic uterine bleeding) are accurate? (Select all that apply.) a. Menstruation occurs every 28 days. b. During menstruation, the entire uterine lining is shed. c. Menstruation begins 7 to 10 days after ovulation. d. Menstruation leads to fertilization. e. Average blood loss during menstruation is 50 ml.

ANS: A, B, E Menstruation is the periodic uterine bleeding that is controlled by a feedback system involving three cycles: the endometrial cycle, the hypothalamic-pituitary cycle, and the ovarian cycle. The average length of a menstrual cycle is 28 days; however, variations are normal. During the endometrial cycle, the functional two thirds of the endometrium is shed. The average blood loss is 50 ml with a normal range of 20 to 80 ml. Menstruation occurs 14 days after ovulation. The lack of fertilization leads to menstruation.

Relating to the fetal circulatory system, which special characteristics allow the fetus to obtain sufficient oxygen from the maternal blood? (Select all that apply.) a. Fetal hemoglobin (Hb) carries 20% to 30% more oxygen than maternal Hb. b. Fetal Hb carries 40% to 50% more oxygen than maternal Hb. c. Hb concentration is 50% higher than that of the mother. d. Fetal heart rate is 110 to 160 beats per minute. e. Fetal heart rate is 160 to 200 beats per minute.

ANS: A, C, D The following three special characteristics enable the fetus to obtain sufficient oxygen from maternal blood: (1) the fetal Hb carries 20% to 30% more oxygen; (2) the concentration is 50% higher than that of the mother; and (3) the fetal heart rate is 110 to 160 beats per minute, a cardiac output that is higher than that of an adult.

Which laboratory marker is indicative of DIC? a.Bleeding time of 10 minutes b.Presence of fibrin split products c.Thrombocytopenia d.Hypofibrinogenemia

ANS: B Degradation of fibrin leads to the accumulation of multiple fibrin clots throughout the body's vasculature. Bleeding time in DIC is normal. Low platelets may occur but are not indicative of DIC because they may be the result from other coagulopathies. Hypofibrinogenemia occurs with DIC.

To prevent gastrointestinal (GI) upset, when should a pregnant client be instructed to take the recommended iron supplements? a. On a full stomach b. At bedtime c. After eating a meal d. With milk

ANS: B Iron supplements taken at bedtime may reduce GI upset and should be taken at bedtime if abdominal discomfort occurs when iron supplements are taken between meals. Iron supplements are best absorbed if they are taken when the stomach is empty. Bran, tea, coffee, milk, and eggs may reduce absorption.

Which statement regarding the probable signs of pregnancy is most accurate? a. Determined by ultrasound b. Observed by the health care provider c. Reported by the client d. Confirmed by diagnostic tests

ANS: B Probable signs are those detected through trained examination. Fetal visualization is a positive sign of pregnancy. Presumptive signs are those reported by the client. The term diagnostic tests is open for interpretation. To actually diagnose pregnancy, one would have to see positive signs of pregnancy.

What is the primary difference between the labor of a nullipara and that of a multipara? a. Amount of cervical dilation b. Total duration of labor c. Level of pain experienced d. Sequence of labor mechanisms

ANS: B In a first-time pregnancy, the descent is usually slow but steady; in subsequent pregnancies, the descent is more rapid, resulting in a shorter duration of labor. Cervical dilation is the same for all labors. The level of pain is individual to the woman, not to the number of labors she has experienced. The sequence of labor mechanisms is the same with all labors.

The client makes an appointment for preconception counseling. The woman has a known heart condition and is unsure if she should become pregnant. Which is the only cardiac condition that would cause concern? a. Marfan syndrome b. Eisenmenger syndrome c. Heart transplant d. Ventricular septal defect (VSD)

ANS: B Pregnancy is contraindicated in clients with Eisenmenger syndrome. Women who have had heart transplants are successfully having babies. However, conception should be postponed for at least 1 year after transplantation. Management of the client with Marfan syndrome during pregnancy includes bed rest, beta-blockers, and surgery before conception. VSD is usually corrected early in life and is therefore not a contraindication to pregnancy.

Bell palsy is an acute idiopathic facial paralysis, the cause for which remains unknown. Which statement regarding this condition is correct? a. Bell palsy is the sudden development of bilateral facial weakness. b. Women with Bell palsy have an increased risk for hypertension. c. Pregnant women are affected twice as often as nonpregnant women. d. Bell palsy occurs most frequently in the first trimester.

ANS: B The clinical manifestations of Bell palsy include the development of unilateral facial weakness, pain surrounding the ears, difficulty closing the eye, and hyperacusis. The cause is unknown; however, Bell palsy may be related to a viral infection. Pregnant women are affected at a rate of three to five times that of nonpregnant women. The incidence rate peaks during the third trimester and puerperium. Women who develop Bell palsy in pregnancy have an increased risk for hypertension.

Maternal hypotension is a potential side effect of regional anesthesia and analgesia. What nursing interventions could the nurse use to increase the client's blood pressure? (Select all that apply.) a.Place the woman in a supine position. b.Place the woman in a lateral position. c.Increase IV fluids. d.Administer oxygen. e.Perform a vaginal examination.

ANS: B, C, D Nursing interventions for maternal hypotension arising from analgesia or anesthesia include turning the woman to a lateral position, increasing IV fluids, administering oxygen via face mask, elevating the woman's legs, notifying the physician, administering an IV vasopressor, and monitoring the maternal and fetal status at least every 5 minutes until the woman is stable. Placing the client in a supine position causes venous compression, thereby limiting blood flow to and oxygenation of the placenta and fetus. A sterile vaginal examination has no bearing on maternal blood pressure.

2. Foodborne illnesses can cause adverse effects for both mother and fetus. The nurse is in an ideal position to evaluate the clients knowledge regarding steps to prevent a foodborne illness. The nurse asks the client to teach back the fours simple steps of food preparation. What are they? (Select all that apply.) a. Purchase b. Clean c. Separate d. Cook e. Chill

ANS: B, C, D, E According to the U.S. Food and Drug Administration (2013), the four simple steps are: Clean: Frequently cleanse hands, food preparation surfaces, and utensils. Separate: Avoid contact among raw meat, fish, or poultry and other foods that will not be cooked before consumption. Cook: Cook foods to the proper temperature. Chill: Properly store foods, and promptly refrigerate.

The microscopic examination of scrapings from the cervix, endocervix, or other mucous membranes to detect premalignant or malignant cells is called what? a. Bimanual palpation b. Rectovaginal palpation c. Papanicolaou (Pap) test d. Four As procedure

ANS: C The Pap test is a microscopic examination for cancer that should be regularly performed, depending on the clients age. Bimanual palpation is a physical examination of the vagina. Rectovaginal palpation is a physical examination performed through the rectum. The four As procedure is an intervention to help a client stop smoking.

The measurement of lecithin in relation to sphingomyelin (lecithin/sphingomyelin [L/S] ratio) is used to determine fetal lung maturity. Which ratio reflects fetal maturity of the lungs? a. 1.4:1 b. 1.8:1 c. 2:1 d. 1:1

ANS: C The L/S ratio indicates a 2:1 ratio of lecithin to sphingomyelin, which is an indicator of fetal lung maturity and occurs at approximately the middle of the third trimester. L/S ratios of 1.4:1, 1.8:1, and 1:1 each indicate immaturity of the fetal lungs.

The nurse is performing an initial assessment of a client in labor. What is the appropriate terminology for the relationship of the fetal body parts to one another? a. Lie b. Presentation c. Attitude d. Position

ANS: C Attitude is the relationship of the fetal body parts to one another. Lie is the relationship of the long axis (spine) of the fetus to the long axis (spine) of the mother. Presentation refers to the part of the fetus that enters the pelvic inlet first and leads through the birth canal during labor at term. Position is the relationship of the presenting part of the fetus to the four quadrants of the mother's pelvis.

A woman with asthma is experiencing a postpartum hemorrhage. Which drug should be avoided when treating postpartum bleeding to avoid exacerbating asthma? a. Oxytocin (Pitocin) b. Nonsteroidal antiinflammatory drugs (NSAIDs) c. Hemabate d. Fentanyl

ANS: C Prostaglandin derivatives should not be used to treat women with asthma, because they may exacerbate symptoms. Oxytocin is the drug of choice to treat this woman's bleeding; it will not exacerbate her asthma. NSAIDs are not used to treat bleeding. Fentanyl is used to treat pain, not bleeding.

A pregnant womans diet may not meet her increased need for folates. Which food is a rich source of this nutrient? a. Chicken b. Cheese c. Potatoes d. Green leafy vegetables

ANS: D Sources of folates include green leafy vegetables, whole grains, fruits, liver, dried peas, and beans. Chicken and cheese are excellent sources of protein but are poor sources for folates. Potatoes contain carbohydrates and vitamins and minerals but are poor sources for folates.

Which statement(s) might the nurse appropriately include when teaching a client about calcium intake for osteoporosis? (Select all that apply.) a. You should try to increase your protein intake when you are taking calcium. b. It is best to take calcium in one large dose. c. Tums are the most soluble form of calcium. d. You should take calcium with vitamin D because the vitamin D helps your body better absorb calcium. e. Its okay to take calcium if you have had a history of kidney stones.

ANS: C, D Teaching the client to take calcium with vitamin D is accurate. Excessive protein should be avoided. Calcium is best taken in divided doses to increase absorption. Calcium should be taken with vitamin D to increase absorption. Calcium is contraindicated in women with a history of kidney stones.

A client states that she does not drink milk. Which foods should the nurse encourage this woman to consume in greater amounts to increase her calcium intake? a. Fresh apricots b. Canned clams c. Spaghetti with meat sauce d. Canned sardines

ANS: D Sardines are rich in calcium. Fresh apricots, canned clams, and spaghetti with meat sauce are not high in calcium.

Postoperative care of the pregnant woman who requires abdominal surgery for appendicitis includes which additional assessment? a. Intake and output (I&O) and intravenous (IV) site b. Signs and symptoms of infection c. Vital signs and incision d. Fetal heart rate (FHR) and uterine activity

ANS: D Care of a pregnant woman undergoing surgery for appendicitis differs from that for a nonpregnant woman in one significant aspect: the presence of the fetus. Continuous fetal and uterine monitoring should take place. An assessment of I&O levels, along with an assessment of the IV site, are normal postoperative care procedures. Evaluating the client for signs and symptoms of infection is also part of routine postoperative care. Routine vital signs and evaluation of the incision site are expected components of postoperative care.

When caring for a pregnant woman with cardiac problems, the nurse must be alert for the signs and symptoms of cardiac decompensation. Which critical findings would the nurse find on assessment of the client experiencing this condition? a. Regular heart rate and hypertension b. Increased urinary output, tachycardia, and dry cough c. Shortness of breath, bradycardia, and hypertension d. Dyspnea, crackles, and an irregular, weak pulse

ANS: D Signs of cardiac decompensation include dyspnea; crackles; an irregular, weak, and rapid pulse; rapid respirations; a moist and frequent cough; generalized edema; increasing fatigue; and cyanosis of the lips and nailbeds. A regular heart rate and hypertension are not generally associated with cardiac decompensation. Of the symptoms of increased urinary output, tachycardia, and dry cough, only tachycardia is indicative of cardiac decompensation. Of the symptoms of shortness of breath, bradycardia, and hypertension, only dyspnea is indicative of cardiac decompensation.

A new mother asks the nurse when the "soft spot" on her son's head will go away. What is the nurse's best response, based upon her understanding of when the anterior frontal closes? a. 2 months b. 8 months c. 12 months d. 18 months

ANS: D The larger of the two fontanels, the anterior fontanel, closes by 18 months after birth. The posterior fontanel closes at 6 to 8 weeks. The remaining three options are too early for the anterior fontanel to close.

A 25-year-old gravida 1 para 1 who had an emergency cesarean birth 3 days ago is scheduled for discharge. As you prepare her for discharge, she begins to cry. Your initial action should be to: a. Assess her for pain. b. Point out how lucky she is to have a healthy baby. c. Explain that she is experiencing postpartum blues. d. Allow her time to express her feelings.

D

A postpartum patient asks, "Will these stretch marks go away?" The nurse's best response is a. "They will continue to fade and should be gone by your 6-week checkup." b. "No, never." c. "Yes, eventually." d. "They will fade to silvery lines but won't disappear completely."

D

A woman gave birth vaginally to a 9-pound, 12-ounce girl yesterday. Her primary health care provider has written orders for perineal ice packs, use of a sitz bath tid, and a stool softener. What information is most closely correlated with these orders? a. The woman is a gravida 2, para 2. b. The woman had a vacuum-assisted birth. c. The woman received epidural anesthesia. d. The woman has an episiotomy.

D

A 25-year-old multiparous woman gave birth to an infant boy 1 day ago. Today her husband brings a large container of brown seaweed soup to the hospital. When the nurse enters the room, the husband asks for help with warming the soup so that his wife can eat it. The nurse's most appropriate response is to ask the woman: a. "Didn't you like your lunch?" b. "Does your doctor know that you are planning to eat that?" c. "What is that anyway?" d. "I'll warm the soup in the microwave for you."

D

A nurse is discussing the storage of breast milk with a mother whose infant is preterm and in the special care nursery. Which statement indicates that the mother requires additional teaching? a. "I can store my breast milk in the refrigerator for 3 months." b. "I can store my breast milk in the freezer for 3 months." c. "I can store my breast milk at room temperature for 4 hours." d. "I can store my breast milk in the refrigerator for 3 to 5 days."

a. "I can store my breast milk in the refrigerator for 3 months."

The maternity nurse must be cognizant that cultural practices have significant influence on infant feeding methods. Many regional and ethnic cultures can be found within the United States. One cannot assume that generalized observations about any cultural group will hold for all members of the group. Which statement related to cultural practices influencing infant feeding practice is correct? a. A common practice among Mexican women is known as las dos cosas. b. Muslim cultures do not encourage breastfeeding because of modesty concerns. c. Latino women born in the United States are more likely to breastfeed. d. East Indian and Arab women believe that cold foods are best for a new mother.

a. A common practice among Mexican women is known as las dos cosas.

A pregnant woman wants to breastfeed her infant; however, her husband is not convinced that there are any scientific reasons to do so. The nurse can give the couple printed information comparing breastfeeding and bottle feeding. Which statement regarding bottle feeding using commercially prepared infant formulas might influence their choice? a. Bottle feeding using a commercially prepared formula increases the risk that the infant will develop allergies. b. Bottle feeding helps the infant sleep through the night. c. Commercially prepared formula ensures that the infant is getting iron in a form that is easily absorbed. d. Bottle feeding requires that multivitamin supplements be given to the infant.

a. Bottle feeding using a commercially prepared formula increases the risk that the infant will develop allergies.

A new mother asks whether she should feed her newborn colostrum, because it is not "real milk." What is the nurse's most appropriate answer? a. Colostrum is high in antibodies, protein, vitamins, and minerals. b. Colostrum is lower in calories than milk and should be supplemented by formula. c. Giving colostrum is important in helping the mother learn how to breastfeed before she goes home. d. Colostrum is unnecessary for newborns.

a. Colostrum is high in antibodies, protein, vitamins, and minerals.

The birth weight of a breastfed newborn was 8 lb, 4 oz. On the third day the newborn's weight is 7 lb, 12 oz. On the basis of this finding, the nurse should: a. Encourage the mother to continue breastfeeding because it is effective in meeting the newborn's nutrient and fluid needs. b. Suggest that the mother switch to bottle feeding because breastfeeding is ineffective in meeting newborn needs for fluid and nutrients. c. Notify the physician because the newborn is being poorly nourished. d. Refer the mother to a lactation consultant to improve her breastfeeding technique.

a. Encourage the mother to continue breastfeeding because it is effective in meeting the newborn's nutrient and fluid needs.

A new mother asks the nurse what the "experts say" about the best way to feed her infant. Which recommendation of the American Academy of Pediatrics (AAP) regarding infant nutrition should be shared with this client? a. Infants should be given only human milk for the first 6 months of life. b. Infants fed on formula should be started on solid food sooner than breastfed infants. c. If infants are weaned from breast milk before 12 months, then they should receive cow's milk, not formula. d. After 6 months, mothers should shift from breast milk to cow's milk.

a. Infants should be given only human milk for the first 6 months of life.

Which statement is the best rationale for recommending formula over breastfeeding? a. Mother has a medical condition or is taking drugs that could be passed along to the infant via breast milk. b. Mother lacks confidence in her ability to breastfeed. c. Other family members or care providers also need to feed the baby. d. Mother sees bottle feeding as more convenient.

a. Mother has a medical condition or is taking drugs that could be passed along to the infant via breast milk.

Parents have been asked by the neonatologist to provide breast milk for their newborn son, who was born prematurely at 32 weeks of gestation. The nurse who instructs them regarding pumping, storing, and transporting the milk needs to assess their knowledge of lactation. Which statement is valid? a. Premature infants more easily digest breast milk than formula. b. A glass of wine just before pumping will help reduce stress and anxiety. c. The mother should only pump as much milk as the infant can drink. d. The mother should pump every 2 to 3 hours, including during the night.

a. Premature infants more easily digest breast milk than formula.

The breastfeeding mother should be taught a safe method to remove the breast from the baby's mouth. Which suggestion by the nurse is most appropriate? a. Slowly remove the breast from the baby's mouth when the infant has fallen asleep and the jaws are relaxed. b. Break the suction by inserting your finger into the corner of the infant's mouth. c. A popping sound occurs when the breast is correctly removed from the infant's mouth. d. Elicit the Moro reflex to wake the baby and remove the breast when the baby cries.

b. Break the suction by inserting your finger into the corner of the infant's mouth.

Which action of a breastfeeding mother indicates the need for further instruction? a. Holds breast with four fingers along bottom and thumb at top. b. Leans forward to bring breast toward the baby. c. Stimulates the rooting reflex and then inserts nipple and areola into newborn's open mouth. d. Puts her finger into newborn's mouth before removing breast.

b. Leans forward to bring breast toward the baby.

A new mother recalls from prenatal class that she should try to feed her newborn daughter when she exhibits feeding readiness cues rather than waiting until the baby is frantically crying. Which feeding cue would indicate that the baby is ready to eat? a. Waves her arms in the air b. Makes sucking motions c. Has the hiccups d. Stretches out her legs straight

b. Makes sucking motions

Which instruction should the nurse provide to reduce the risk of nipple trauma? a. Limit the feeding time to less than 5 minutes. b. Position the infant so the nipple is far back in the mouth. c. Assess the nipples before each feeding. d. Wash the nipples daily with mild soap and water.

b. Position the infant so the nipple is far back in the mouth.

A new mother wants to be sure that she is meeting her daughter's needs while feeding the baby commercially prepared infant formula. The nurse should evaluate the mother's knowledge about appropriate infant feeding techniques. Which statement by the client reassures the nurse that correct learning has taken place? a. "Since reaching 2 weeks of age, I add rice cereal to my daughter's formula to ensure adequate nutrition." b. "I warm the bottle in my microwave oven." c. "I burp my daughter during and after the feeding as needed." d. "I refrigerate any leftover formula for the next feeding."

c. "I burp my daughter during and after the feeding as needed."

How many kilocalories per kilogram (kcal/kg) of body weight does a breastfed term infant require each day? a. 50 to 65 b. 75 to 90 c. 95 to 110 d. 150 to 200

c. 95 to 110

A breastfeeding woman develops engorged breasts at 3 days postpartum. What action will help this client achieve her goal of reducing the engorgement? a. Skip feedings to enable her sore breasts to rest. b. Avoid using a breast pump. c. Breastfeed her infant every 2 hours. d. Reduce her fluid intake for 24 hours.

c. Breastfeed her infant every 2 hours.

The concept of tandem feeding is based on: a. Adequate nutritional stores for the mother and infant. b. Using both breasts to nurse the baby. c. Breastfeeding an infant and an older sibling during the same period. d. Supplementing breastfeeding with bottle feeding to maintain adequate weight gain.

c. Breastfeeding an infant and an older sibling during the same period.

While discussing the societal impacts of breastfeeding, the nurse should be cognizant of the benefits and educate the client accordingly. Which statement as part of this discussion would be incorrect? a. Breastfeeding requires fewer supplies and less cumbersome equipment. b. Breastfeeding saves families money. c. Breastfeeding costs employers in terms of time lost from work. d. Breastfeeding benefits the environment.

c. Breastfeeding costs employers in terms of time lost from work.

At a 2-month well-baby examination, it was discovered that an exclusively breastfed infant had only gained 10 ounces in the past 4 weeks. The mother and the nurse develop a feeding plan for the infant to increase his weight gain. Which change in dietary management will assist the client in meeting this goal? a. Begin solid foods. b. Have a bottle of formula after every feeding. c. Have one extra breastfeeding session every 24 hours. d. Start iron supplements.

c. Have one extra breastfeeding session every 24 hours.

Which action by the mother will initiate the milk ejection reflex (MER)? a. Wearing a firm-fitting bra b. Drinking plenty of fluids c. Placing the infant to the breast d. Applying cool packs to her breast

c. Placing the infant to the breast

Which type of formula is not diluted with water, before being administered to an infant? a. Powdered b. Concentrated c. Ready-to-use d. Modified cow's milk

c. Ready-to-use

According to demographic research, which woman is least likely to breastfeed and therefore most likely to need education regarding the benefits and proper techniques of breastfeeding? a. Between 30 and 35 years of age, Caucasian, and employed part time outside the home b. Younger than 25 years of age, Hispanic, and unemployed c. Younger than 25 years of age, African-American, and employed full time outside the home d. 35 years of age or older, Caucasian, and employed full time at home

c. Younger than 25 years of age, African-American, and employed full time outside the home

Nurses should be able to teach breastfeeding mothers the signs that the infant has correctly latched on. Which client statement indicates a poor latch? a. "I feel a firm tugging sensation on my nipples but not pinching or pain." b. "My baby sucks with cheeks rounded, not dimpled." c. "My baby's jaw glides smoothly with sucking." d. "I hear a clicking or smacking sound."

d. "I hear a clicking or smacking sound."

A primiparous woman is delighted with her newborn son and wants to begin breastfeeding as soon as possible. How should the client be instructed to position the infant to facilitate correct latch-on? a. The infant should be positioned with his or her arms folded together over the chest. b. The infant should be curled up in a fetal position. c. The woman should cup the infant's head in her hand. d. The infant's head and body should be in alignment with the mother.

d. The infant's head and body should be in alignment with the mother.

Which statement regarding infant weaning is correct? a. Weaning should proceed from breast to bottle to cup. b. The feeding of most interest should be eliminated first. c. Abrupt weaning is easier than gradual weaning. d. Weaning can be mother or infant initiated.

d. Weaning can be mother or infant initiated.

Which analysis of maternal serum may predict chromosomal abnormalities in the fetus? a. Multiple-marker screening b. L/S ratio c. BPP d. Blood type and crossmatch of maternal and fetal serum

ANS: A Maternal serum can be analyzed for abnormal levels of alpha-fetoprotein, human chorionic gonadotropin, and estriol. The multiple-marker screening may predict chromosomal defects in the fetus. The L/S ratio is used to determine fetal lung maturity. A BPP is used for evaluating fetal status during the antepartum period. Five variables are used, but none is concerned with chromosomal problems. The blood type and crossmatch would not predict chromosomal defects in the fetus.

When assessing a woman in the first stage of labor, which clinical finding will alert the nurse that uterine contractions are effective? a. Dilation of the cervix b. Descent of the fetus to -2 station c. Rupture of the amniotic membranes d. Increase in bloody show

ANS: A The vaginal examination reveals whether the woman is in true labor. Cervical change, especially dilation, in the presence of adequate labor, indicates that the woman is in true labor. Engagement and descent of the fetus are not synonymous and may occur before labor. ROM may occur with or without the presence of labor. Bloody show may indicate a slow, progressive cervical change (e.g., effacement) in both true and false labor.

A serious but uncommon complication of undiagnosed or partially treated hyperthyroidism is a thyroid storm, which may occur in response to stress such as infection, birth, or surgery. What are the signs and symptoms of this emergency disorder? (Select all that apply.) a. Fever b. Hypothermia c. Restlessness d. Bradycardia e. Hypertension

ANS: A, C Fever, restlessness, tachycardia, vomiting, hypotension, and stupor are symptoms of a thyroid storm. Fever, not hypothermia; tachycardia, not bradycardia; and hypotension, not hypertension, are symptoms of thyroid storm.

Which statement regarding the structure and function of the placenta is correct? a. Produces nutrients for fetal nutrition b. Secretes both estrogen and progesterone c. Forms a protective, impenetrable barrier to microorganisms such as bacteria and viruses d. Excretes prolactin and insulin

ANS: B As one of its early functions, the placenta acts as an endocrine gland, producing four hormones necessary to maintain the pregnancy and to support the embryo or fetus: human chorionic gonadotropin (hCG), human placental lactogen (hPL), estrogen, and progesterone. The placenta does not produce nutrients. It functions as a means of metabolic exchange between the maternal and fetal blood supplies. Many bacteria and viruses can cross the placental membrane.

An unmarried young woman describes her sex life as active and involving many partners. She wants a contraceptive method that is reliable and does not interfere with sex. She requests an intrauterine device (IUD). Which information is most important for the nurse to share? a. The IUD does not interfere with sex. b. The risk of pelvic inflammatory disease will be higher with the IUD. c. The IUD will protect you from sexually transmitted infections. d. Pregnancy rates are high with the IUD.

ANS: B Disadvantages of IUDs include an increased risk of pelvic inflammatory disease (PID) in the first 20 days after insertion, as well as the risks of bacterial vaginosis and uterine perforation. The IUD offers no protection against sexually transmitted infections (STIs) or the human immunodeficiency virus (HIV), as does a barrier method. Because this woman has multiple sex partners, she is at higher risk of developing an STI. Stating that an IUD does not interfere with sex may be correct; however, it is not the most appropriate response. The typical failure rate of the IUD is approximately 1%

Which characteristic correctly matches the type of deceleration with its likely cause? a.Early deceleration—umbilical cord compression b.Late deceleration—uteroplacental insufficiency c.Variable deceleration—head compression d.Prolonged deceleration—unknown cause

ANS: B Late deceleration is caused by uteroplacental insufficiency. Early deceleration is caused by head compression. Variable deceleration is caused by umbilical cord compression. Prolonged deceleration has a variety of either benign or critical causes.

After an emergency birth, the nurse encourages the woman to breastfeed her newborn. What is the primary purpose of this activity? a. To facilitate maternal-newborn interaction b. To stimulate the uterus to contract c. To prevent neonatal hypoglycemia d. To initiate the lactation cycle

ANS: B Stimulation of the nipples through breastfeeding or manual stimulation causes the release of oxytocin and prevents maternal hemorrhage. Breastfeeding facilitates maternal-newborn interaction, but it is not the primary reason a woman is encouraged to breastfeed after an emergency birth. The primary intervention for preventing neonatal hypoglycemia is thermoregulation. Cold stress can result in hypoglycemia. The woman is encouraged to breastfeed after an emergency birth to stimulate the release of oxytocin, which prevents hemorrhaging. Breastfeeding is encouraged to initiate the lactation cycle, but it is not the primary reason for this activity after an emergency birth

Which nursing intervention is paramount when providing care to a client with preterm labor who has received terbutaline? a.Assess deep tendon reflexes (DTRs). b.Assess for dyspnea and crackles. c.Assess for bradycardia. d.Assess for hypoglycemia.

ANS: B Terbutaline is a beta2-adrenergic agonist that affects the mother's cardiopulmonary and metabolic systems. Signs of cardiopulmonary decompensation include adventitious breath sounds and dyspnea. An assessment for dyspnea and crackles is important for the nurse to perform if the woman is taking magnesium sulfate. Assessing DTRs does not address the possible respiratory side effects of using terbutaline. Since terbutaline is a beta2-adrenergic agonist, it can lead to hyperglycemia, not hypoglycemia. Beta2-adrenergic agonist drugs cause tachycardia, not bradycardia.

Which statement regarding the laboratory test for glycosylated hemoglobin Alc is correct? a. The laboratory test for glycosylated hemoglobin Alc is performed for all pregnant women, not only those with or likely to have diabetes. b. This laboratory test is a snapshot of glucose control at the moment. c. This laboratory test measures the levels of hemoglobin Alc, which should remain at less than 7%. d. This laboratory test is performed on the womans urine, not her blood.

ANS: C Hemoglobin Alc levels greater than 7% indicate an elevated glucose level during the previous 4 to 6 weeks. This extra laboratory test is for diabetic women and defines glycemic control over the previous 4 to 6 weeks. Glycosylated hemoglobin level tests are performed on the blood.

A primigravida is being monitored at the prenatal clinic for preeclampsia. Which finding is of greatest concern to the nurse? A. Blood pressure (BP) increase to 138/86 mm Hg b. Weight gain of 0.5 kg during the past 2 weeks c. Dipstick value of 3+ for protein in her urine d. Pitting pedal edema at the end of the day

ANS: C Proteinuria is defined as a concentration of 1+ or greater via dipstick measurement. A dipstick value of 3+ alerts the nurse that additional testing or assessment should be performed. A 24-hour urine collection is preferred over dipstick testing attributable to accuracy. Generally, hypertension is defined as a BP of 140/90 mm Hg or an increase in systolic pressure of 30 mm Hg or diastolic pressure of 15 mm Hg. Preeclampsia may be demonstrated as a rapid weight gain of more than 2 kg in 1 week. Edema occurs in many normal pregnancies, as well as in women with preeclampsia. Therefore, the presence of edema is no longer considered diagnostic of preeclampsia.

The nurse is teaching a client with preterm premature rupture of membranes (PPROM) regarding self-care activities. Which activities should the nurse include in her teaching? a.Report a temperature higher than 40° C. b.Tampons are safe to use to absorb the leaking amniotic fluid. c.Do not engage in sexual activity. d.Taking frequent tub baths is safe.

ANS: C Sexual activity should be avoided because it may induce preterm labor. A temperature higher than 38° C should be reported. To prevent the risk of infection, tub baths should be avoided and nothing should be inserted into the vagina. Further, foul-smelling vaginal fluid, which may be a sign of infection, should be reported.

Which statement regarding the term contraceptive failure rate is the most accurate? a. The contraceptive failure rate refers to the percentage of users expected to have an accidental pregnancy over a 5-year span. b. It refers to the minimum rate that must be achieved to receive a government license. c. The contraceptive failure rate increases over time as couples become more careless. d. It varies from couple to couple, depending on the method and the users.

ANS: D Contraceptive effectiveness varies from couple to couple, depending on how well a contraceptive method is used and how well it suits the couple. The contraceptive failure rate measures the likelihood of accidental pregnancy in the first year only. Failure rates decline over time because users gain experience.

Many pregnant women have questions regarding work and travel during pregnancy. Which education is a priority for the nurse to provide? a. Women should sit for as long as possible and cross their legs at the knees from time to time for exercise. b. Women should avoid seat belts and shoulder restraints in the car because they press on the fetus. c. Metal detectors at airport security checkpoints can harm the fetus if the woman passes through them a number of times. d. While working or traveling in a car or on an airplane, women should arrange to walk around at least every hour or so.

ANS: D Periodic walking helps prevent thrombophlebitis. Pregnant women should avoid sitting or standing for long periods and crossing the legs at the knees. Pregnant women must wear lap belts and shoulder restraints. The most common injury to the fetus comes from injury to the mother. Metal detectors at airport security checkpoints do not harm fetuses.

What is one of the initial signs and symptoms of puerperal infection in the postpartum client? a.Fatigue continuing for longer than 1 week b.Pain with voiding c.Profuse vaginal lochia with ambulation d.Temperature of 38° C (100.4° F) or higher on 2 successive days

ANS: D Postpartum or puerperal infection is any clinical infection of the genital canal that occurs within 28 days after miscarriage, induced abortion, or childbirth. The definition used in the United States continues to be the presence of a fever of 38° C (100.4° F) or higher on 2 successive days of the first 10 postpartum days, starting 24 hours after birth. Fatigue is a late finding associated with infection. Pain with voiding may indicate a urinary tract infection (UTI), but it is not typically one of the earlier symptoms of infection. Profuse lochia may be associated with endometritis, but it is not the first symptom associated with infection.

. A primigravida at 39 weeks of gestation is observed for 2 hours in the intrapartum unit. The FHR has been normal. Contractions are 5 to 9 minutes apart, 20 to 30 seconds in duration, and of mild intensity. Cervical dilation is 1 to 2 cm and uneffaced (unchanged from admission). Membranes are intact. What disposition would the nurse anticipate? a. Admitted and prepared for a cesarean birth b. Admitted for extended observation c. Discharged home with a sedative d. Discharged home to await the onset of true labor

ANS: D This situation describes a woman with normal assessments who is probably in false labor and will likely not deliver rapidly once true labor begins. No further assessments or observations are indicated; therefore, the client will be discharged along with instructions to return when contractions increase in intensity and frequency. Neither a cesarean birth nor a sedative is required at this time.

A woman at 39 weeks of gestation with a history of preeclampsia is admitted to the labor and birth unit. She suddenly experiences increased contraction frequency of every 1 to 2 minutes, dark red vaginal bleeding, and a tense, painful abdomen. Which clinical change does the nurse anticipate? a. Eclamptic seizure b. Rupture of the uterus c. Placenta previa d. Abruptio placentae

ANS: D Uterine tenderness in the presence of increasing tone may be the earliest sign of abruptio placentae. Women with preeclampsia are at increased risk for an abruption attributable to decreased placental perfusion. Eclamptic seizures are evidenced by the presence of generalized tonic-clonic convulsions. Uterine rupture exhibits hypotonic uterine activity, signs of hypovolemia, and, in many cases, the absence of pain. Placenta previa exhibits bright red, painless vaginal bleeding.

As the nurse assists a new mother with breastfeeding, the client asks, "If formula is prepared to meet the nutritional needs of the newborn, what is in breast milk that makes it better?" What is the nurse's best response? a. More calories b. Essential amino acids c. Important immunoglobulins d. More calcium

c. Important immunoglobulins

The nurse working with pregnant clients must seek to gain understanding of the process whereby women accept their pregnancy. Which statement regarding this process is most accurate? a. Nonacceptance of the pregnancy very often equates to a rejection of the child. b. Mood swings are most likely the result of worries about finances and a changed lifestyle, as well as profound hormonal changes. c. Ambivalent feelings during pregnancy are usually only expressed in emotionally immature or very young mothers. d. Conflicts such as not wanting to be pregnant or childrearing and career-related decisions need not be addressed during pregnancy because they will naturally resolve themselves after birth.

ANS: B Mood swings are natural and are likely to affect every woman to some degree. A woman may dislike being pregnant, refuse to accept it, and still love and accept the child. Ambivalent feelings about pregnancy are normal for the mature or immature woman and for the younger or older woman. Conflicts such as not wanting to be pregnant or childrearing and career-related decisions need to be resolved. The baby ends the pregnancy but not all the issues.

The nurse is planning the care for a laboring client with diabetes mellitus. This client is at greater risk for which clinical finding? a. Oligohydramnios b. Polyhydramnios c. Postterm pregnancy d. Chromosomal abnormalities

ANS: B Polyhydramnios or amniotic fluid in excess of 2000 ml is 10 times more likely to occur in the client with diabetes mellitus rather than in nondiabetic pregnancies. This complication places the mother at risk for premature rupture of membranes, premature labor, and postpartum hemorrhage. Prolonged rupture of membranes, IUGR, intrauterine fetal death, and renal agenesis (Potter syndrome) place the client at risk for developing oligohydramnios. Anencephaly, placental insufficiency, and perinatal hypoxia contribute to the risk for postterm pregnancy. Maternal age older than 35 years and balanced translocation (maternal and paternal) are risk factors for chromosomal abnormalities.

Which woman has the highest risk for endometrial cancer? a.Postmenopausal woman with hypertension b.Woman who has an intrauterine device (IUD) c.Client who has been on birth control for 15 years d.Perimenopausal woman who has a cystocele

ANS: A Endometrial cancer is most often seen in postmenopausal women between the ages of 50 and 65 years. Hypertension is a risk factor associated with the development of this malignancy. The use of an IUD does not increase a woman's risk for endometrial cancer. A client who has been on birth control for 15 years is not at increased risk for endometrial cancer; the birth control contraceptives might actually offer some protection. The development of a cystocele will not increase a woman's risk for endometrial cancer.

A pregnant client tells her nurse that she is worried about the blotchy, brownish coloring over her cheeks, nose, and forehead. The nurse can reassure her that this is a normal condition related to hormonal changes. What is the correct term for this integumentary finding? a. Melasma b. Linea nigra c. Striae gravidarum d. Palmar erythema

ANS: A Melasma, (also called chloasma, the mask of pregnancy), usually fades after birth. This hyperpigmentation of the skin is more common in women with a dark complexion. Melasma appears in 50% to 70% of pregnant women. Linea nigra is a pigmented line that runs vertically up the abdomen. Striae gravidarum are also known as stretch marks. Palmar erythema is signified by pinkish red blotches on the hands.

The nurse caring for a pregnant client is evaluating his or her health teaching regarding fetal circulation. Which statement from the client reassures the nurse that his or her teaching has been effective? a. Optimal fetal circulation is achieved when I am in the side-lying position. b. Optimal fetal circulation is achieved when I am on my back with a pillow under my knees. c. Optimal fetal circulation is achieved when the head of the bed is elevated. d. Optimal fetal circulation is achieved when I am on my abdomen.

ANS: A Optimal circulation is achieved when the woman is lying at rest on her side. Decreased uterine circulation may lead to intrauterine growth restriction. Previously, it was believed that the left lateral position promoted maternal cardiac output, enhancing blood flow to the fetus. However, it is now known that the side-lying position enhances uteroplacental blood flow. If a woman lies on her back with the pressure of the uterus compressing the vena cava, then blood return to the right atrium is diminished. Although having the head of the bed elevated is recommended and ideal for later in pregnancy, the woman still must maintain a lateral tilt to the pelvis to avoid compressing the vena cava. Many women find lying on their abdomen uncomfortable as pregnancy advances. Side-lying is the ideal position to promote blood flow to the fetus.

Cell-free deoxyribonucleic acid (DNA) screening is a new method of noninvasive prenatal testing (NIPT) that has recently become available in the clinical setting. This technology can provide a definitive diagnosis of which findings? (Select all that apply.) a. Fetal Rh status b. Fetal gender c. Maternally transmitted gene disorder d. Paternally transmitted gene disorder e. Trisomy 21

ANS: A, B, D, E The NIPT cannot actually distinguish fetal from maternal DNA. It can determine fetal Rh status, gender, trisomies 13, 18, and 21, as well as paternally transmitted gene disorders. The test can be performed any time after 10 weeks of gestation and is recommended for women who have previously given birth to a child with chromosomal abnormalities.

Which contraceptive method best protects against STIs and the HIV? a. Periodic abstinence b. Barrier methods c. Hormonal methods d. Same protection with all methods

ANS: B Barrier methods, such as condoms, protect against STIs and the HIV the best of all contraceptive methods. Periodic abstinence and hormonal methods, such as birth control pills, offer no protection against STIs or the HIV.

The number of routine laboratory tests during follow-up visits is limited; however, those that are performed are essential. Which statements regarding group B Streptococcus (GBS) testing are correct? (Select all that apply.) a. Performed between 32 and 34 weeks of gestation. b. Performed between 35 and 37 weeks of gestation. c. All women should be tested. d. Only women planning a vaginal birth should be tested. e. Women with a history of GBS should be retested.

ANS: B, D, E GBS testing is recommended between 35 and 37 weeks of gestation; cultures collected earlier will not accurately predict the presence of GBS at birth. All women should be tested, even those planning an elective cesarean birth. Membranes may rupture early, requiring prophylactic antibiotics. Clients with a history of GBS should be retested.

What is the most dangerous effect on the fetus of a mother who smokes cigarettes while pregnant? a. Genetic changes and anomalies b. Extensive central nervous system damage c. Fetal addiction to the substance inhaled d. Intrauterine growth restriction

ANS: D The major consequences of smoking tobacco during pregnancy are low-birth-weight infants, prematurity, and increased perinatal loss. Cigarettes will not normally cause genetic changes or extensive central nervous system damage. Addiction to tobacco is not usually a concern related to the neonate.

Which statements concerning the benefits or limitations of breastfeeding are accurate? (Select all that apply.) a. Breast milk changes over time to meet the changing needs as infants grow. b. Breastfeeding increases the risk of childhood obesity. c. Breast milk and breastfeeding may enhance cognitive development. d. Long-term studies have shown that the benefits of breast milk continue after the infant is weaned. e. Benefits to the infant include a reduced incidence of SIDS.

a. Breast milk changes over time to meet the changing needs as infants grow. c. Breast milk and breastfeeding may enhance cognitive development. d. Long-term studies have shown that the benefits of breast milk continue after the infant is weaned. e. Benefits to the infant include a reduced incidence of SIDS.

The Baby Friendly Hospital Initiative endorsed by the World Health Organization (WHO) and the United Nations Children's Fund (UNICEF) was founded to encourage institutions to offer optimal levels of care for lactating mothers. Which actions are included in the "Ten Steps to Successful Breastfeeding for Hospitals"? (Select all that apply.) a. Give newborns no food or drink other than breast milk. b. Have a written breastfeeding policy that is communicated to all staff members. c. Help mothers initiate breastfeeding within hour of childbirth. d. Give artificial teats or pacifiers as necessary. e. Return infants to the nursery at night.

a. Give newborns no food or drink other than breast milk. b. Have a written breastfeeding policy that is communicated to all staff members. c. Help mothers initiate breastfeeding within hour of childbirth.

What is the correct term for a woman who has completed one pregnancy with a fetus (or fetuses) reaching the stage of fetal viability? a. Primipara b. Primigravida c. Multipara d. Nulligravida

ANS: A A primipara is a woman who has completed one pregnancy with a viable fetus. To help remember the terms: gravida is a pregnant woman; para comes from parity, meaning a viable fetus; primi means first; multi means many; and null means none. Therefore, a primigravida is a woman pregnant for the first time; a multipara is a woman who has completed two or more pregnancies with a viable fetus; and a nulligravida is a woman who has never been pregnant.

A client is scheduled for surgery after a recent breast cancer diagnosis. The nurse is discussing the procedure with the client. To allay her fears, which explanation best describes a skin-sparing mastectomy? a.Removal of the breast, nipple, and areola, leaving only the skin b.Removal of the breast, nipple, areola, and axillary node dissection c.Incision on the outside of the breast, leaving the nipple intact d.Removal of both breasts in their entirety

ANS: A A skin-sparing mastectomy is a special procedure that keeps the outer breast of the skin intact. The breast, nipple, and areola are removed. A tissue expander may be placed for later reconstruction. A modified radical mastectomy also removes the axillary lymph nodes. The nipple-sparing mastectomy is reserved for a small number of women during which the areola is removed leaving the nipple intact. Women who test positive for the BRCA1 or BRCA2 gene mutation may have both breasts removed to reduce the risk of cancer and is most commonly known as a prophylactic or preventative mastectomy.

A pregnant woman at 33 weeks of gestation is brought to the birthing unit after a minor automobile accident. The client is experiencing no pain and no vaginal bleeding, her vital signs are stable, and the FHR is 132 beats per minute with variability. What is the nurse's highest priority? a. Monitoring the woman for a ruptured spleen b. Obtaining a physician's order to discharge her home c. Monitoring her for 24 hours d. Using continuous EFM for a minimum of 4 hours

ANS: D Monitoring the external FHR and contractions is recommended after blunt trauma in a viable gestation for a minimum of 4 hours, regardless of injury severity. Fetal monitoring should be initiated as soon as the woman is stable. In this scenario, no clinical findings indicate the possibility of a ruptured spleen. If the maternal and fetal findings are normal, then EFM should continue for a minimum of 4 hours after a minor trauma or a minor automobile accident. Once the monitoring has been completed and the health care provider is reassured of fetal well-being, the client may be discharged home. Monitoring for 24 hours is unnecessary unless the ERM strip is abnormal or nonreassuring.

Nurses must be aware of the conditions that increase the risk of hemorrhage, one of the most common complications of the puerperium. What are the conditions? Select all that apply. a. Primipara b. Rapid or prolonged labor c. Overdistention of the uterus d. Uterine fibroids e. Preeclampsia

B, C, D, E

Which hormone remains elevated in the immediate postpartum period of the breastfeeding woman? a. Estrogen b. Progesterone c. Prolactin d. Human placental lactogen

C

A new father states, "I know nothing about babies," but he seems to be interested in learning. The nurse should a. Continue to observe his interaction with the newborn. b. Tell him when he does something wrong. c. Show no concern, as he will learn on his own. d. Include him in teaching sessions.

D

The woman complains that the afterpains are worse after this baby than after her first child. The nurse's best response would be that A. Afterbirth pains should be the same unless the uterus is boggy B. Afterbirth pains should be the same unless the bladder is full C. Afterbirth pains should be easier with each pregnancy D. Afterbirth pains increase in intensity with each pregnancy

D

Which nursing action is most appropriate to correct a boggy uterus that is displaced above and to the right of the umbilicus? a. Notify the physician of an impending hemorrhage. b. Assess the blood pressure and pulse. c. Evaluate the lochia. d. Assist the patient in emptying her bladder.

D

The AAP recommends pasteurized donor milk for preterm infants if the mother's own milk in not available. Which statements regarding donor milk and milk banking are important for the nurse to understand and communicate to her client? (Select all that apply.) a. All milk bank donors are screened for communicable diseases. b. Internet milk sharing is an acceptable source for donor milk. c. Donor milk may be given to transplant clients. d. Donor milk is used in neonatal intensive care units (NICUs) for severely low-birth-weight infants only. e. Donor milk may be used for children with immunoglobulin A (IgA) deficiencies.

a. All milk bank donors are screened for communicable diseases. c. Donor milk may be given to transplant clients. e. Donor milk may be used for children with immunoglobulin A (IgA) deficiencies.

The nurse should be cognizant of which statement regarding the unique qualities of human breast milk? a. Frequent feedings during predictable growth spurts stimulate increased milk production. b. Milk of preterm mothers is the same as the milk of mothers who gave birth at term. c. Milk at the beginning of the feeding is the same as the milk at the end of the feeding. d. Colostrum is an early, less concentrated, less rich version of mature milk.

a. Frequent feedings during predictable growth spurts stimulate increased milk production.

Which information should the nurse provide to a breastfeeding mother regarding optimal self-care? a. She will need an extra 1000 calories a day to maintain energy and produce milk. b. She can return to prepregnancy consumption patterns of any drinks as long as she gets enough calcium. c. She should avoid trying to lose large amounts of weight. d. She must avoid exercising because it is too fatiguing.

c. She should avoid trying to lose large amounts of weight.

In assisting the breastfeeding mother to position the baby, which information regarding positioning is important for the nurse to keep in mind? a. The cradle position is usually preferred by mothers who had a cesarean birth. b. Women with perineal pain and swelling prefer the modified cradle position. c. Whatever the position used, the infant is "belly to belly" with the mother. d. While supporting the head, the mother should push gently on the occiput.

c. Whatever the position used, the infant is "belly to belly" with the mother.

What is the primary role of the doula during labor? a. Helps the woman perform Lamaze breathing techniques and to provide support to the woman and her partner b. Checks the fetal monitor tracing for effects of the labor process on the fetal heart rate c. Takes the place of the father as a coach and support provider d. Administers pain medications as needed by the woman

ANS: A A doula is professionally trained to provide labor support, including physical, emotional, and informational support, to both the woman and her partner during labor and the birth. The doula does not become involved with clinical tasks.

What is important for the nurse to recognize regarding the new father and his acceptance of the pregnancy and preparation for childbirth? a. The father goes through three phases of acceptance of his own. b. The fathers attachment to the fetus cannot be as strong as that of the mother because it does not start until after the birth. c. In the last 2 months of pregnancy, most expectant fathers suddenly get very protective of their established lifestyle and resist making changes to the home. d. Typically, men remain ambivalent about fatherhood right up to the birth of their child.

ANS: A A father typically goes through three phases of development to reach acceptance of fatherhood: the announcement phase, the moratorium phase, and the focusing phase. The father-child attachment can be as strong as the mother-child relationship and can also begin during pregnancy. During the last 2 months of the pregnancy, many expectant fathers work hard to improve the environment of the home for the child. Typically, the expectant fathers ambivalence ends by the first trimester, and he progresses to adjusting to the reality of the situation and then to focusing on his role.

A client at 39 weeks of gestation has been admitted for an external version. Which intervention would the nurse anticipate the provider to order? a.Tocolytic drug b.Contraction stress test (CST) c.Local anesthetic d.Foley catheter

ANS: A A tocolytic drug will relax the uterus before and during the version, thus making manipulation easier. CST is used to determine the fetal response to stress. A local anesthetic is not used with external version. Although the bladder should be emptied, catheterization is not necessary.

The nurse has formulated a diagnosis of Imbalanced nutrition: Less than body requirements for the client. Which goal is most appropriate for this client to obtain? a. Gain a total of 30 pounds. b. Consistently take daily supplements. c. Decrease her intake of snack foods. d. Increase her intake of complex carbohydrates.

ANS: A A weight gain of 30 pounds is one indication that the client has gained a sufficient amount for the nutritional needs of pregnancy. A daily supplement is not the best goal for this client and does not meet the basic need of proper nutrition during pregnancy. Decreasing snack foods may be needed and should be assessed; however, assessing weight gain is the best method of monitoring nutritional intake for this pregnant client. Although increasing the intake of complex carbohydrates is important for this client, monitoring the weight gain should be the end goal.

Which information regarding the procedures and criteria for admitting a woman to the hospital labor unit is important for the nurse to understand? a. Client is considered to be in active labor when she arrives at the facility with contractions. b. Client can have only her male partner or predesignated doula with her at assessment. c. Children are not allowed on the labor unit. d. Non-English speaking client must bring someone to translate.

ANS: A According to the Emergency Medical Treatment and Active Labor Act (EMTALA), a woman is entitled to active labor care and is presumed to be in true labor until a qualified health care provider certifies otherwise. A woman may have anyone she wishes present for her support. An interpreter must be provided by the hospital, either in person or by a telephonic service. Siblings of the new infant may be allowed at the delivery, depending on hospital policy and adequate preparation and supervision.

Nursing care measures are commonly offered to women in labor. Which nursing measure reflects the application of the gate-control theory? a.Massage the woman's back. b.Change the woman's position. c.Give the prescribed medication. d.Encourage the woman to rest between contractions.

ANS: A According to the gate-control theory, pain sensations travel along sensory nerve pathways to the brain, but only a limited number of sensations, or messages, can travel through these nerve pathways at one time. Distraction techniques, such as massage or stroking, music, focal points, and imagery, reduce or completely block the capacity of the nerve pathways to transmit pain. These distractions are thought to work by closing down a hypothetic gate in the spinal cord, thus preventing pain signals from reaching the brain. The perception of pain is thereby diminished. Changing the woman's position, administering pain medication, and resting between contractions do not reduce or block the capacity of the nerve pathways to transmit pain using the gate-control theory.

Which information is the highest priority for the nurse to comprehend regarding the BPP? a. BPP is an accurate indicator of impending fetal well-being. b. BPP is a compilation of health risk factors of the mother during the later stages of pregnancy. c. BPP consists of a Doppler blood flow analysis and an amniotic fluid index (AFI). d. BPP involves an invasive form of an ultrasonic examination.

ANS: A An abnormal BPP score is one indication that labor should be induced. The BPP evaluates the health of the fetus, requires many different measures, and is a noninvasive procedure.

What kind of fetal anomalies are most often associated with oligohydramnios? a. Renal b. Cardiac c. Gastrointestinal d. Neurologic

ANS: A An amniotic fluid volume of less than 300 ml (oligohydramnios) is often associated with fetal renal anomalies. The amniotic fluid volume has no bearing on the fetal cardiovascular system. Gastrointestinal anomalies are associated with hydramnios or an amniotic fluid volume greater than 2 L. The amniotic fluid volume has no bearing on the fetal neurologic system.

A 39-year-old primigravida woman believes that she is approximately 8 weeks pregnant, although she has had irregular menstrual periods all her life. She has a history of smoking approximately one pack of cigarettes a day; however, she tells the nurse that she is trying to cut down. Her laboratory data are within normal limits. What diagnostic technique would be useful at this time? a. Ultrasound examination b. Maternal serum alpha-fetoprotein (MSAFP) screening c. Amniocentesis d. Nonstress test (NST)

ANS: A An ultrasound examination could be performed to confirm the pregnancy and to determine the gestational age of the fetus. An MSAFP screening is performed at 16 to 18 weeks of gestation; therefore, it is too early in the womans pregnancy to perform this diagnostic test. An amniocentesis is performed if the MSAFP levels are abnormal or if fetal or maternal anomalies are detected. An NST is performed to assess fetal well-being in the third trimester.

A woman in preterm labor at 30 weeks of gestation receives two 12-mg intramuscular (IM) doses of betamethasone. What is the purpose of this pharmacologic intervention? a.To stimulate fetal surfactant production b.To reduce maternal and fetal tachycardia associated with ritodrine administration c.To suppress uterine contractions d.To maintain adequate maternal respiratory effort and ventilation during magnesium sulfate therapy

ANS: A Antenatal glucocorticoids administered as IM injections to the mother accelerate fetal lung maturity. Propranolol (Inderal) is given to reduce the effects of ritodrine administration. Betamethasone has no effect on uterine contractions. Calcium gluconate is given to reverse the respiratory depressive effects of magnesium sulfate therapy.

What is the highest priority nursing intervention when admitting a pregnant woman who has experienced a bleeding episode in late pregnancy? a.Assessing FHR and maternal vital signs b.Performing a venipuncture for hemoglobin and hematocrit levels c.Placing clean disposable pads to collect any drainage d.Monitoring uterine contractions

ANS: A Assessment of the FHR and maternal vital signs will assist the nurse in determining the degree of the blood loss and its effect on the mother and fetus. The most important assessment is to check the well-being of both the mother and the fetus. The blood levels can be obtained later. Assessing future bleeding is important; however, the top priority remains mother/fetal well-being. Monitoring uterine contractions is important but not a top priority.

Which description most accurately describes the augmentation of labor? a.Is part of the active management of labor that is instituted when the labor process is unsatisfactory b.Relies on more invasive methods when oxytocin and amniotomy have failed c.Is a modern management term to cover up the negative connotations of forceps-assisted birth d.Uses vacuum cups

ANS: A Augmentation is part of the active management of labor that stimulates uterine contractions after labor has started but is not progressing satisfactorily. Augmentation uses amniotomy and oxytocin infusion, as well as some more gentle, noninvasive methods. Forceps-assisted births are less common than in the past and not considered a method of augmentation. A vacuum-assisted delivery occurs during childbirth if the mother is too exhausted to push. Vacuum extraction is not considered an augmentation methodology.

Of these psychosocial factors, which has the least negative effect on the health of the mother and/or fetus? a. Moderate coffee consumption b. Moderate alcohol consumption c. Cigarette smoke d. Emotional distress

ANS: A Birth defects in humans have not been related to caffeine consumption. Pregnant women who consume more than 300 mg of caffeine daily may be at increased risk for miscarriage or IUGR. Although the exact effects of alcohol in pregnancy have not been quantified, it exerts adverse effects on the fetus including fetal alcohol syndrome, fetal alcohol effects, learning disabilities, and hyperactivity. A strong, consistent, causal relation has been established between maternal smoking and reduced birth weight. Childbearing triggers profound and complex physiologic and psychologic changes on the mother. Evidence suggests a relationship between emotional distress and birth complications.

A pregnant woman reports that she is still playing tennis at 32 weeks of gestation. Which recommendation would the nurse make for this particular client after a tennis match? a. Drink several glasses of fluid. b. Eat extra protein sources such as peanut butter. c. Enjoy salty foods to replace lost sodium. d. Consume easily digested sources of carbohydrate.

ANS: A If no medical or obstetric problems contraindicate physical activity, then pregnant women should get 30 minutes of moderate physical exercise daily. Liberal amounts of fluid should be consumed before, during, and after exercise because dehydration can trigger premature labor. The womans caloric intake should be sufficient to meet the increased needs of pregnancy and the demands of exercise.

With regard to medications, herbs, boosters, and other substances normally encountered by pregnant women, what is important for the nurse to be aware of? a. Both prescription and over-the-counter (OTC) drugs that otherwise are harmless can be made hazardous by metabolic deficiencies of the fetus. b. The greatest danger of drug-caused developmental deficits in the fetus is observed in the final trimester. c. Killed-virus vaccines (e.g., tetanus) should not be administered during pregnancy, but live-virus vaccines (e.g., measles) are permissible. d. No convincing evidence exists that secondhand smoke is potentially dangerous to the fetus.

ANS: A Both prescription and OTC drugs that otherwise are harmless can be made hazardous by metabolic deficiencies of the fetus. This is especially true for new medications and combinations of drugs. The greatest danger of drug-caused developmental defects exists in the interval from fertilization through the first trimester, during which a woman may not realize that she is pregnant. Live-virus vaccines should be part of postpartum care; killed-virus vaccines may be administered during pregnancy. Secondhand smoke is associated with fetal growth restriction and increases in infant mortality.

When managing the care of a woman in the second stage of labor, the nurse uses various measures to enhance the progress of fetal descent. Which instruction best describes these measures? a. Encouraging the woman to try various upright positions, including squatting and standing b. Telling the woman to start pushing as soon as her cervix is fully dilated c. Continuing an epidural anesthetic so pain is reduced and the woman can relax d. Coaching the woman to use sustained, 10- to 15-second, closed-glottis bearing-down efforts with each contraction

ANS: A Both upright and squatting positions may enhance the progress of fetal descent. Many factors dictate when a woman should begin pushing. Complete cervical dilation is necessary, but complete dilation is only one factor. If the fetal head is still in a higher pelvic station, then the physician or midwife may allow the woman to "labor down" if the woman is able (allowing more time for fetal descent and thereby reducing the amount of pushing needed). The epidural may mask the sensations and muscle control needed for the woman to push effectively. Closed glottic breathing may trigger the Valsalva maneuver, which increases intrathoracic and cardiovascular pressures, reducing cardiac output and inhibiting perfusion of the uterus and placenta. In addition, holding her breath for longer than 5 to 7 seconds diminishes the perfusion of oxygen across the placenta and results in fetal hypoxia.

Which intervention is most important when planning care for a client with severe gestational hypertension? a. Induction of labor is likely, as near term as possible. b. If at home, the woman should be confined to her bed, even with mild gestational hypertension. c. Special diet low in protein and salt should be initiated. d. Vaginal birth is still an option, even in severe cases.

ANS: A By 34 weeks of gestation, the risk of continuing the pregnancy may be considered greater than the risks of a preterm birth. Strict bed rest is controversial for mild cases; some women in the hospital are even allowed to move around. Diet and fluid recommendations are essentially the same as for healthy pregnant women, although some authorities have suggested a diet high in protein. Women with severe gestational hypertension should expect a cesarean delivery.

The nurse should be cognizant of which important information regarding nerve block analgesia and anesthesia? a.Most local agents are chemically related to cocaine and end in the suffix -caine. b.Local perineal infiltration anesthesia is effective when epinephrine is added, but it can be injected only once. c.Pudendal nerve block is designed to relieve the pain from uterine contractions. d.Pudendal nerve block, if performed correctly, does not significantly lessen the bearing-down reflex.

ANS: A Common agents include lidocaine and chloroprocaine. Injections can be repeated to prolong the anesthesia. A pudendal nerve block relieves pain in the vagina, vulva, and perineum but not the pain from uterine contractions. A pudendal nerve block lessens or shuts down the bearing-down reflex.

Which order should the nurse expect for a client admitted with a threatened abortion? a.Bed rest b.Administration of ritodrine IV c.Nothing by mouth (nil per os [NPO]) d.Narcotic analgesia every 3 hours, as needed

ANS: A Decreasing the woman's activity level may alleviate the bleeding and allow the pregnancy to continue. Ritodrine is not the first drug of choice for tocolytic medications. Having the woman placed on NPO is unnecessary. At times, dehydration may produce contractions; therefore, hydration is important. Narcotic analgesia will not decrease the contractions and may mask the severity of the contractions.

Fibrocystic changes in the breast most often appear in women in their 20s and 30s. Although the cause is unknown, an imbalance of estrogen and progesterone may be the cause. The nurse who cares for this client should be aware that treatment modalities are conservative. Which proven modality may offer relief for this condition? a.Diuretic administration b.Daily inclusion of caffeine in the diet c.Increased vitamin C supplementation d.Application of cold packs to the breast as necessary

ANS: A Diuretic administration plus a decrease in sodium and fluid intake are recommended. Although not supported by research, some advocate eliminating dimethylxanthines (caffeine) from the diet. Smoking should also be avoided, and alcohol consumption should be reduced. Vitamin E supplements are recommended; however, the client should avoid megadoses because vitamin E is a fat-soluble vitamin. Pain relief measures include applying heat to the breast, wearing a supportive bra, and taking nonsteroidal antiinflammatory drugs.

The nurse sees a woman for the first time when she is 30 weeks pregnant. The client has smoked throughout the pregnancy, and fundal height measurements now are suggestive of intrauterine growth restriction (IUGR) in the fetus. In addition to ultrasound to measure fetal size, what is another tool useful in confirming the diagnosis? a. Doppler blood flow analysis b. Contraction stress test (CST) c. Amniocentesis d. Daily fetal movement counts

ANS: A Doppler blood flow analysis allows the examiner to study the blood flow noninvasively in the fetus and the placenta. It is a helpful tool in the management of high-risk pregnancies because of IUGR, diabetes mellitus, multiple fetuses, or preterm labor. Because of the potential risk of inducing labor and causing fetal distress, a CST is not performed on a woman whose fetus is preterm. Indications for an amniocentesis include diagnosis of genetic disorders or congenital anomalies, assessment of pulmonary maturity, and the diagnosis of fetal hemolytic disease, not IUGR. Fetal kick count monitoring is performed to monitor the fetus in pregnancies complicated by conditions that may affect fetal oxygenation. Although this may be a useful tool at some point later in this womans pregnancy, it is not used to diagnose IUGR.

A woman is in her seventh month of pregnancy. She has been complaining of nasal congestion and occasional epistaxis. Which statement best describes why this may be happening to this client? a. This respiratory change is normal in pregnancy and caused by an elevated level of estrogen. b. This cardiovascular change is abnormal, and the nosebleeds are an ominous sign. c. The woman is a victim of domestic violence and is being hit in the face by her partner. d. The woman has been intranasally using cocaine.

ANS: A Elevated levels of estrogen cause capillaries to become engorged in the respiratory tract, which may result in edema in the nose, larynx, trachea, and bronchi. This congestion may cause nasal stuffiness and epistaxis. Cardiovascular changes in pregnancy may cause edema in the lower extremities. Domestic violence cannot be determined on the basis on the sparse facts provided. If the woman had been hit in the face, then she most likely would have additional physical findings. Cocaine use cannot be determined on the basis on the sparse facts provided.

A woman asks the nurse, What protects my babys umbilical cord from being squashed while the babys inside of me? What is the nurses best response? a. Your babys umbilical cord is surrounded by connective tissue called Whartons jelly, which prevents compression of the blood vessels. b. Your babys umbilical cord floats around in blood and amniotic fluid. c. You dont need to be worrying about things like that. d. The umbilical cord is a group of blood vessels that are very well protected by the placenta.

ANS: A Explaining the structure and function of the umbilical cord is the most appropriate response. Connective tissue called Whartons jelly surrounds the umbilical cord, prevents compression of the blood vessels, and ensures continued nourishment of the embryo or fetus. The umbilical cord does not float around in blood or fluid. Telling the client not to worry negates her need for information and discounts her feelings. The placenta does not protect the umbilical cord.

Which term best describes the conscious decision concerning when to conceive or avoid pregnancy as opposed to the intentional prevention of pregnancy during intercourse? a. Family planning b. Birth control c. Contraception d. Assisted reproductive therapy

ANS: A Family planning is the process of deciding when and if to have children. Birth control is the device and/or practice used to reduce the risk of conceiving or bearing children. Contraception is the intentional prevention of pregnancy during sexual intercourse. Assisted reproductive therapy is one of several possible treatments for infertility.

Which client would not be a suitable candidate for internal EFM? a.Client who still has intact membranes b.Woman whose fetus is well engaged in the pelvis c.Pregnant woman who has a comorbidity of obesity d.Client whose cervix is dilated to 4 to 5 cm

ANS: A For internal EFM, the membranes must have ruptured and the cervix must be dilated at least 2 to 3 cm. The presenting part must be low enough to allow placement of the spiral electrode necessary for internal EFM. The accuracy of EFM is not affected by maternal size. However, evaluating fetal well-being using external EFM may be more difficult on an obese client. The client whose cervix is dilated to 4 to 5 cm is indeed a candidate for internal monitoring.

What is the correct name describing a benign breast condition that includes dilation and inflammation of the collecting ducts? a.Mammary duct ectasia b.Intraductal papilloma c.Chronic cystic disease d.Fibroadenoma

ANS: A Generally occurring in women approaching menopause, mammary duct ectasia results in a firm irregular mass in the breast, enlarged axillary nodes, and nipple discharge. Intraductal papillomas develop in the epithelium of the ducts of the breasts; as the mass grows, it causes trauma or erosion within the ducts. Chronic cystic disease causes pain and tenderness. The cysts that form are multiple, smooth, and well delineated. Fibroadenoma is evidenced by fibrous and glandular tissues. They are felt as firm, rubbery, and freely mobile nodules.

Which information regarding protein in the diet of a pregnant woman is most helpful to the client? a. Many protein-rich foods are also good sources of calcium, iron, and B vitamins. b. Many women need to increase their protein intake during pregnancy. c. As with carbohydrates and fat, no specific recommendations exist for the amount of protein in the diet. d. High-protein supplements can be used without risk by women on macrobiotic diets.

ANS: A Good sources for protein, such as meat, milk, eggs, and cheese, have a lot of calcium and iron. Most women already eat a high-protein diet and do not need to increase their intake. Protein is sufficiently important that specific servings of meat and dairy are recommended. High-protein supplements are not recommended because they have been associated with an increased incidence of preterm births.

As the United States and Canada continue to become more culturally diverse, recognizing a wide range of varying cultural beliefs and practices is increasingly important for the nursing staff. A client is from which country if she requests to have the baby's father in attendance? a. Mexico b. China c. Iran d. India

ANS: A Hispanic women routinely have fathers and female relatives in attendance during the second stage of labor. The father of the baby is expected to provide encouragement, support, and reassurance that all will be well. In China, fathers are usually not present. The side-lying position is preferred for labor and birth because it is believed that this will reduce trauma to the infant. In China, the client has a stoic response to pain. In Iran, the father will not be present. Female support persons and female health care providers are preferred. For many, a male caregiver is unacceptable. In India, the father is usually not present, but female relatives are usually in attendance. Natural childbirth methods are preferred.

A woman with severe preeclampsia has been receiving magnesium sulfate by intravenous infusion for 8 hours. The nurse assesses the client and documents the following findings: temperature of 37.1 C, pulse rate of 96 beats per minute, respiratory rate of 24 breaths per minute, BP of 155/112 mm Hg, 3+ DTRs, and no ankle clonus. The nurse calls the provider with an update. The nurse should anticipate an order for which medication? a. Hydralazine b. Magnesium sulfate bolus c. Diazepam d. Calcium gluconate

ANS: A Hydralazine is an antihypertensive medication commonly used to treat hypertension in severe preeclampsia. Typically, it is administered for a systolic BP higher than 160 mm Hg or a diastolic BP higher than 110 mm Hg. An additional bolus of magnesium sulfate may be ordered for increasing signs of CNS irritability related to severe preeclampsia (e.g., clonus) or if eclampsia develops. Diazepam is sometimes used to stop or shorten eclamptic seizures. Calcium gluconate is used as the antidote for magnesium sulfate toxicity. The client is not currently displaying any signs or symptoms of magnesium toxicity.

In recovery, if a woman is asked to either raise her legs (knees extended) off the bed or flex her knees, and then place her feet flat on the bed and raise her buttocks well off the bed, the purpose of this exercise is to assess what? a. Recovery from epidural or spinal anesthesia b. Hidden bleeding underneath her c. Flexibility d. Whether the woman is a candidate to go home after 6 hours

ANS: A If the numb or prickly sensations are gone from her legs after these movements, then she has likely recovered from the epidural or spinal anesthesia. Assessing the client for bleeding beneath her buttocks before discharge from the recovery is always important; however, she should be rolled to her side for this assessment. The nurse is not required to assess the woman for flexibility. This assessment is performed to evaluate whether the client has recovered from spinal anesthesia, not to determine if she is a candidate for early discharge.

An expectant couple attending childbirth classes have questions regarding multiple births since twins run in the family. What information regarding multiple births is important for the nurse to share? a. Twinning and other multiple births are increasing because of the use of fertility drugs and delayed childbearing. b. Dizygotic twins (two fertilized ova) have the potential to be conjoined twins. c. Identical twins are more common in Caucasian families. d. Fraternal twins are the same gender, usually male.

ANS: A If the parents-to-be are older and have taken fertility drugs, then they would be very interested to know about twinning and other multiple births. Conjoined twins are monozygotic; that is, they are from a single fertilized ovum in which division occurred very late. Identical twins show no racial or ethnic preference, and fraternal twins are more common among African-American women. Fraternal twins can be different genders or the same gender, and identical twins are the same gender.

The exact cause of preterm labor is unknown but believed to be multifactorial. Infection is thought to be a major factor in many preterm labors. Which type of infection has not been linked to preterm birth? a.Viral b.Periodontal c.Cervical d.Urinary tract

ANS: A Infections that increase the risk of preterm labor and birth are bacterial and include cervical, urinary tract, periodontal, and other bacterial infections. Therefore, early, continual, and comprehensive participation by the client in her prenatal care is important. Recent evidence has shown a link between periodontal infections and preterm labor. Researchers recommend regular dental care before and during pregnancy, oral assessment as a routine part of prenatal care, and scrupulous oral hygiene to prevent periodontal infections.

While evaluating an external monitor tracing of a woman in active labor, the nurse notes that the FHR for five sequential contractions begins to decelerate late in the contraction, with the nadir of the decelerations occurring after the peak of the contraction. What is the nurse's first priority? a.Change the woman's position. b.Notify the health care provider. c.Assist with amnioinfusion d.Insert a scalp electrode.

ANS: A Late FHR decelerations may be caused by maternal supine hypotension syndrome. These decelerations are usually corrected when the woman turns onto her side to displace the weight of the gravid uterus from the vena cava. If the fetus does not respond to primary nursing interventions for late decelerations, then the nurse should continue with subsequent intrauterine resuscitation measures and notify the health care provider. An amnioinfusion may be used to relieve pressure on an umbilical cord that has not prolapsed. The FHR pattern associated with this situation most likely will reveal variable decelerations. Although a fetal scalp electrode will provide accurate data for evaluating the well-being of the fetus, it is not a nursing intervention that will alleviate late decelerations nor is it the nurse's first priority.

What is the most common reason for late postpartum hemorrhage (PPH)? a.Subinvolution of the uterus b.Defective vascularity of the decidua c.Cervical lacerations d.Coagulation disorders

ANS: A Late PPH may be the result of subinvolution of the uterus. Recognized causes of subinvolution include retained placental fragments and pelvic infection. Although defective vascularity, cervical lacerations, and coagulation disorders of the decidua may also cause PPH, late PPH typically results from subinvolution of the uterus, pelvic infection, or retained placental fragments.

With one exception, the safest pregnancy is one during which the woman is drug and alcohol free. What is the optimal treatment for women addicted to opioids? a. Methadone maintenance treatment (MMT) b. Detoxification c. Smoking cessation d. 4 Ps Plus

ANS: A MMT is currently considered the standard of care for pregnant women who are dependent on heroin or other narcotics. Buprenorphine is another medication approved for the treatment of opioid addiction that is increasingly being used during pregnancy. Opioid replacement therapy has been shown to decrease opioid and other drug use, reduce criminal activity, improve individual functioning, and decrease the rates of infections such as hepatitis B and C, human immunodeficiency virus (HIV), and other STIs. Detoxification is the treatment used for alcohol addiction. Pregnant women requiring withdrawal from alcohol should be admitted for inpatient management. Women are more likely to stop smoking during pregnancy than at any other time in their lives. A smoking cessation program can assist in achieving this goal. The 4 Ps Plus is a screening tool specifically designed to identify pregnant women who need in-depth assessment related to substance abuse.

Which nursing intervention would result in an increase in maternal cardiac output? a.Change in position b.Oxytocin administration c.Regional anesthesia d.IV analgesic

ANS: A Maternal supine hypotension syndrome is caused by the weight and pressure of the gravid uterus on the ascending vena cava when the woman is in a supine position. This position reduces venous return to the woman's heart, as well as cardiac output, and subsequently reduces her blood pressure. The nurse can encourage the woman to change positions and to avoid the supine position. Oxytocin administration, regional anesthesia, and IV analgesic may reduce maternal cardiac output.

A postmenopausal woman has been diagnosed with two leiomyomas (fibroids). Which clinical finding is most commonly associated with the presence of leiomyomas? a.Abnormal uterine bleeding b.Diarrhea c.Weight loss d.Acute abdominal pain

ANS: A Most women are asymptomatic. Abnormal uterine bleeding is the most common symptom of leiomyomas. Diarrhea is not commonly associated with leiomyomas. Weight loss does not usually occur in the woman with leiomyomas, and the client with leiomyomas is unlikely to experience abdominal pain.

Which neonatal complications are associated with hypertension in the mother? a. Intrauterine growth restriction (IUGR) and prematurity b. Seizures and cerebral hemorrhage c. Hepatic or renal dysfunction d. Placental abruption and DIC

ANS: A Neonatal complications are related to placental insufficiency and include IUGR, prematurity, and necrotizing enterocolitis. Seizures and cerebral hemorrhage are maternal complications. Hepatic and renal dysfunction are maternal complications of hypertensive disorders in pregnancy. Placental abruption and DIC are conditions related to maternal morbidity and mortality.

What are the legal responsibilities of the perinatal nurses? a.Correctly interpreting FHR patterns, initiating appropriate nursing interventions, and documenting the outcomes b.Greeting the client on arrival, assessing her status, and starting an IV line c.Applying the external fetal monitor and notifying the health care provider d.Ensuring that the woman is comfortable

ANS: A Nurses who care for women during childbirth are legally responsible for correctly interpreting FHR patterns, initiating appropriate nursing interventions based on those patterns, and documenting the outcomes of those interventions. Greeting the client on arrival, assessing her, and starting an IV line are activities that should be performed when any client arrives to the maternity unit. The nurse is not the only one legally responsible for performing these functions. Applying the external fetal monitor and notifying the health care provider is a nursing function that is part of the standard of care for all obstetric clients and falls within the registered nurse's scope of practice. Everyone caring for the pregnant woman should ensure that both she and her support partner are comfortable.

Which statement best describes a normal uterine activity pattern in labor? a.Contractions every 2 to 5 minutes b.Contractions lasting approximately 2 minutes c.Contractions approximately 1 minute apart d.Contraction intensity of approximately 500 mm Hg with relaxation at 50 mm Hg

ANS: A Overall contraction frequency generally ranges from two to five contractions per 10 minutes of labor, with lower frequencies during the first stage and higher frequencies observed during the second stage. Contraction duration remains fairly stable throughout the first and second stages, ranging from 45 to 80 seconds, generally not exceeding 90 seconds. Contractions 1 minute apart are occurring too often and would be considered an abnormal labor pattern. The intensity of uterine contractions generally ranges from 25 to 50 mm Hg in the first stage of labor and may rise to more than 80 mm Hg in the second stage.

Which symptom described by a client is characteristic of premenstrual syndrome (PMS)? a. I feel irritable and moody a week before my period is supposed to start. b. I have lower abdominal pain beginning on the third day of my menstrual period. c. I have nausea and headaches after my period starts, and they last 2 to 3 days. d. I have abdominal bloating and breast pain after a couple days of my period.

ANS: A PMS is a cluster of physical, psychologic, and behavioral symptoms that begin in the luteal phase of the menstrual cycle and resolve within a couple of days of the onset of menses. Complaints of lower abdominal pain, nausea and headaches, and abdominal bloating all are associated with PMS; however, the timing reflected is inaccurate

When assessing a multiparous woman who has just given birth to an 8-pound boy, the nurse notes that the woman's fundus is firm and has become globular in shape. A gush of dark red blood comes from her vagina. What is the nurse's assessment of the situation? a. The placenta has separated. b. A cervical tear occurred during the birth. c. The woman is beginning to hemorrhage. d. Clots have formed in the upper uterine segment.

ANS: A Placental separation is indicated by a firmly contracting uterus, a change in the uterus from a discoid to a globular ovoid shape, a sudden gush of dark red blood from the introitus, an apparent lengthening of the umbilical cord, and a finding of vaginal fullness. Cervical tears that do not extend to the vagina result in minimal blood loss. Signs of hemorrhage are a boggy uterus, bright red vaginal bleeding, alterations in vital signs, pallor, lightheadedness, restlessness, decreased urinary output, and alteration in the level of consciousness. If clots have formed in the upper uterine segment, then the nurse would expect to find the uterus boggy and displaced to the side.

A 21yearold client complains of severe pain immediately after the commencement of her menses. Which gynecologic condition is the most likely cause of this clients presenting complaint? a. Primary dysmenorrhea b. Secondary dysmenorrhea c. Dyspareunia d. Endometriosis

ANS: A Primary dysmenorrhea, or pain during or shortly before menstruation, has a biochemical basis and arises from the release of prostaglandins with menses. Secondary dysmenorrhea develops after the age of 25 years and is usually associated with a pelvic pathologic condition. Dyspareunia, or painful intercourse, is commonly associated with endometriosis. Endometriosis is characterized by endometrial glands and stoma outside of the uterus.

In caring for the woman with DIC, which order should the nurse anticipate? a.Administration of blood b.Preparation of the client for invasive hemodynamic monitoring c.Restriction of intravascular fluids d.Administration of steroids

ANS: A Primary medical management in all cases of DIC involves a correction of the underlying cause, volume replacement, blood component therapy, optimization of oxygenation and perfusion status, and continued reassessment of laboratory parameters. Central monitoring would not be initially ordered in a client with DIC because it could contribute to more areas of bleeding. Management of DIC would include volume replacement, not volume restriction. Steroids are not indicated for the management of DIC.

A new mother asks the nurse about the white substance covering her infant. How should the nurse explain the purpose of vernix caseosa? a. Vernix caseosa protects the fetal skin from the amniotic fluid. b. Vernix caseosa promotes the normal development of the peripheral nervous system. c. Vernix caseosa allows the transport of oxygen and nutrients across the amnion. d. Vernix caseosa regulates fetal temperature.

ANS: A Prolonged exposure to the amniotic fluid during the fetal period could result in the breakdown of the skin without the protection of the vernix caseosa. Normal development of the peripheral nervous system was dependent on nutritional intake of the mother. The amnion was the inner membrane that surrounded the fetus and was not involved in the oxygen and nutrient exchange. The amniotic fluid helped maintain fetal temperature.

Which statement concerning the complication of maternal diabetes is the most accurate? a. Diabetic ketoacidosis (DKA) can lead to fetal death at any time during pregnancy. b. Hydramnios occurs approximately twice as often in diabetic pregnancies than in nondiabetic pregnancies. c. Infections occur about as often and are considered about as serious in both diabetic and nondiabetic pregnancies. d. Even mild-to-moderate hypoglycemic episodes can have significant effects on fetal well-being.

ANS: A Prompt treatment of DKA is necessary to save the fetus and the mother. Hydramnios occurs 10 times more often in diabetic pregnancies. Infections are more common and more serious in pregnant women with diabetes. Mild-to-moderate hypoglycemic episodes do not appear to have significant effects on fetal well-being.

After the nurse completes nutritional counseling for a pregnant woman, she asks the client to repeat the instructions to assess the clients understanding. Which statement indicates that the client understands the role of protein in her pregnancy? a. Protein will help my baby grow. b. Eating protein will prevent me from becoming anemic. c. Eating protein will make my baby have strong teeth after he is born. d. Eating protein will prevent me from being diabetic.

ANS: A Protein is the nutritional element basic to growth. An adequate protein intake is essential to meeting the increasing demands of pregnancy. These demands arise from the rapid growth of the fetus; the enlargement of the uterus, mammary glands, and placenta; the increase in the maternal blood volume; and the formation of the amniotic fluid. Iron intake prevents anemia. Calcium intake is needed for fetal bone and tooth development. Glycemic control is needed in those with diabetes; protein is one nutritional factor to consider for glycemic control but not the primary role of protein intake.

The most conservative approach for early breast cancer treatment involves lumpectomy followed by which procedure? a.Radiation b.Adjuvant systemic therapy c.Hormonal therapy d.Chemotherapy

ANS: A Radiation therapy, in the form of either brachytherapy or accelerated breast radiation, is the standard therapy after lumpectomy for the treatment of early-stage breast cancer. Chemotherapy administered soon after surgical removal of the tumor is referred to as adjuvant chemotherapy. Not all women are candidates for hormonal therapy. After the entire tumor or portion is removed by excision, a receptor assay must be performed. Chemotherapy with multiple-drug combinations is used in the treatment of recurrent and advanced breast cancer with positive results.

What nursing diagnosis is the most appropriate for a woman experiencing severe preeclampsia? a. Risk for injury to mother and fetus, related to central nervous system (CNS) irritability b. Risk for altered gas exchange c. Risk for deficient fluid volume, related to increased sodium retention secondary to the administration of magnesium sulfate d. Risk for increased cardiac output, related to the use of antihypertensive drugs

ANS: A Risk for injury is the most appropriate nursing diagnosis for this client scenario. Gas exchange is more likely to become impaired, attributable to pulmonary edema. A risk for excess, not deficient, fluid volume, related to increased sodium retention, is increased, and a risk for decreased, not increased, cardiac output, related to the use of antihypertensive drugs, also is increased.

Which alterations in the perception of pain by a laboring client should the nurse understand? a.Sensory pain for nulliparous women is often greater than for multiparous women during early labor. b.Affective pain for nulliparous women is usually less than for multiparous women throughout the first stage of labor. c.Women with a history of substance abuse experience more pain during labor. d.Multiparous women have more fatigue from labor and therefore experience more pain.

ANS: A Sensory pain is greater for nulliparous women because their reproductive tract structures are less supple. Affective pain is greater for nulliparous women during the first stage but decreases for both nulliparous and multiparous during the second stage. Women with a history of substance abuse experience the same amount of pain as those without such a history. Nulliparous women have longer labors and therefore experience more fatigue.

With regard to hemorrhagic complications that may occur during pregnancy, what information is most accurate? a.An incompetent cervix is usually not diagnosed until the woman has lost one or two pregnancies. b.Incidences of ectopic pregnancy are declining as a result of improved diagnostic techniques. c.One ectopic pregnancy does not affect a woman's fertility or her likelihood of having a normal pregnancy the next time. d.Gestational trophoblastic neoplasia (GTN) is one of the persistently incurable gynecologic malignancies.

ANS: A Short labors and recurring losses of pregnancy at progressively earlier gestational ages are characteristics of reduced cervical competence. Because diagnostic technology is improving, more ectopic pregnancies are being diagnosed. One ectopic pregnancy places the woman at increased risk for another one. Ectopic pregnancy is a leading cause of infertility. Once invariably fatal, GTN now is the most curable gynecologic malignancy.

Which nutritional recommendation regarding fluids is accurate? a. A womans daily intake should be six to eight glasses of water, milk, and/or juice. b. Coffee should be limited to no more than 2 cups, but tea and cocoa can be consumed without worry. c. Of the artificial sweeteners, only aspartame has not been associated with any maternity health concerns. d. Water with fluoride is especially encouraged because it reduces the childs risk of tooth decay.

ANS: A Six to eight glasses is still the standard for fluids; however, they should be the right fluids. All beverages containing caffeine, including tea, cocoa, and some soft drinks, should be avoided or should be consumed only in limited amounts. Artificial sweeteners, including aspartame, have no ill effects on the normal mother or fetus. However, mothers with phenylketonuria (PKU) should avoid aspartame. Although no evidence indicates that prenatal fluoride consumption reduces childhood tooth decay, fluoride still helps the mother.

Breathing patterns are taught to laboring women. Which breathing pattern should the nurse support for the woman and her coach during the latent phase of the first stage of labor if the couple has attended childbirth preparation classes? a.Slow-paced breathing b.Deep abdominal breathing c.Modified-paced breathing d.Patterned-paced breathing

ANS: A Slow-paced breathing is approximately one half the woman's normal breathing rate and is used during the early stages of labor when a woman can no longer walk or talk through her contractions. No such pattern called deep abdominal breathing exists in childbirth preparation. Modified-paced breathing is shallow breathing that is twice the woman's normal breathing rate. It is used when labor progresses and the woman can no longer maintain relaxation through paced breathing. Patterned-pace breathing is a fast, 4:1 breathe, breathe, breathe, blow pattern that is used during the transitional phase of labor just before pushing and delivery.

A first-time mother at 18 weeks of gestation is in for her regularly scheduled prenatal visit. The client tells the nurse that she is afraid that she is going into premature labor because she is beginning to have regular contractions. The nurse explains that these are Braxton Hicks contractions. What other information is important for the nurse to share? a. Braxton Hicks contractions should be painless. b. They may increase in frequency with walking. c. These contractions might cause cervical dilation. d. Braxton Hicks contractions will impede oxygen flow to the fetus.

ANS: A Soon after the fourth month of gestation, uterine contractions can be felt through the abdominal wall. Braxton Hicks contractions are regular and painless and continue throughout the pregnancy. Although they are not painful, some women complain that they are annoying. This type of contraction usually ceases with walking or exercise. Braxton Hicks contractions can be mistaken for true labor; however, they do not increase in intensity, frequency, or cause cervical dilation. These contractions facilitate uterine blood flow through the intervillous spaces of the placenta and thereby promote oxygen delivery to the fetus.

Which technique is an adequate means of controlling the birth of the fetal head during delivery in a vertex presentation? a. Ritgen maneuver b. Fundal pressure c. Lithotomy position d. De Lee apparatus

ANS: A The Ritgen maneuver extends the head during the actual birth and protects the perineum. Gentle, steady pressure against the fundus of the uterus facilitates vaginal birth. The lithotomy position has been commonly used in Western cultures, partly because it is convenient for the health care provider. The De Lee apparatus is used to suction fluid from the infant's mouth.

A woman has chosen the calendar method of conception control. Which is the most important action the nurse should perform during the assessment process? a. Obtain a history of the womans menstrual cycle lengths for the past 6 to 12 months. b. Determine the clients weight gain and loss pattern for the previous year. c. Examine skin pigmentation and hair texture for hormonal changes. d. Explore the clients previous experiences with conception control.

ANS: A The calendar method of conception control is based on the number of days in each cycle, counting from the first day of menses. The fertile period is determined after the lengths of menstrual cycles have been accurately recorded for 6 months. Weight gain or loss may be partly related to hormonal fluctuations, but it has no bearing on the use of the calendar method. Integumentary changes may be related to hormonal changes, but they are not indicators for use of the calendar method. Exploring previous experiences with conception control may demonstrate client understanding and compliancy, but these experiences are not the most important aspect to assess for the discussion of the calendar method.

What is the most critical nursing action in caring for the newborn immediately after the birth? a. Keeping the airway clear b. Fostering parent-newborn attachment c. Drying the newborn and wrapping the infant in a blanket d. Administering eye drops and vitamin K

ANS: A The care given immediately after the birth focuses on assessing and stabilizing the newborn. Although fostering parent-newborn attachment is an important task for the nurse, it is not the most critical nursing action in caring for the newborn immediately after birth. The care given immediately after birth focuses on assessing and stabilizing the newborn. The nursing activities are (in order of importance) to maintain a patent airway, to support respiratory effort, and to prevent cold stress by drying the newborn and covering him or her with a warmed blanket or placing the newborn under a radiant warmer. After the newborn has been stabilized, the nurse assesses the newborn's physical condition, weighs and measures the newborn, administers prophylactic eye ointment and a vitamin K injection, affixes an identification bracelet, wraps the newborn in warm blankets, and then gives the newborn to the partner or to the mother of the infant.

In caring for an immediate postpartum client, the nurse notes petechiae and oozing from her intravenous (IV) site. The client would be closely monitored for which clotting disorder? a.DIC b.Amniotic fluid embolism (AFE) c.Hemorrhage d.HELLP syndrome

ANS: A The diagnosis of DIC is made according to clinical findings and laboratory markers. A physical examination reveals unusual bleeding. Petechiae may appear around a blood pressure cuff on the woman's arm. Excessive bleeding may occur from the site of slight trauma such as venipuncture sites. These symptoms are not associated with AFE, nor is AFE a bleeding disorder. Hemorrhage occurs for a variety of reasons in the postpartum client. These symptoms are associated with DIC. Hemorrhage would be a finding associated with DIC and is not a clotting disorder in and of itself. HELLP syndrome is not a clotting disorder, but it may contribute to the clotting disorder DIC.

With regard to the treatment plan for a pregnant woman with gynecologic cancer, which statement about timing or type of treatment is correct? a.The fetus is most at risk during the first trimester. b.The fetus is most at risk during the second trimester. c.The fetus is most at risk during the third trimester. d.Surgery is more risky than chemotherapy in the first trimester.

ANS: A The first trimester is the most vulnerable period for the growing fetus. Women may be faced with making a decision about terminating the pregnancy, depending on the stage and extent of the disease. For advanced disease in the second trimester, alkylating agents, 5-fluorouracil (5-FU), and vincristine are relatively safe for the fetus. For advanced disease in the third trimester, alkylating agents, 5-FU, and vincristine are relatively safe for the fetus. Surgery is less risky than chemotherapy in the first trimester.

Which behavior indicates that a woman is seeking safe passage for herself and her infant? a. She keeps all prenatal appointments. b. She eats for two. c. She slowly drives her car. d. She wears only low-heeled shoes.

ANS: A The goal of prenatal care is to foster a safe birth for the infant and mother. Although properly eating, carefully driving, and using proper body mechanics all are healthy measures that a mother can take, obtaining prenatal care is the optimal method for providing safety for both herself and her baby.

Under which circumstance should the nurse assist the laboring woman into a hands-and-knees position? a. Occiput of the fetus is in a posterior position. b. Fetus is at or above the ischial spines. c. Fetus is in a vertex presentation. d. Membranes have ruptured.

ANS: A The hands-and-knees position is effective in helping to rotate the fetus from a posterior to an anterior position. Many women experience the irresistible urge to push when the fetus is at the level of the ischial spines. In some cases, this urge is felt before the woman is fully dilated. The woman should be instructed not to push until complete cervical dilation has occurred. No one position is correct for childbirth. The two most common positions assumed by women are the sitting and side-lying positions. The woman may be encouraged into a hands-and-knees position if the umbilical cord prolapsed when the membranes ruptured.

The female athlete triad includes which common menstrual disorder? a. Amenorrhea b. Dysmenorrhea c. Menorrhagia d. Metrorrhagia

ANS: A The interrelatedness of disordered eating, amenorrhea, and altered bone mineral density have been described as the female athlete triad. Dysmenorrhea is painful menstruation that begins 2 to 6 months after menarche. Menorrhagia is abnormally profuse or excessive bleeding from the uterus. Metrorrhagia is bleeding between periods and can be caused by progestin injections and implants.

Which condition is likely to be identified by the quadruple marker screen? a. Down syndrome b. Diaphragmatic hernia c. Congenital cardiac abnormality d. Anencephaly

ANS: A The maternal serum level marker of alpha-fetoprotein is used to screen for Down syndrome, trisomy 18, neural tube defects, and other chromosomal anomalies. The quadruple-marker screen will not detect diaphragmatic hernia. Additional testing, such as ultrasonography, is required to diagnose diaphragmatic hernia. Congenital cardiac abnormality will most likely be identified during an ultrasound examination. The quadruple-marker screen will not detect anencephaly.

Although reported in small numbers, toxic shock syndrome (TSS) can occur with the use of a diaphragm. If a client is interested in this form of conception control, then the nurse must instruct the woman on how best to reduce her risk of TSS. Which comment by the nurse would be most helpful in achieving this goal? a. You should always remove your diaphragm 6 to 8 hours after intercourse. Dont use the diaphragm during menses, and watch for danger signs of TSS, including a sudden onset of fever over 38.4 C, hypotension, and a rash. b. You should remove your diaphragm right after intercourse to prevent TSS. c. Its okay to use your diaphragm during your menstrual cycle. Just be sure to wash it thoroughly first to prevent TSS. d. Make sure you dont leave your diaphragm in for longer than 24 hours, or you may get TSS.

ANS: A The nurse should instruct the client on the proper use and removal of the diaphragm and include the danger signs of TSS. The diaphragm must remain against the cervix for 6 to 8 hours to prevent pregnancy, but it should not remain in place longer than 8 hours to avoid the risk of TSS. The diaphragm should not be used during menses.

What is the correct term used to describe the mucous plug that forms in the endocervical canal? a. Operculum b. Leukorrhea c. Funic souffle d. Ballottement

ANS: A The operculum protects against bacterial invasion. Leukorrhea is the mucus that forms the endocervical plug (the operculum). The funic souffle is the sound of blood flowing through the umbilical vessels. Ballottement is a technique for palpating the fetus.

Due to the effects of cyclic ovarian changes in the breast, when is the best time for breast self-examination (BSE)? a. Between 5 and 7 days after menses ceases b. Day 1 of the endometrial cycle c. Midmenstrual cycle d. Any time during a shower or bath

ANS: A The physiologic alterations in breast size and activity reach their minimal level approximately 5 to 7 days after menstruation ceases. Therefore, BSE is best performed during this phase of the menstrual cycle. Day 1 of the endometrial cycle is too early to perform an accurate BSE. After the midmenstrual cycle, breasts are likely to become tender and increase in size, which is not the ideal time to perform BSE. Lying down after a shower or bath with a small towel under the shoulder of the side being examined is appropriate teaching for BSE. A secondary BSE may be performed while in the shower.

A client at 34 weeks of gestation seeks guidance from the nurse regarding personal hygiene. Which information should the nurse provide? a. Tub bathing is permitted even in late pregnancy unless membranes have ruptured. b. The perineum should be wiped from back to front. c. Bubble bath and bath oils are permissible because they add an extra soothing and cleansing action to the bath. d. Expectant mothers should use specially treated soap to cleanse the nipples.

ANS: A The primary danger from taking baths is falling in the tub. The perineum should be wiped from front to back. Bubble baths and bath oils should be avoided because they may irritate the urethra. Soap, alcohol, ointments, and tinctures should not be used to cleanse the nipples because they remove protective oils. Warm water is sufficient.

Which female reproductive organ(s) is(are) responsible for cyclic menstruation? a. Uterus b. Ovaries c. Vaginal vestibule d. Urethra

ANS: A The uterus is responsible for cyclic menstruation and also houses and nourishes the fertilized ovum and the fetus. The ovaries are responsible for ovulation and the production of estrogen. The vaginal vestibule is an external organ that has openings to the urethra and vagina. The urethra is not a reproductive organ, although it is found in the area.

A pregnant woman's amniotic membranes have ruptured. A prolapsed umbilical cord is suspected. What intervention would be the nurse's highest priority? a.Placing the woman in the knee-chest position b.Covering the cord in sterile gauze soaked in saline c.Preparing the woman for a cesarean birth d.Starting oxygen by face mask

ANS: A The woman is assisted into a modified Sims position, Trendelenburg position, or the knee-chest position in which gravity keeps the pressure of the presenting part off the cord. Although covering the cord in sterile gauze soaked saline, preparing the woman for a cesarean, and starting oxygen by face mark are appropriate nursing interventions in the event of a prolapsed cord, the intervention of top priority would be positioning the mother to relieve cord compression.

Part of the nurse's role is assisting with pushing and positioning. Which guidance should the nurse provide to her client in active labor? a.Encourage the woman's cooperation in avoiding the supine position. b.Advise the woman to avoid the semi-Fowler position. c.Encourage the woman to hold her breath and tighten her abdominal muscles to produce a vaginal response. d.Instruct the woman to open her mouth and close her glottis, letting air escape after the push.

ANS: A The woman should maintain a side-lying position. The semi-Fowler position is the recommended side-lying position with a lateral tilt to the uterus. Encouraging the woman to hold her breath and tighten her abdominal muscles is the Valsalva maneuver, which should be avoided. Both the mouth and glottis should be open, allowing air to escape during the push.

Which neurologic condition would require preconception counseling, if at all possible? a. Eclampsia b. Bell palsy c. Epilepsy d. Multiple sclerosis

ANS: C Women with epilepsy should receive preconception counseling, if at all possible. Achieving seizure control before becoming pregnant is a desirable state. Medication should also be carefully reviewed. Eclampsia may sometimes be confused with epilepsy, and Bell palsy is a form of facial paralysis; preconception counseling for either condition is not essential to care. Multiple sclerosis is a patchy demyelination of the spinal cord that does not affect the normal course of pregnancy or birth.

. A laboring woman is reclining in the supine position. What is the most appropriate nursing action at this time? a. Ask her to turn to one side. b. Elevate her feet and legs. c. Take her blood pressure. d. Determine whether fetal tachycardia is present.

ANS: A The woman's supine position may cause the heavy uterus to compress her inferior vena cava, thus reducing blood return to her heart and reducing placental blood flow. Elevating her legs will not relieve the pressure from the inferior vena cava. If the woman is allowed to stay in the supine position and blood flow to the placental is reduced significantly, then fetal tachycardia may occur. The most appropriate nursing action is to prevent this from occurring by turning the woman to her side. Blood pressure readings may be obtained when the client is in the appropriate and safest position.

The client has delivered by urgent caesarean birth for fetal compromise. Umbilical cord gases were obtained for acid-base determination. The pH is 6.9, partial pressure of carbon dioxide (PCO2) is elevated, and the base deficit is 11 mmol/L. What type of acidemia is displayed by the infant? a.Respiratory b.Metabolic c.Mixed d.Turbulent

ANS: A These findings are evidence of respiratory acidemia. Metabolic acidemia is expressed by a pH <7.20, normal carbon dioxide pressure, and a base excess of 12 mmol/L. Mixed acidemia is evidenced by a pH <7.20, elevated carbon dioxide pressure, and a base excess of 12 mmol/L. There is no such finding as turbulent acidemia.

A perinatal nurse is caring for a woman in the immediate postbirth period. Assessment reveals that the client is experiencing profuse bleeding. What is the most likely cause for this bleeding? a.Uterine atony b.Uterine inversion c.Vaginal hematoma d.Vaginal laceration

ANS: A Uterine atony is significant hypotonia of the uterus and is the leading cause of postpartum hemorrhage. Uterine inversion may lead to hemorrhage; however, it is not the most likely source of this client's bleeding. Further, if the woman were experiencing a uterine inversion, it would be evidenced by the presence of a large, red, rounded mass protruding from the introitus. A vaginal hematoma may be associated with hemorrhage. However, the most likely clinical finding for vaginal hematoma is pain, not the presence of profuse bleeding. A vaginal laceration should be suspected if vaginal bleeding continues in the presence of a firm, contracted uterine fundus.

Which development related to the integumentary system is correct? a. Very fine hairs called lanugo appear at 12 weeks of gestation. b. Eyelashes, eyebrows, and scalp hair appear at 28 weeks of gestation. c. Fingernails and toenails develop at 28 weeks of gestation. d. By 32 weeks, scalp hair becomes apparent.

ANS: A Very fine hairs, called lanugo appear first at 12 weeks of gestational age on the fetus eyebrows and upper lip. By 20 weeks of gestation, lanugo covers the entire body. By 20 weeks of gestation the eyelashes, eyebrows, and scalp hair also begin to grow. By 28 weeks of gestation, the scalp hair is longer than these fine hairs, which is thin and may disappear by term. Fingernails and toenails develop from thickened epidermis, beginning during the 10th week. Fingernails reach the fingertips at 32 weeks of gestation, and the toenails reach the toe tips at 36 weeks of gestation.

A nurse caring for a woman in labor should understand that absent or minimal variability is classified as either abnormal or indeterminate. Which condition related to decreased variability is considered benign? a.Periodic fetal sleep state b.Extreme prematurity c.Fetal hypoxemia d.Preexisting neurologic injury

ANS: A When the fetus is temporarily in a sleep state, minimal variability is present. Periodic fetal sleep states usually last no longer than 30 minutes. A woman in labor with extreme prematurity may display a FHR pattern of minimal or absent variability. Abnormal variability may also be related to fetal hypoxemia and metabolic acidemia. Congenital anomalies or a preexisting neurologic injury may also result in absent or minimal variability. Other possible causes might be central nervous system (CNS) depressant medications, narcotics, or general anesthesia.

Conscious relaxation is associated with which method of childbirth preparation? a.Grantly Dick-Read childbirth method b.Lamaze method c.Bradley method d.Psychoprophylactic method

ANS: A With the Grantly Dick-Read method, women are taught to consciously and progressively relax different muscle groups throughout the body until a high degree of skill at relaxation is achieved. The Lamaze method combines controlled muscular relaxation with breathing techniques. The Bradley method advocates natural labor, without any form of anesthesia or analgesia, assisted by a husband-coach and using breathing techniques for labor. The psychoprophylactic method is another name for the Lamaze method.

Which description of the four stages of labor is correct for both the definition and the duration? a. First stage: onset of regular uterine contractions to full dilation; less than 1 hour to 20 hours b. Second stage: full effacement to 4 to 5 cm; visible presenting part; 1 to 2 hours c. Third stage: active pushing to birth; 20 minutes (multiparous woman), 50 minutes (nulliparous woman) d. Fourth stage: delivery of the placenta to recovery; 30 minutes to 1 hour

ANS: A Full dilation may occur in less than 1 hour, but in first-time pregnancies full dilation can take up to 20 hours. The second stage of labor extends from full dilation to birth and takes an average of 20 to 50 minutes, although 2 hours is still considered normal. The third stage of labor extends from birth to the expulsion of the placenta and usually takes a few minutes. The fourth stage begins after the expulsion of the placenta and lasts until homeostasis is reestablished (approximately 2 hours).

Which presentation is accurately described in terms of both the resenting part and the frequency of occurrence? a. Cephalic: occiput, at least 96% b. Breech: sacrum, 10% to 15% c. Shoulder: scapula, 10% to 15% d. Cephalic: cranial, 80% to 85%

ANS: A In cephalic presentations (head first), the presenting part is the occiput; this presentation occurs in 96% of births. In a breech birth, the sacrum emerges first; this presentation occurs in approximately 3% of births. In shoulder presentations, the scapula emerges first; this presentation occurs in only 1% of births. In a cephalic presentation, the part of the head or cranium that emerges first is the occiput; cephalic presentations occur in 96% of births.

Which nursing intervention is necessary before a first-trimester transabdominal ultrasound? a. Place the woman on nothing by mouth (nil per os [NPO]) for 12 hours. b. Instruct the woman to drink 1 to 2 quarts of water. c. Administer an enema. d. Perform an abdominal preparation.

ANS: B When the uterus is still in the pelvis, visualization may be difficult. Performing a first-trimester transabdominal ultrasound requires the woman to have a full bladder, which will elevate the uterus upward and provide a better visualization of the fetus; therefore, being NPO is not appropriate. Neither an enema nor an abdominal preparation is necessary for this procedure.

Which stage of labor varies the most in length? a. First b. Second c. Third d. Fourth

ANS: A The first stage of labor is considered to last from the onset of regular uterine contractions to the full dilation of the cervix. The first stage is significantly longer than the second and third stages combined. In a first-time pregnancy, the first stage of labor can take up to 20 hours. The second stage of labor lasts from the time the cervix is fully dilated to the birth of the fetus. The average length is 20 minutes for a multiparous woman and 50 minutes for a nulliparous woman. The third stage of labor lasts from the birth of the fetus until the placenta is delivered. This stage may be as short as 3 minutes or as long as 1 hour. The fourth stage of labor, recovery, lasts approximately 2 hours after the delivery of the placenta.

Which signs and symptoms should a woman immediately report to her health care provider? (Select all that apply.) a. Vaginal bleeding b. Rupture of membranes c. Heartburn accompanied by severe headache d. Decreased libido e. Urinary frequency

ANS: A, B, C Vaginal bleeding, rupture of membranes, and severe headaches are signs of potential complications in pregnancy. Clients should be advised to report these signs to their health care provider. Decreased libido and urinary frequency are common discomforts of pregnancy that do not require immediate health care interventions.

Which congenital anomalies can occur as a result of the use of antiepileptic drugs (AEDs) in pregnancy? (Select all that apply.) a. Cleft lip b. Congenital heart disease c. Neural tube defects d. Gastroschisis e. Diaphragmatic hernia

ANS: A, B, C Congenital anomalies that can occur with AEDs include cleft lip or palate, congenital heart disease, urogenital defects, and neural tube defects. Carbamazepine and valproate should be avoided if all possible; they may cause neural tube defects. Congenital anomalies of gastroschisis and diaphragmatic hernia are not associated with the use of AEDs.

In assessing the immediate condition of the newborn after birth, a sample of cord blood may be a useful adjunct to the Apgar score. Cord blood is then tested for pH, carbon dioxide, oxygen, and base deficit or excess. Which clinical situation warrants this additional testing? (Select all that apply.) a.Low 5-minute Apgar score b.Intrauterine growth restriction (IUGR) c.Maternal thyroid disease d.Intrapartum fever e.Vacuum extraction

ANS: A, B, C, D The American College of Obstetricians and Gynecologists (ACOG) suggests obtaining cord blood values in all of these clinical situations except for vacuum extractions deliveries. Cord blood gases should also be performed for multifetal pregnancies or abnormal FHR tracings. Samples can be drawn from both the umbilical artery and the umbilical vein. Results may indicate that fetal compromise has occurred.

Most women with uncomplicated pregnancies can use the nurse as their primary source for nutritional information. However, the nurse or midwife may need to refer a client to a registered dietitian for in-depth nutritional counseling. Which conditions would require such a consultation? (Select all that apply.) a. Preexisting or gestational illness such as diabetes b. Ethnic or cultural food patterns c. Obesity d. Vegetarian diets e. Multifetal pregnancy

ANS: A, B, C, D The nurse should be especially aware that conditions such as diabetes can require in-depth dietary planning and evaluation. To prevent issues with hypoglycemia and hyperglycemia, as well as an increased risk for perinatal morbidity and mortality, the client with a preexisting or gestational illness would benefit from a referral to a dietitian. Consultation with a dietitian may ensure that cultural food beliefs are congruent with modern knowledge of fetal development and that adjustments can be made to ensure that all nutritional needs are met. The obese pregnant client may be under the misapprehension that, because of her excess weight, little or no weight gain is necessary. According to the Institute of Medicine, a client with a BMI in the obese range should gain at least 7 kg to ensure a healthy outcome. This client may require in-depth counseling on the optimal food choices. The vegetarian client needs to have her dietary intake carefully assessed to ensure that the optimal combination of amino acids and protein intake is achieved. Very strict vegetarians (vegans) who consume only plant products may also require vitamin B and mineral supplementation. A multifetal pregnancy can be managed by increasing the number of servings of complex carbohydrates and proteins.

Women who are obese are at risk for several complications during pregnancy and birth. Which of these would the nurse anticipate with an obese client? (Select all that apply.) a.Thromboembolism b.Cesarean birth c.Wound infection d.Breech presentation e.Hypertension

ANS: A, B, C, E A breech presentation is not a complication of pregnancy or birth for the client who is obese. Venous thromboembolism is a known risk for obese women. Therefore, the use of thromboembolism-deterrent (TED) hose and sequential compression devices may help decrease the chance for clot formation. Women should also be encouraged to ambulate as soon as possible. In addition to having an increased risk for complications with a cesarean birth, in general, obese women are also more likely to require an emergency cesarean birth. Many obese women have a pannus (i.e., large roll of abdominal fat) that overlies a lower transverse incision made just above the pubic area. The pannus causes the area to remain moist, which encourages the development of infection. Obese women are more likely to begin pregnancy with comorbidities such as hypertension and type 2 diabetes.

Congenital disorders refer to those conditions that are present at birth. These disorders may be inherited and caused by environmental factors or maternal malnutrition. Toxic exposures have the greatest effect on development between 15 and 60 days of gestation. For the nurse to be able to conduct a complete assessment of the newly pregnant client, he or she should be knowledgeable regarding known human teratogens. Which substances might be considered a teratogen? (Select all that apply.) a. Cytomegalovirus (CMV) b. Ionizing radiation c. Hypothermia d. Carbamazepine e. Lead

ANS: A, B, D, E Exposure to radiation and a number of infections may result in profound congenital deformities. These include but are not limited to varicella, rubella, syphilis, parvovirus, CMV, and toxoplasmosis. Certain maternal conditions such as diabetes and phenylketonuria (PKU) may also affect organs and other parts of the embryo during this developmental period. Drugs such as antiseizure medications (e.g., carbamazepine) and some antibiotics, as well as chemicals including lead, mercury, tobacco, and alcohol, may also result in structural and functional abnormalities

While developing an intrapartum care plan for the client in early labor, which psychosocial factors would the nurse recognize upon the client's pain experience? (Select all that apply.) a.Culture b.Anxiety and fear c.Previous experiences with pain d.Intervention of caregivers e.Support systems

ANS: A, B, C, E Culture: A woman's sociocultural roots influence how she perceives, interprets, and responds to pain during childbirth. Some cultures encourage loud and vigorous expressions of pain, whereas others value self-control. The nurse should avoid praising some behaviors (stoicism) while belittling others (noisy expression). Anxiety and fear: Extreme anxiety and fear magnify the sensitivity to pain and impair a woman's ability to tolerate it. Anxiety and fear increase muscle tension in the pelvic area, which counters the expulsive forces of uterine contractions and pushing efforts. Previous experiences with pain: Fear and withdrawal are a natural response to pain during labor. Learning about these normal sensations ahead of time helps a woman suppress her natural reactions of fear regarding the impending birth. If a woman previously had a long and difficult labor, she is likely to be anxious. She may also have learned ways to cope and may use these skills to adapt to the present labor experience. Support systems: An anxious partner is less able to provide help and support to a woman during labor. A woman's family and friends can be an important source of support if they convey realistic and positive information about labor and delivery. Although the intervention of caregivers may be necessary for the well-being of the woman and her fetus, some interventions add discomfort to the natural pain of labor (i.e., fetal monitor straps, IV lines).

IUGR is associated with which pregnancy-related risk factors? (Select all that apply.) a. Poor nutrition b. Maternal collagen disease c. Gestational hypertension d. Premature rupture of membranes e. Smoking

ANS: A, B, C, E Poor nutrition, maternal collagen disease, gestational hypertension, and smoking are risk factors associated with the occurrence of IUGR. Premature rupture of membranes is associated with preterm labor, not IUGR.

Many pregnant teenagers wait until the second or third trimester to seek prenatal care. What should the nurse recognize as reasons for this delay? (Select all that apply.) a. Lack of realization that they are pregnant b. Uncertainty as to where to go for care c. Continuing to deny the pregnancy d. Desire to gain control over their situation e. Wanting to hide the pregnancy as long as possible

ANS: A, B, C, E These reasons are all valid explanations why teens delay seeking prenatal care. An adolescent often has little to no understanding of the increased physiologic needs that a pregnancy places on her body. Once care is sought, it is often sporadic, and many appointments are usually missed. The nurse should formulate a diagnosis that assists the pregnant teen to receive adequate prenatal care. Planning for her pregnancy and impending birth actually provides some sense of control for the teen and increases her feelings of competency. Receiving praise from the nurse when she attends her prenatal appointments will reinforce the teens positive self-image.

The nurse is responsible for providing health teaching regarding the side effects of COCs. These side effects are attributed to estrogen, progesterone, or both. Which side effects are related to the use of COCs? (Select all that apply.) a. Gallbladder disease b. Myocardial infarction and stroke c. Hypotension d. Breast tenderness and fluid retention e. Dry skin and scalp

ANS: A, B, D Serious side effects include stroke, myocardial infarction, hypertension, gallbladder disease, and liver tumors. More common side effects include nausea, breast tenderness, fluid retention, increased appetite, oily skin and scalp, and chloasma.

A woman has requested an epidural block for her pain. She is 5 cm dilated and 100% effaced. The baby is in a vertex position and is engaged. The nurse increases the woman's IV fluid for a preprocedural bolus. Before the initiation of the epidural, the woman should be informed regarding the disadvantages of an epidural block. Which concerns should the nurse share with this client? (Select all that apply.) a.Ability to move freely is limited. b.Orthostatic hypotension and dizziness may occur. c.Gastric emptying is not delayed. d.Higher body temperature may occur. e.Blood loss is not excessive.

ANS: A, B, D The woman's ability to move freely and to maintain control of her labor is limited, related to the use of numerous medical interventions (IV lines and electronic fetal monitoring [EFM]). Significant disadvantages of an epidural block include the occurrence of orthostatic hypotension, dizziness, sedation, and leg weakness. Women who receive an epidural block have a higher body temperature (38° C or higher), especially when labor lasts longer than 12 hours, and may result in an unnecessary neonatal workup for sepsis. An advantage of an epidural block is that blood loss is not excessive. Other advantages include the following: the woman remains alert and able to participate, good relaxation is achieved, airway reflexes remain intact, and only partial motor paralysis develops.

The reported incidence of ectopic pregnancy has steadily risen over the past 2 decades. Causes include the increase in sexually transmitted infections (STIs) accompanied by tubal infection and damage. The popularity of contraceptive devices such as the IUD has also increased the risk for ectopic pregnancy. The nurse suspects that a client has early signs of ectopic pregnancy. The nurse should be observing the client for which signs or symptoms? (Select all that apply.) a. Pelvic pain b.Abdominal pain c.Unanticipated heavy bleeding d.Vaginal spotting or light bleeding e.Missed period

ANS: A, B, D, E A missed period or spotting can be easily mistaken by the client as an early sign of pregnancy. More subtle signs depend on exactly where the implantation occurs. The nurse must be thorough in her assessment because pain is not a normal symptom of early pregnancy. As the fallopian tube tears open and the embryo is expelled, the client often exhibits severe pain accompanied by intraabdominal hemorrhage, which may progress to hypovolemic shock with minimal or even no external bleeding. In approximately one half of women, shoulder and neck pain results from irritation of the diaphragm from the hemorrhage.

Screening questions for alcohol and drug abuse should be included in the overall assessment during the first prenatal visit for all women. The 4 Ps Plus is a screening tool specifically designed to identify the need for a more in-depth assessment. Which are the correct components of the 4 Ps Plus? (Select all that apply.) a. Parents b. Partner c. Present d. Past e. Pregnancy

ANS: A, B, D, E The nurse who is screening the client using the 4 Ps Plus would use the following format: Parents: Did either of your parents have a problem with alcohol or drugs? Partner: Does your partner have a problem with alcohol or drugs? Past: Have you ever had any beer, wine, or liquor? Pregnancy: In the month before you knew you were pregnant, how many cigarettes did you smoke? How much beer, wine, or liquor did you drink? Present: Is not a component of the 4 Ps Plus.

Diabetes refers to a group of metabolic diseases characterized by hyperglycemia resulting from defects in insulin action, insulin secretion, or both. Over time, diabetes causes significant changes in the microvascular and macrovascular circulations. What do these complications include? (Select all that apply.) a. Atherosclerosis b. Retinopathy c. Intrauterine fetal death (IUFD) d. Nephropathy e. Neuropathy f. Autonomic neuropathy

ANS: A, B, D, E These structural changes will most likely affect a variety of systems, including the heart, eyes, kidneys, and nerves. IUFD (stillbirth) remains a major complication of diabetes in pregnancy; however, this is a fetal complication.

Which factors influence cervical dilation? (Select all that apply.) a. Strong uterine contractions b. Force of the presenting fetal part against the cervix c. Size of the woman d. Pressure applied by the amniotic sac e. Scarring of the cervix

ANS: A, B, D, E Dilation of the cervix occurs by the drawing upward of the musculofibrous components of the cervix, which is caused by strong uterine contractions. Pressure exerted by the amniotic fluid while the membranes are intact or by the force applied by the presenting part can also promote cervical dilation. Scarring of the cervix as a result of a previous infection or surgery may slow cervical dilation. Pelvic size or the size of the woman does not affect cervical dilation.

One of the most important components of the physical assessment of the pregnant client is the determination of BP. Consistency in measurement techniques must be maintained to ensure that the nuances in the variations of the BP readings are not the result of provider error. Which techniques are important in obtaining accurate BP readings? (Select all that apply.) a. The client should be seated. b. The clients arm should be placed at the level of the heart. c. An electronic BP device should be used. d. The cuff should cover a minimum of 60% of the upper arm. e. The same arm should be used for every reading.

ANS: A, B, E BP readings are easily affected by maternal position. Ideally, the client should be seated. An alternative position is left lateral recumbent with the arm at the level of the heart. The arm should always be held in a horizontal position at approximately the level of the heart. The same arm should be used at every visit. The manual sphygmomanometer is the most accurate device. If manual and electronic devices are used in the care setting, then the nurse must use caution when interpreting the readings. A proper size cuff should cover at least 80% of the upper arm or be approximately 1.5 times the length of the upper arm.

A woman has just moved to the United States from Mexico. She is 3 months pregnant and has arrived for her first prenatal visit. During her assessment interview, the nurse learns that the client has not had any immunizations. Which immunizations should she receive at this point in her pregnancy? (Select all that apply.) a. Tetanus b. Diphtheria c. Chickenpox d. Rubella e. Hepatitis B

ANS: A, B, E Vaccines consisting of killed viruses may be used. Those that may be administered during pregnancy include tetanus, diphtheria, recombinant hepatitis B, and rabies vaccines. Immunizations with live or attenuated viruses are contraindicated during pregnancy because of their potential teratogenicity. Live-virus vaccines include those for measles (rubeola and rubella), chickenpox, and mumps.

Which suggestions are appropriate for a client who complains of hot flashes? (Select all that apply.) a. Avoid caffeine. b. Drink a glass of wine to relax. c. Wear layered clothing. d. Drink ice water. e. Drink warm beverages for their calming effect.

ANS: A, C Layered clothing allows the client to remove layers if a hot flash occurs. Ice water may help alleviate the hot flashes. Slow, deep breathing is also beneficial. Avoid triggers such as exercising on hot days, spicy foods, hot beverages, and alcohol.

The diagnosis of pregnancy is based on which positive signs of pregnancy? (Select all that apply.) a. Identification of fetal heartbeat b. Palpation of fetal outline c. Visualization of the fetus d. Verification of fetal movement e. Positive hCG test

ANS: A, C, D Identification of a fetal heartbeat, the visualization of the fetus, and verification of fetal movement are all positive, objective signs of pregnancy. Palpation of fetal outline and positive hCG test are probable signs of pregnancy. A tumor also can be palpated. Medication and tumors may lead to false-positive results on pregnancy tests.

Which changes take place in the woman's reproductive system, days or even weeks before the commencement of labor? (Select all that apply.) a. Lightening b. Exhaustion c. Bloody show d. Rupture of membranes e. Decreased fetal movement

ANS: A, C, D Signs that precede labor may include lightening, urinary frequency, backache, weight loss, surge of energy, bloody show, and rupture of membranes. Many women experience a burst of energy before labor. A decrease in fetal movement is an ominous sign that does not always correlate with labor.

In caring for a pregnant woman with sickle cell anemia, the nurse must be aware of the signs and symptoms of a sickle cell crisis. What do these include? (Select all that apply.) a. Fever b. Endometritis c. Abdominal pain d. Joint pain e. Urinary tract infection (UTI)

ANS: A, C, D Women with sickle cell anemia have recurrent attacks (crises) of fever and pain, most often in the abdomen, joints, and extremities. These attacks are attributed to vascular occlusion when red blood cells (RBCs) assume the characteristic sickled shape. Crises are usually triggered by dehydration, hypoxia, or acidosis. Women with the sickle cell trait are usually at a greater risk for postpartum endometritis (uterine wall infection); however, this development is not likely to occur during the pregnancy and is not a sign for the disorder. Although women with sickle cell anemia are at an increased risk for UTIs, these infections are not an indication of a sickle cell crisis.

The nurse is reviewing the educational packet provided to a client about tubal ligation. Which information regarding this procedure is important for the nurse to share? (Select all that apply.) a. It is highly unlikely that you will become pregnant after the procedure. b. Tubal ligation is an effective form of 100% permanent sterilization. You wont be able to get pregnant. c. Sterilization offers some form of protection against STIs. d. Sterilization offers no protection against STIs. e. Your menstrual cycle will greatly increase after your sterilization.

ANS: A, D A woman is unlikely to become pregnant after tubal ligation. However, sterilization offers no protection against STIs and is not 100% effective. Typically, the menstrual cycle remains the same after a tubal ligation.

Lacerations of the cervix, vagina, or perineum are also causes of PPH. Which factors influence the causes and incidence of obstetric lacerations of the lower genital tract? (Select all that apply.) a.Operative and precipitate births b.Adherent retained placenta c.Abnormal presentation of the fetus d.Congenital abnormalities of the maternal soft tissue e.Previous scarring from infection

ANS: A, C, D, E Abnormal adherence of the placenta occurs for unknown reasons. Attempts to remove the placenta in the usual manner can be unsuccessful, and lacerations or a perforation of the uterine wall may result. However, attempts to remove the placenta do not influence lower genital tract lacerations. Lacerations of the perineum are the most common of all lower genital tract injuries and often occur with both precipitate and operative births and are classified as first-, second-, third-, and fourth-degree lacerations. An abnormal presentation or position of the fetus, the relative size of the presenting part, and the birth canal may contribute to lacerations of the lower genital tract. Congenital abnormalities, previous scarring from infection or injury, and a contracted pelvis may also influence injury to the lower genital tract, followed by hemorrhage.

Chemotherapy with multiple drug agents is used in the treatment of recurrent and advanced breast cancer with positive results. Which side effects would the nurse anticipate for the client once treatment has begun? (Select all that apply.) a.Hair loss b.Severe constipation c.Anemia d.Leukopenia e.Thrombocytopenia

ANS: A, C, D, E Because chemotherapeutic agents rapidly kill reproducing cells, treatment also affects normal cells that frequently reproduce. The side effects that the nurse would anticipate and on which the nurse will provide education include partial or full hair loss, gastrointestinal effects (e.g., nausea, vomiting, anorexia, mucositis), leukopenia, neutropenia, thrombocytopenia, and anemia.

Which medications can be taken by postmenopausal women to treat and/or prevent osteoporosis? (Select all that apply.) a. Calcium b. NSAIDs c. Fosamax d. Actonel e. Calcitonin

ANS: A, C, D, E Calcium, Evista, Fosamax, Actonel, and Calcitonin can be used by postmenopausal women to treat or prevent osteoporosis. Parathyroid hormone and estrogen may also be of value. NSAIDs may provide pain relief; however, these medications neither prevent nor treat osteoporosis.

Approximately 10% to 15% of all clinically recognized pregnancies end in miscarriage. What are possible causes of early miscarriage? (Select all that apply.) a.Chromosomal abnormalities b.Infections c.Endocrine imbalance d.Systemic disorders e.Varicella

ANS: A, C, D, E Infections are not a common cause of early miscarriage. At least 50% of pregnancy losses result from chromosomal abnormalities. Endocrine imbalances such as hypothyroidism or diabetes are also possible causes for early pregnancy loss. Other systemic disorders that may contribute to pregnancy loss include lupus and genetic conditions. Although infections are not a common cause of early miscarriage, varicella infection in the first trimester has been associated with pregnancy loss.

A lupus flare-up during pregnancy or early postpartum occurs in 15% to 60% of women with this disorder. Which conditions associated with systemic lupus erythematosus (SLE) are maternal risks? (Select all that apply.) a. Miscarriage b. Intrauterine growth restriction (IUGR) c. Nephritis d. Preeclampsia e. Cesarean birth

ANS: A, C, D, E Maternal risks associated with SLE include miscarriage, nephritis, preeclampsia, and cesarean birth. IUGR is a fetal risk related to SLE. Other fetal risks include stillbirth and prematurity.

Which adverse prenatal outcomes are associated with the HELLP syndrome? (Select all that apply.) a. Placental abruption b. Placenta previa c. Renal failure d. Cirrhosis e. Maternal and fetal death

ANS: A, C, E The HELLP syndrome is associated with an increased risk for adverse perinatal outcomes, including placental abruption, acute renal failure, subcapsular hepatic hematoma, hepatic rupture, recurrent preeclampsia, preterm birth, and fetal and maternal death. The HELLP syndrome is associated with an increased risk for placental abruption, not placenta previa. It is also associated with an increased risk for hepatic hematoma, not cirrhosis.

The class of drugs known as opioid analgesics (butorphanol, nalbuphine) is not suitable for administration to women with known opioid dependence. The antagonistic activity could precipitate withdrawal symptoms (abstinence syndrome) in both mothers and newborns. Which signs would indicate opioid or narcotic withdrawal in the mother? (Select all that apply.) a.Yawning, runny nose b.Increase in appetite c.Chills or hot flashes d.Constipation e.Irritability, restlessness

ANS: A, C, E The woman experiencing maternal opioid withdrawal syndrome will exhibit yawning, runny nose, sneezing, anorexia, chills or hot flashes, vomiting, diarrhea, abdominal pain, irritability, restlessness, muscle spasms, weakness, and drowsiness. Assessing both the mother and the newborn and planning the care accordingly are important steps for the nurse to take

A client who has undergone a D&C for early pregnancy loss is likely to be discharged the same day. The nurse must ensure that her vital signs are stable, that bleeding has been controlled, and that the woman has adequately recovered from the administration of anesthesia. To promote an optimal recovery, what information should discharge teaching include? (Select all that apply.) a.Iron supplementation b.Resumption of intercourse at 6 weeks postprocedure c.Referral to a support group, if necessary d.Expectation of heavy bleeding for at least 2 weeks e.Emphasizing the need for rest

ANS: A, C, E The woman should be advised to consume a diet high in iron and protein. For many women, iron supplementation also is necessary. The nurse should acknowledge that the client has experienced a loss, however early. She can be taught to expect mood swings and possibly depression. Referral to a support group, clergy, or professional counseling may be necessary. Discharge teaching should emphasize the need for rest. Nothing should be placed in the vagina for 2 weeks after the procedure, including tampons and vaginal intercourse. The purpose of this recommendation is to prevent infection. Should infection occur, antibiotics may be prescribed. The client should expect a scant, dark discharge for 1 to 2 weeks. Should heavy, profuse, or bright bleeding occur, she should be instructed to contact her health care provider.

The client has undergone hysteroscopic uterine ablation to destroy a number of smaller fibroids. The nurse is preparing to provide discharge instructions. Which information is a priority for the patient and should be included in the teaching plan? (Select all that apply.) a.Analgesics and nonsteroidal antiinflammatory drugs can be used for pain control. b.Vaginal discharge is to be expected for 5 to 7 days. c.Sexual activity can be resumed after 48 hours. d.Next menstrual period will be irregular. e.Provider should be notified if heavy bleeding occurs.

ANS: A, D, E Before discharging the client, the following information should be given: analgesics can be used for pain relief as needed, normal activities can be resumed within several days, vaginal discharge is to be expected for 4 to 6 weeks, and the use of tampons and sexual activity should be avoided for 2 weeks. The next menstrual period will be irregular, and the provider should be notified for heavy bleeding or signs of infection.

Researchers have found a number of common risk factors that increase a woman's chance of developing a breast malignancy. It is essential for the nurse who provides care to women of any age to be aware of which risk factors? (Select all that apply.) a.Family history b.Late menarche c.Early menopause d.Race e.Nulliparity or first pregnancy after age 40 years

ANS: A, D, E Family history, race, and nulliparity or the first pregnancy after age 40 years are known risk factors for the development of breast cancer. Others risk factors include age, personal history of cancer, high socioeconomic status, sedentary lifestyle, hormone replacement therapy, recent use of oral contraceptives, never having breastfed a child, and drinking more than one alcoholic beverage per day. Early menarche and late menopause are not risk factors for breast malignancy.

A woman who is 39 weeks pregnant expresses fear about her impending labor and how she will manage. What is the nurse's ideal response? a. "Don't worry about it. You'll do fine." b. "It's normal to be anxious about labor. Let's discuss what makes you afraid." c. "Labor is scary to think about, but the actual experience isn't." d. "You can have an epidural. You won't feel anything."

ANS: B "It's normal to be anxious about labor. Let's discuss what makes you afraid" is a statement that allows the woman to share her concerns with the nurse and is a therapeutic communication tool. "Don't worry about it. You'll do fine" negates the woman's fears and is not therapeutic. "Labor is scary to think about, but the actual experience isn't" negates the woman's fears and offers a false sense of security. To suggest that every woman can have an epidural is untrue. A number of criteria must be met before an epidural is considered. Furthermore, many women still experience the feeling of pressure with an epidural.

A client states that she plans to breastfeed her newborn infant. What guidance would be useful for this new mother? a. The mothers intake of vitamin C, zinc, and protein can now be lower than during pregnancy. b. Caffeine consumed by the mother accumulates in the infant, who may be unusually active and wakeful. c. Critical iron and folic acid levels must be maintained. d. Lactating women can go back to their prepregnant caloric intake.

ANS: B A lactating woman needs to avoid consuming too much caffeine. Vitamin C, zinc, and protein levels need to be moderately higher during lactation than during pregnancy. The recommendations for iron and folic acid are lower during lactation. Lactating women should consume approximately 500 kcal more than their prepregnancy intake, at least 1800 kcal daily overall.

During the first trimester, which of the following changes regarding her sexual drive should a client be taught to expect? a. Increased sexual drive, because of enlarging breasts b. Decreased sexual drive, because of nausea and fatigue c. No change in her sexual drive d. Increased sexual drive, because of increased levels of female hormones

ANS: B A pregnant woman usually experiences a decrease, not an increase, in libido during the first trimester. Maternal physiologic changes, such as breast enlargement, nausea, fatigue, abdominal changes, perineal enlargement, leukorrhea, pelvic vasocongestion, and orgasmic responses, may affect sexuality and sexual expression. Libido may be depressed in the first trimester but often increases during the second and third trimesters. During pregnancy, the breasts may become enlarged and tender, which tends to interfere with coitus, thereby decreasing the desire to engage in sexual activity.

Assessment of a womans nutritional status includes a diet history, medication regimen, physical examination, and relevant laboratory tests. Which finding might require consultation to a higher level of care? a. Oral contraceptive use may interfere with the absorption of iron. b. Illnesses that have created nutritional deficits, such as PKU, may require nutritional care before conception. c. The womans socioeconomic status and educational level are not relevant to her examination; they are the province of the social worker. d. Testing for diabetes is the only nutrition-related laboratory test most pregnant women need.

ANS: B A registered dietitian can help with therapeutic diets. Oral contraceptive use may interfere with the absorption of folic acid. Iron deficiency can appear if placement of an intrauterine device (IUD) results in blood loss. A womans finances can affect her access to good nutrition; her education (or lack thereof) can influence the nurses teaching decisions. The nutrition-related laboratory test that pregnant women usually need is a screen for anemia.

Which action is the highest priority for the nurse when educating a pregnant adolescent? a. Emphasize the need to eliminate common teenage snack foods because they are high in fat and sodium. b. Determine the weight gain needed to meet adolescent growth, and add 35 pounds. c. Suggest that she not eat at fast-food restaurants to avoid foods of poor nutritional value. d. Realize that most adolescents are unwilling to make dietary changes during pregnancy.

ANS: B Adolescents should gain in the upper range of the recommended weight gain. They also need to gain weight that would be expected for their own normal growth. Changes in the diet should be kept at a minimum. Snack foods can be included in moderation, and other foods can be added to make up for lost nutrients. Eliminating fast foods would make the adolescent appear different to her peers. The client should be taught to choose foods that add needed nutrients. Adolescents are willing to make changes; however, they still have the need to be similar to their peers.

A woman who is gravida 3 para 2 arrives on the intrapartum unit. What is the most important nursing assessment at this time? a. Contraction pattern, amount of discomfort, and pregnancy history b. FHR, maternal vital signs, and the woman's nearness to birth c. Identification of ruptured membranes, woman's gravida and para, and her support person d. Last food intake, when labor began, and cultural practices the couple desires

ANS: B All options describe relevant intrapartum nursing assessments; however, this focused assessment has a priority. If the maternal and fetal conditions are normal and birth is not imminent, then other assessments can be performed in an unhurried manner; these include: gravida, para, support person, pregnancy history, pain assessment, last food intake, and cultural practices.

Under which circumstance would it be unnecessary for the nurse to perform a vaginal examination? a. Admission to the hospital at the start of labor b. When accelerations of the FHR are noted c. On maternal perception of perineal pressure or the urge to bear down d. When membranes rupture

ANS: B An accelerated FHR is a positive sign; therefore, a vaginal examination would not be necessary. A vaginal examination should be performed when the woman is admitted to the hospital, when she perceives perineal pressure or the urge to bear down, when her membranes rupture, when a significant change in her uterine activity has occurred, or when variable decelerations of the FHR are noted.

A client with a history of bipolar disorder is called by the postpartum support nurse for follow-up. Which symptoms would reassure the nurse that the client is not experiencing a manic episode? a. Psychomotor agitation and lack of sleep b. Increased appetite and lack of interest in activities c. Hyperactivity and distractibility d. Pressured speech and grandiosity

ANS: B An increased appetite and a lack of interest would reassure the nurse that the client is not experiencing an episode of mania. Clinical manifestations of a manic episode include at least three of the following: grandiosity, decreased need for sleep, pressured speech, flight of ideas, distractibility, psychomotor agitation, and excessive involvement in pleasurable activities. The pregnant woman exhibiting symptoms of a manic episode will likely have a decreased interest in eating and an increased level of interest in pleasurable activities without regard for negative consequences. Psychomotor agitation and a lack of sleep, hyperactivity and distractibility, and pressured speech and grandiosity are all clinical manifestations of a manic episode.

A 31-year-old woman believes that she may be pregnant. She took an over-the-counter (OTC) pregnancy test 1 week ago after missing her period; the test was positive. During her assessment interview, the nurse inquires about the womans last menstrual period and asks whether she is taking any medications. The client states that she takes medicine for epilepsy. She has been under considerable stress lately at work and has not been sleeping well. Her physical examination does not indicate that she is pregnant. She has an ultrasound scan, which confirms that she is not pregnant. What is the most likely cause of the false-positive pregnancy test result? a. The pregnancy test was taken too early. b. Anticonvulsant medications may cause the false-positive test result. c. The woman has a fibroid tumor. d. She has been under considerable stress and has a hormone imbalance.

ANS: B Anticonvulsants may cause false-positive pregnancy test results. OTC pregnancy tests use enzyme-linked immunosorbent assay (ELISA) technology, which can yield positive results as soon as 4 days after implantation. Implantation occurs 6 to 10 days after conception. If the woman were pregnant, then she would be into her third week at this point (having missed her period 1 week ago). Fibroid tumors do not produce hormones and have no bearing on human chorionic gonadotropin (hCG) pregnancy tests. Although stress may interrupt normal hormone cycles (menstrual cycles), it does not affect hCG levels or produce positive pregnancy test results.

Anxiety is commonly associated with pain during labor. Which statement regarding anxiety is correct? a.Even mild anxiety must be treated. b.Severe anxiety increases tension, increases pain, and then, in turn, increases fear and anxiety, and so on. c.Anxiety may increase the perception of pain, but it does not affect the mechanism of labor. d.Women who have had a painful labor will have learned from the experience and have less anxiety the second time because of increased familiarity.

ANS: B Anxiety and pain reinforce each other in a negative cycle that will slow the progress of labor. Mild anxiety is normal for a woman in labor and likely needs no special treatment other than the standard reassurances. Anxiety increases muscle tension and ultimately can sufficiently build to slow the progress of labor. Unfortunately, an anxious, painful first labor is likely to carry over, through expectations and memories, into an anxious and painful experience in the second pregnancy.

Many clients are concerned about the increased levels of mercury in fish and may be reluctant to include this source of nutrients in their diet. What is the best advice for the nurse to provide? a. Canned white tuna is a preferred choice. b. Shark, swordfish, and mackerel should be avoided. c. Fish caught in local waterways is the safest. d. Salmon and shrimp contain high levels of mercury.

ANS: B As a precaution, the pregnant client should avoid eating shark, swordfish, and mackerel, as well as the less common tilefish. High levels of mercury can harm the developing nervous system of the fetus. Assisting the client in understanding the differences between numerous sources of mercury is essential for the nurse. A pregnant client may eat as much as 12 ounces a week of canned light tuna; however, canned white, albacore, or tuna steaks contain higher levels of mercury and should be limited to no more than 6 ounces per week. Pregnant women and mothers of young children should check with local advisories about the safety of fish caught by families and friends in nearby bodies of water. If no information is available, then these fish sources should be avoided, limited to less than 6 ounces per week, or the only fish consumed that week. Commercially caught fish that is low in mercury includes salmon, shrimp, pollock, or catfish. The pregnant client may eat up to 12 ounces of commercially caught fish per week. Additional information on levels of mercury in commercially caught fish is available at www.cfsan.fda.gov.

A number of methods can be used for inducing labor. Which cervical ripening method falls under the category of mechanical or physical? a.Prostaglandins are used to soften and thin the cervix. b.Labor can sometimes be induced with balloon catheters or laminaria tents. c.Oxytocin is less expensive and more effective than prostaglandins but creates greater health risks. d.Amniotomy can be used to make the cervix more favorable for labor.

ANS: B Balloon catheters or laminaria tents are mechanical means of ripening the cervix. Ripening the cervix, making it softer and thinner, increases the success rate of induced labor. Prostaglandin E1 is less expensive and more effective than oxytocin but carries a greater risk. Amniotomy is the artificial rupture of membranes, which is used to induce labor only when the cervix is already ripe.

What three measures should the nurse implement to provide intrauterine resuscitation? a.Call the provider, reposition the mother, and perform a vaginal examination. b.Turn the client onto her side, provide oxygen (O2) via face mask, and increase intravenous (IV) fluids. c.Administer O2 to the mother, increase IV fluids, and notify the health care provider. d.Perform a vaginal examination, reposition the mother, and provide O2 via face mask.

ANS: B Basic interventions for the management of any abnormal FHR pattern include administering O2 via a nonrebreather face mask at a rate of 8 to 10 L/min, assisting the woman onto a side-lying (lateral) position, and increasing blood volume by increasing the rate of the primary IV infusion. The purpose of these interventions is to improve uterine blood flow and intervillous space blood flow and to increase maternal oxygenation and cardiac output. The term intrauterine resuscitation is sometimes used to refer to these interventions. If these interventions do not quickly resolve the abnormal FHR issue, then the primary provider should be immediately notified.

Developing a realistic birth plan with the pregnant woman regarding her care is important for the nurse. How would the nurse explain the major advantage of nonpharmacologic pain management? a.Greater and more complete pain relief is possible. b.No side effects or risks to the fetus are involved. c.The woman will remain fully alert at all times. d.Labor will likely be more rapid.

ANS: B Because nonpharmacologic pain management does not include analgesics, adjunct drugs, or anesthesia, it is harmless to the mother and the fetus. However, pain relief is lessened with nonpharmacologic pain management during childbirth. Although the woman's alertness is not altered by medication, the increase in pain may decrease alertness. Pain management may or may not alter the length of labor. At times when pain is decreased, the mother relaxes and labor progresses at a quicker pace.

A woman who has a seizure disorder and takes barbiturates and phenytoin sodium daily asks the nurse about the pill as a contraceptive choice. What is the nurses best response? a. Oral contraceptives are a highly effective method, but they have some side effects. b. Your current medications will reduce the effectiveness of the pill. c. Oral contraceptives will reduce the effectiveness of your seizure medication. d. The pill is a good choice for a woman of your age and with your personal history.

ANS: B Because the liver metabolizes oral contraceptives, their effectiveness is reduced when they are simultaneously taken with anticonvulsants. Stating that the pill is an effective birth control method with side effects is a true statement, but this response is not the most appropriate. The anticonvulsant reduces the effectiveness of the pill, not the other way around. Stating that the pill is a good choice for a woman of her age and personal history does not teach the client that the effectiveness of the pill may be reduced because of her anticonvulsant therapy.

The nurse who elects to work in the specialty of obstetric care must have the ability to distinguish between preterm birth, preterm labor, and low birth weight. Which statement regarding this terminology is correct? a.Terms preterm birth and low birth weight can be used interchangeably. b.Preterm labor is defined as cervical changes and uterine contractions occurring between 20 and 37 weeks of gestation. c.Low birth weight is a newborn who weighs below 3.7 pounds. d.Preterm birth rate in the United States continues to increase.

ANS: B Before 20 weeks of gestation, the fetus is not viable (miscarriage); after 37 weeks, the fetus can be considered term. Although these terms are used interchangeably, they have different meanings: preterm birth describes the length of gestation (before 37 weeks), regardless of the newborn's weight; low birth weight describes only the infant's weight at the time of birth (2500 g or less), whenever it occurs. Low birth weight is anything below 2500 g or approximately pounds. In 2011, the preterm birth rate in the United States was 11.7 %; it has dropped every year since 2008.

An 18-year-old client who has reached 16 weeks of gestation was recently diagnosed with pregestational diabetes. She attends her centering appointment accompanied by one of her girlfriends. This young woman appears more concerned about how her pregnancy will affect her social life than her recent diagnosis of diabetes. A number of nursing diagnoses are applicable to assist in planning adequate care. What is the most appropriate diagnosis at this time? a. Risk for injury, to the fetus related to birth trauma b. Deficient knowledge, related to diabetic pregnancy management c. Deficient knowledge, related to insulin administration d. Risk for injury, to the mother related to hypoglycemia or hyperglycemia

ANS: B Before a treatment plan is developed or goals for the outcome of care are outlined, this client must come to an understanding of diabetes and the potential effects on her pregnancy. She appears more concerned about changes to her social life than adopting a new self-care regimen. Risk for injury to the fetus related to either placental insufficiency or birth trauma may come later in the pregnancy. At this time, the client is having difficulty acknowledging the adjustments that she needs to make to her lifestyle to care for herself during pregnancy. The client may not yet be on insulin. Insulin requirements increase with gestation. The importance of glycemic control must be part of health teaching for this client. However, she has not yet acknowledged that changes to her lifestyle need to be made and may not participate in the plan of care until understanding takes place.

Which renal system adaptation is an anticipated anatomic change of pregnancy? a. Increased urinary output makes pregnant women less susceptible to urinary infections. b. Increased bladder sensitivity and then compression of the bladder by the enlarging uterus result in the urge to urinate even when the bladder is almost empty. c. Renal (kidney) function is more efficient when the woman assumes a supine position. d. Using diuretic agents during pregnancy can help keep kidney function regular.

ANS: B Bladder sensitivity and then compression of the bladder by the uterus result in the urge to urinate more often, even when the bladder is almost empty. A number of anatomic changes in pregnancy make a woman more susceptible to urinary tract infections. Renal function is more efficient when the woman lies in the lateral recumbent position and is less efficient when she is supine. Diuretic use during pregnancy can overstress the system and cause problems.

Ideally, when should prenatal care begin? a. Before the first missed menstrual period b. After the first missed menstrual period c. After the second missed menstrual period d. After the third missed menstrual period

ANS: B Prenatal care should begin soon after the first missed menstrual period. This offers the greatest opportunities to ensure the health of the expectant mother and her infant. Prenatal care before missing the first menstrual period is too early. It is unlikely the woman is even aware of the pregnancy. Ideally, prenatal visits should begin soon after the first period is missed. Beginning prenatal care after the third missed menstrual period is too late. The woman will have completed the first trimester by that time.

A 3-year-old girls mother is 6 months pregnant. What concern is this child most likely to verbalize? a. How the baby will get out? b. How will the baby eat? c. Will you die having the baby? d. What color eyes will the baby have?

ANS: B By age 3 or 4 years, children like to be told the story of their own beginning and accept it being compared with the present pregnancy. They like to listen to the fetal heartbeat and feel the baby move. Sometimes they worry about how the baby is being fed and what it will wear. School-age children take a more clinical interest in their mothers pregnancy and may want to know How did the baby get in there? and How will it get out? Whether the childs mother will die does not tend to be the focus of her questions about the impending birth of a sibling. The babys eye color does not tend to be the focus of childrens questions about the impending birth of a sibling.

Which ovarian neoplasm is described as a growth that contains hair, teeth, and sebaceous secretions? a.Ovarian fibroma b.Dermoid cyst c.Uterine polyp d.Follicular cyst

ANS: B Dermoid cysts are germ cell tumors, usually occurring in childhood, that may contain teeth, hair, bones, and sebaceous secretions and may unilaterally or bilaterally develop. Treatment is most often surgical removal. An ovarian fibroma is a solid ovarian neoplasm that develops from connective tissue, usually after menopause. A uterine polyp is a tumor that grows on the uterine wall on a stalk or pedicle. A follicular cyst develops within the ovaries of young women in response to follicle rupture and should resolve within one or two menstrual cycles.

Nurses, certified nurse-midwives, and other advanced practice nurses have the knowledge and expertise to assist women in making informed choices regarding contraception. A multidisciplinary approach should ensure that the womans social, cultural, and interpersonal needs are met. Which action should the nurse first take when meeting with a new client to discuss contraception? a. Obtain data about the frequency of coitus. b. Determine the womans level of knowledge concerning contraception and her commitment to any particular method. c. Assess the womans willingness to touch her genitals and cervical mucus. d. Evaluate the womans contraceptive life plan.

ANS: B Determining the womans level of knowledge concerning contraception and her commitment to any particular method is the primary step of this nursing assessment and necessary before completing the process and moving on to a nursing diagnosis. Once the clients level of knowledge is determined, the nurse can interact with the woman to compare options, reliability, cost, comfort level, protection from STIs, and her partners willingness to participate. Although important, obtaining data about the frequency of coitus is not the first action that the nurse should undertake when completing an assessment. Data should include not only the frequency of coitus but also the number of sexual partners, level of contraceptive involvement, and the partners objections. Assessing the womans willingness to touch herself is a key factor for the nurse to discuss should the client express an interest in using one of the fertility awareness methods of contraception. The nurse must be aware of the clients plan regarding whether she is attempting to prevent conception, delay conception, or conceive.

During a health history interview, a woman states that she thinks that she has bumps on her labia. She also states that she is not sure how to check herself. The correct response by the nurse would be what? a. Reassure the woman that the examination will reveal any problems. b. Explain the process of vulvar self-examination, and reassure the woman that she should become familiar with normal and abnormal findings during the examination. c. Reassure the woman that bumps can be treated. d. Reassure her that most women have bumps on their labia.

ANS: B During the assessment and evaluation, the responsibility for self-care, health promotion, and enhancement of wellness is emphasized. The pelvic examination provides a good opportunity for the practitioner to emphasize the need for regular vulvar self-examination. Providing reassurance to the woman concerning the bumps would not be an accurate response.

A new client and her partner arrive on the labor, delivery, recovery, and postpartum (LDRP) unit for the birth of their first child. The nurse applies the electronic fetal monitor (EFM) to the woman. Her partner asks you to explain what is printing on the graph, referring to the EFM strip. He wants to know what the baby's heart rate should be. What is the nurse's best response? a."Don't worry about that machine; that's my job." b."The baby's heart rate will fluctuate in response to what is happening during labor." c."The top line graphs the baby's heart rate, and the bottom line lets me know how strong the contractions are." d."Your physician will explain all of that later."

ANS: B Explaining what indicates a normal FHR teaches the partner about fetal monitoring and provides support and information to alleviate his fears. Telling the partner not to worry discredits his feelings and does not provide the teaching he is requesting. Telling the partner that the graph indicates how strong the contractions are provides inaccurate information and does not address the partner's concerns about the FHR. The EFM graphs the frequency and duration of the contractions, not their intensity. Nurses should take every opportunity to provide teaching to the client and her family, especially when information is requested.

A client is concerned because she has been experiencing some milky, sticky breast discharge. Which nonmalignant condition is exhibited with this finding? a.Relative inflammatory lesion b.Galactorrhea c.Mammary duct ectasia d.Breast infection

ANS: B Galactorrhea bilaterally exhibits a spontaneous, milky, and sticky discharge and is a normal finding during pregnancy; however, it may also occur as the result of elevated prolactin levels. Prolactin can become elevated as a result of a thyroid disorder, pituitary tumor, stress, coitus, trauma, or chest wall surgery.

To assist a client in managing the symptoms of PMS, what should the nurse recommend based on current evidence? a. Diet with more body-building and energy foods, such as carbohydrates b. Herbal therapies, yoga, and massage c. Antidepressants for symptom control d. Discouraging the use of diuretics

ANS: B Herbal therapies, yoga, and massage have been reported to have a beneficial effect on the symptoms of PMS. Limiting red meat, simple carbohydrates, caffeinated beverages, and alcohol improves the diet and may mitigate symptoms. Medication is usually begun only if lifestyle changes fail to provide significant relief. Natural diuretics may help reduce fluid retention.

An MSAFP screening indicates an elevated level of alpha-fetoprotein. The test is repeated, and again the level is reported as higher than normal. What is the next step in the assessment sequence to determine the well-being of the fetus? a. PUBS b. Ultrasound for fetal anomalies c. BPP for fetal well-being d. Amniocentesis for genetic anomalies

ANS: B If MSAFP findings are abnormal, then follow-up procedures include genetic counseling for families with a history of NTD, repeated MSAFP screenings, an ultrasound examination, and possibly amniocentesis. Indications for the use of PUBS include prenatal diagnosis of inherited blood disorders, karyotyping of malformed fetuses, detection of fetal infection, determination of the acid-base status of fetuses with IUGR, and assessment and treatment of isoimmunization and thrombocytopenia in the fetus. A BPP is a method of assessing fetal well-being in the third trimester. Before an amniocentesis, the client would have an ultrasound for direct visualization of the fetus.

Macromastia, or breast hyperplasia, is a condition in which women have very large and pendulous breasts. Breast hyperplasia can be corrected with a reduction mammoplasty. Which statement regarding this procedure is themost accurate? a.Breast reduction surgery is covered by insurance. b.Breastfeeding might be difficult. c.No sequelae after the procedure is known. d.Pain in the back and shoulders may not be relieved.

ANS: B If breast reduction surgery is performed, then establishing breastfeeding at a later date may be difficult. Macromastia may not be covered by all insurance companies. A consequence of surgery may be decreased sensation and pain, secondary to scar tissue. Reduction mammoplasty will relieve chronic neck and back pain.

Which maternal condition always necessitates delivery by cesarean birth? a.Marginal placenta previa b.Complete placenta previa c.Ectopic pregnancy d.Eclampsia

ANS: B In complete placenta previa, the placenta completely covers the cervical os. A cesarean birth is the acceptable method of delivery. The risk of fetal death occurring is due to preterm birth. If the previa is marginal (i.e., 2 cm or greater away from the cervical os), then labor can be attempted. A cesarean birth is not indicated for an ectopic pregnancy. Labor can be safely induced if the eclampsia is under control.

A client is seen at the clinic at 14 weeks of gestation for a follow-up appointment. At which level does the nurse expect to palpate the fundus? a. Nonpalpable above the symphysis at 14 weeks of gestation b. Slightly above the symphysis pubis c. At the level of the umbilicus d. Slightly above the umbilicus

ANS: B In normal pregnancies, the uterus grows at a predictable rate. It may be palpated above the symphysis pubis sometime between the 12th and 14th weeks of pregnancy. As the uterus grows, it may be palpated above the symphysis pubis sometime between the 12th and 14th weeks of pregnancy. At 14 weeks, the uterus is not yet at the level of the umbilicus. The fundus is not palpable above the umbilicus until 22 to 24 weeks of gestation.

The pancreas forms in the foregut during the 5th to 8th week of gestation. A client with poorly controlled gestational diabetes asks the nurse what the effects of her condition will be on the fetus. What is the best response by the nurse? Poorly controlled maternal gestational diabetes will: a. produce fetal hypoglycemia. b. result in a macrocosmic fetus. c. result in a microcosmic fetus. d. enhance lung maturation.

ANS: B Insulin is produced by week 20 of gestation. In the fetus of a mother with uncontrolled diabetes, maternal hypoglycemia produces fetal hypoglycemia and macrocosmia results. Hyperinsulinemia blocks lung maturation, placing the neonate at risk for respiratory distress.

Which statement made by a lactating woman leads the nurse to believe that the client might have lactose intolerance? a. I always have heartburn after I drink milk. b. If I drink more than a cup of milk, I usually have abdominal cramps and bloating. c. Drinking milk usually makes me break out in hives. d. Sometimes I notice that I have bad breath after I drink a cup of milk.

ANS: B Lactose intolerance, which is an inability to digest milk sugar because of a lack of the enzyme lactose in the small intestine, is a problem that interferes with milk consumption. Milk consumption may cause abdominal cramping, bloating, and diarrhea in such people, although many lactose-intolerant individuals can tolerate small amounts of milk without symptoms. A woman with lactose intolerance is more likely to experience bloating and cramping, not heartburn. A client who breaks out in hives after consuming milk is more likely to have a milk allergy and should be advised to simply brush her teeth after consuming dairy products.

Which client is at greatest risk for early PPH? a.Primiparous woman (G 2, P 1-0-0-1) being prepared for an emergency cesarean birth for fetal distress b.Woman with severe preeclampsia on magnesium sulfate whose labor is being induced c.Multiparous woman (G 3, P 2-0-0-2) with an 8-hour labor d.Primigravida in spontaneous labor with preterm twins

ANS: B Magnesium sulfate administration during labor poses a risk for PPH. Magnesium acts as a smooth muscle relaxant, thereby contributing to uterine relaxation and atony. A primiparous woman being prepared for an emergency cesarean birth for fetal distress, a multiparous woman with an 8-hour labor, and a primigravida in spontaneous labor with preterm twins do not indicate risk factors or causes of early PPH

A woman is 15 weeks pregnant with her first baby. She asks how long it will be before she feels the baby move. What is the nurses best answer? a. You should have felt the baby move by now. b. Within the next month, you should start to feel fluttering sensations. c. The baby is moving; however, you cant feel it yet. d. Some babies are quiet, and you dont feel them move.

ANS: B Maternal perception of fetal movement usually begins 16 to 20 weeks after conception. Because this is her first pregnancy, movement is felt toward the later part of the 16- to 20-week time period. Stating, you should have felt the baby move by now is incorrect and may be an alarming statement to the client. Fetal movement should be felt by 16 to 20 weeks. If movement is not felt by the end of that time, then further assessment is necessary.

Which menopausal discomfort would the nurse anticipate when evaluating a woman for signs and symptoms of the climacteric? a. Headaches b. Hot flashes c. Mood swings d. Vaginal dryness with dyspareunia

ANS: B Vasomotor instability, in the form of hot flashes or flushing, is a result of fluctuating estrogen levels and is the most common disturbance of the perimenopausal woman. Headaches may be associated with a decline in hormonal levels; however, headaches are not the most frequently reported discomfort for menopausal women. Mood swings may also be associated with a decline in hormonal levels; however, mood swings are not the most frequently reported discomfort for menopausal women. Vaginal dryness and dyspareunia may be associated with a decline in hormonal levels; however, both are not the most frequently reported discomforts for menopausal women.

While interviewing a 31-year-old woman before her routine gynecologic examination, the nurse collects data about the clients recent menstrual cycles. Which statement by the client should prompt the nurse to collect further information? a. My menstrual flow lasts 5 to 6 days. b. My flow is very heavy. c. I have had a small amount of spotting midway between my periods for the past 2 months. d. The length of my menstrual cycles varies from 26 to 29 days.

ANS: B Menorrhagia is defined as excessive menstrual bleeding, either in duration or in amount. Heavy bleeding can have many causes. The amount of bleeding and its effect on daily activities should be evaluated. A menstrual flow that lasts 5 to 6 days is a normal finding. Mittlestaining, a small amount of bleeding or spotting that occurs at the time of ovulation (14 days before the onset of the next menses), is considered normal. During her reproductive years, a woman may have physiologic variations in her menstrual cycle. Variations in the length of a menstrual cycle are considered normal.

The use of methamphetamine (meth) has been described as a significant drug problem in the United States. The nurse who provides care to this client population should be cognizant of what regarding methamphetamine use? a. Methamphetamines are similar to opiates. b. Methamphetamines are stimulants with vasoconstrictive characteristics. c. Methamphetamines should not be discontinued during pregnancy. d. Methamphetamines are associated with a low rate of relapse.

ANS: B Methamphetamines are stimulants with vasoconstrictive characteristics similar to cocaine and are similarly used. As is the case with cocaine users, methamphetamine users are urged to immediately stop all use during pregnancy. Unfortunately, because methamphetamine users are extremely psychologically addicted, the rate of relapse is extremely high.

What is the correct terminology for an abortion in which the fetus dies but is retained within the uterus? a.Inevitable abortion b.Missed abortion c.Incomplete abortion d.Threatened abortion

ANS: B Missed abortion refers to the retention of a dead fetus in the uterus. An inevitable abortion means that the cervix is dilating with the contractions. An incomplete abortion means that not all of the products of conception were expelled. With a threatened abortion, the woman has cramping and bleeding but no cervical dilation.

A newly married couple plans to use the natural family planning method of contraception. Understanding how long an ovum can live after ovulation is important to them. The nurse knows that his or her teaching was effective when the couple responds that an ovum is considered fertile for which period of time? a. 6 to 8 hours b. 24 hours c. 2 to 3 days d. 1 week

ANS: B Most ova remain fertile for approximately 24 hours after ovulation, much longer than 6 to 8 hours. However, ova do not remain fertile for 2 to 3 days or are viable for 1 week. If unfertilized by a sperm after 24 hours, the ovum degenerates and is reabsorbed.

A patient in her first trimester complains of nausea and vomiting. She asks, Why does this happen? What is the nurses best response? a. Nausea and vomiting are due to an increase in gastric motility. b. Nausea and vomiting may be due to changes in hormones. c. Nausea and vomiting are related to an increase in glucose levels. d. Nausea and vomiting are caused by a decrease in gastric secretions.

ANS: B Nausea and vomiting are believed to be caused by increased levels of hormones, decreased gastric motility, and hypoglycemia. Gastric motility decreases during pregnancy. Glucose levels decrease in the first trimester. Although gastric secretions decrease, these secretions are not the primary cause of the nausea and vomiting.

What is the role of the nurse as it applies to informed consent? a.Inform the client about the procedure, and ask her to sign the consent form. b.Act as a client advocate, and help clarify the procedure and the options. c.Call the physician to see the client. d.Witness the signing of the consent form.

ANS: B Nurses play a part in the informed consent process by clarifying and describing procedures or by acting as the woman's advocate and asking the primary health care provider for further explanations. The physician is responsible for informing the woman of her options, explaining the procedure, and advising the client about potential risk factors. The physician must be present to explain the procedure to the client. However, the nurse's responsibilities go further than simply asking the physician to see the client. The nurse may witness the signing of the consent form. However, depending on the state's guidelines, the woman's husband or another hospital health care employee may sign as a witness.

A pregnant woman is being discharged from the hospital after the placement of a cervical cerclage because of a history of recurrent pregnancy loss, secondary to an incompetent cervix. Which information regarding postprocedural care should the nurse emphasize in the discharge teaching? a.Any vaginal discharge should be immediately reported to her health care provider. b.The presence of any contractions, rupture of membranes (ROM), or severe perineal pressure should be reported. c.The client will need to make arrangements for care at home, because her activity level will be restricted. d.The client will be scheduled for a cesarean birth.

ANS: B Nursing care should stress the importance of monitoring for the signs and symptoms of preterm labor. Vaginal bleeding needs to be reported to her primary health care provider. Bed rest is an element of care. However, the woman may stand for periods of up to 90 minutes, which allows her the freedom to see her physician. Home uterine activity monitoring may be used to limit the woman's need for visits and to monitor her status safely at home. The cerclage can be removed at 37 weeks of gestation (to prepare for a vaginal birth), or a cesarean birth can be planned.

Maternal nutritional status is an especially significant factor of the many that influence the outcome of pregnancy. Why is this the case? a. Maternal nutritional status is extremely difficult to adjust because of an individuals ingrained eating habits. b. Adequate nutrition is an important preventive measure for a variety of problems. c. Women love obsessing about their weight and diets. d. A womans preconception weight becomes irrelevant.

ANS: B Nutritional status draws so much attention not only for its effect on a healthy pregnancy and birth but also because significant changes are within relatively easy reach. Pregnancy is a time when many women are motivated to learn about adequate nutrition and make changes to their diet that will benefit their baby. Pregnancy is not the time to begin a weight loss diet. Clients and their caregivers should still be concerned with appropriate weight gain.

According to professional standards (the Association of Women's Health, Obstetric and Neonatal Nurses [AWHONN], 2007), which action cannot be performed by the nonanesthetist registered nurse who is caring for a woman with epidural anesthesia? a.Monitoring the status of the woman and fetus b.Initiating epidural anesthesia c.Replacing empty infusion bags with the same medication and concentrate d.Stopping the infusion, and initiating emergency measures

ANS: B Only qualified, licensed anesthesia care providers are permitted to insert a catheter, initiate epidural anesthesia, verify catheter placement, inject medication through the catheter, or alter the medication or medications including type, amount, or rate of infusion. The nonanesthetist nurse is permitted to monitor the status of the woman, the fetus, and the progress of labor. Replacement of the empty infusion bags or syringes with the same medication and concentration is permitted. If the need arises, the nurse may stop the infusion, initiate emergency measures, and remove the catheter if properly educated to do so. Complications can require immediate interventions. Nurses must be prepared to provide safe and effective care during an emergency situation.

What should the laboring client who receives an opioid antagonist be told to expect? a.Her pain will decrease. b.Her pain will return. c.She will feel less anxious. d.She will no longer feel the urge to push.

ANS: B Opioid antagonists such as naloxone (Narcan) promptly reverse the CNS-depressant effects of opioids. In addition, the antagonist counters the effect of the stress-induced levels of endorphins. An opioid antagonist is especially valuable if the labor is more rapid than expected and the birth is anticipated when the opioid is at its peak effect. The woman should be told that the pain that was relieved by the opioid analgesic will return with the administration of the opioid antagonist. Her pain level will increase rather than decrease. Opioid antagonists have no effect on anxiety levels. They are primarily administered to reverse the excessive CNS depression in the mother, newborn, or both. An opioid antagonist (e.g., naloxone) has no effect on the mother's urge or ability to push. The practice of giving lower doses of IV opioids has reduced the incidence and severity of opioid-induced CNS depression; therefore, opioid antagonists are used less frequently.

What is the rationale for the administration of an oxytocic (e.g., Pitocin, Methergine) after expulsion of the placenta? a. To relieve pain b. To stimulate uterine contraction c. To prevent infection d. To facilitate rest and relaxation

ANS: B Oxytocics stimulate uterine contractions, which reduce blood loss after the third stage of labor. Oxytocics are not used to treat pain, do not prevent infection, and do not facilitate rest and relaxation.

What fatty acids (classified as hormones) are found in many body tissues with complex roles in many reproductive functions? a. GnRH b. Prostaglandins (PGs) c. FSH d. Luteinizing hormone (LH)

ANS: B PGs affect smooth muscle contraction and changes in the cervix. GnRH is part of the hypothalamic-pituitary cycle, which responds to the rise and fall of estrogen and progesterone. FSH is part of the hypothalamic-pituitary cycle, which responds to the rise and fall of estrogen and progesterone. LH is part of the hypothalamic-pituitary cycle, which responds to the rise and fall of estrogen and progesterone.

The prevalence of urinary incontinence (UI) increases as women age, with more than one third of the women in the United States suffering from some form of this disorder. The symptoms of mild-to-moderate UI can be successfully decreased by a number of strategies. Which of these should the nurse instruct the client to use first? a.Pelvic floor support devices b.Bladder training and pelvic muscle exercises c.Surgery d.Medications

ANS: B Pelvic muscle exercises, known as Kegel exercises, along with bladder training can significantly decrease or entirely relieve stress incontinence in many women. Pelvic floor support devices, also known as pessaries, come in a variety of shapes and sizes. Pessaries may not be effective for all women and require scrupulous cleaning to prevent infection. Anterior and posterior repairs and even a hysterectomy may be performed. If surgical repair is performed, then the nurse must focus her care on preventing infection and helping the woman avoid putting stress on the surgical site. Pharmacologic therapy includes selective serotonin-norepinephrine reuptake inhibitors or vaginal estrogen therapy. However, pharmacologic therapy is not the first action a nurse should recommend.

A pregnant woman at 18 weeks of gestation calls the clinic to report that she has been experiencing occasional backaches of mild-to-moderate intensity. Which intervention should the nurse recommend? a. Kegel exercises b. Pelvic rock exercises c. Softer mattress d. Bed rest for 24 hours

ANS: B Pelvic rock exercises may help stretch and strengthen the abdominal and lower back muscles and relieve low back pain. Stretching and other exercises to relieve back pain should be performed several times a day. Kegel exercises increase the tone of the pelvic area, not the back. A softer mattress may not provide the support needed to maintain proper alignment of the spine and may contribute to back pain.

How would the physiologic process of the sexual response best be characterized? a. Coitus, masturbation, and fantasy b. Myotonia and vasocongestion c. Erection and orgasm d. Excitement, plateau, and orgasm

ANS: B Physiologically, according to Masters (1992), sexual response can be analyzed in terms of two processes: vasocongestion and myotonia. Coitus, masturbation, and fantasy are forms of stimulation for the physical manifestation of the sexual response. Erection and orgasm occur in two of the four phases of the sexual response cycle. Excitement, plateau, and orgasm are three of the four phases of the sexual response cycle

Preconception counseling is critical in the safe management of diabetic pregnancies. Which complication is commonly associated with poor glycemic control before and during early pregnancy? a. Frequent episodes of maternal hypoglycemia b. Congenital anomalies in the fetus c. Hydramnios d. Hyperemesis gravidarum

ANS: B Preconception counseling is particularly important since strict metabolic control before conception and in the early weeks of gestation is instrumental in decreasing the risk of congenital anomalies. Frequent episodes of maternal hypoglycemia may occur during the first trimester (not before conception) as a result of hormonal changes and the effects on insulin production and use. Hydramnios occurs approximately 10 times more often in diabetic pregnancies than in nondiabetic pregnancies. Typically, it is observed in the third trimester of pregnancy. Hyperemesis gravidarum may exacerbate hypoglycemic events because the decreased food intake by the mother and glucose transfer to the fetus contribute to hypoglycemia.

A 41-week pregnant multigravida arrives at the labor and delivery unit after a NST indicated that her fetus could be experiencing some difficulties in utero. Which diagnostic tool yields more detailed information about the condition of the fetus? a. Ultrasound for fetal anomalies b. Biophysical profile (BPP) c. MSAFP screening d. Percutaneous umbilical blood sampling (PUBS)

ANS: B Real-time ultrasound permits a detailed assessment of the physical and physiologic characteristics of the developing fetus and a cataloging of normal and abnormal biophysical responses to stimuli. The BPP is a noninvasive, dynamic assessment of a fetus that is based on acute and chronic markers of fetal disease. An ultrasound for fetal anomalies would most likely have occurred earlier in the pregnancy. It is too late in the pregnancy to perform an MSAFP. Furthermore, it does not provide information related to fetal well-being. Indications for PUBS include prenatal diagnosis or inherited blood disorders, karyotyping of malformed fetuses, detection of fetal infection, determination of the acid-base status of the fetus with IUGR, and assessment and treatment of isoimmunization and thrombocytopenia in the fetus.

A first-time mother is concerned about the type of medications she will receive during labor. The client is in a fair amount of pain and is nauseated. In addition, she appears to be very anxious. The nurse explains that opioid analgesics are often used along with sedatives. How should the nurse phrase the rationale for this medication combination? a."The two medications, together, reduce complications." b."Sedatives enhance the effect of the pain medication." c."The two medications work better together, enabling you to sleep until you have the baby." d."This is what your physician has ordered for you."

ANS: B Sedatives may be used to reduce the nausea and vomiting that often accompany opioid use. In addition, some ataractic drugs reduce anxiety and apprehension and potentiate the opioid analgesic affects. A potentiator may cause two drugs to work together more effectively, but it does not ensure zero maternal or fetal complications. Sedation may be a related effect of some ataractic drugs; however, sedation is not the goal. Furthermore, a woman is unlikely to be able to sleep through transitional labor and birth. Although the physician may have ordered the medication, "This is what your physician has ordered for you" is not an acceptable comment for the nurse to make.

Which time-based description of a stage of development in pregnancy is correct? a. Viability22 to 37 weeks of gestation since the last menstrual period (assuming a fetal weight greater than 500 g) b. Termpregnancy from the beginning of 38 weeks of gestation to the end of 42 weeks of gestation c. Pretermpregnancy from 20 to 28 weeks of gestation d. Postdatepregnancy that extends beyond 38 weeks of gestation

ANS: B Term is 38 to 42 weeks of gestation. Viability is the ability of the fetus to live outside the uterus before coming to term, or 22 to 24 weeks since the last menstrual period. Preterm is 20 to 37 weeks of gestation. Postdateor postterm is a pregnancy that extends beyond 42 weeks of gestation or what is considered the limit of full term.

The lactational amenorrhea method (LAM) of birth control is popular in developing countries and has had limited use in the United States. As breastfeeding rates increase, more women may rely upon this method for birth control. Which information is most important to provide to the client interested in using the LAM for contraception? a. LAM is effective until the infant is 9 months of age. b. This popular method of birth control works best if the mother is exclusively breastfeeding. c. Its typical failure rate is 5%. d. Feeding intervals shoul

ANS: B The LAM works best if the mother is exclusively or almost exclusively breastfeeding. Disruption of the breastfeeding pattern increases the risk of pregnancy. After the infant is 6 months of age or menstrual flow has resumed, effectiveness decreases. The typical failure rate is 1% to 2%. Feeding intervals should be no greater than 4 hours during the day and 6 hours at night.

A client's oncologist has just finished explaining the diagnostic workup results to her, and she still has questions. The woman states, "The physician says I have a slow-growing cancer. Very few cells are dividing. How does she know this?" What is the name of the test that gave the health care provider this information? a.Tumor ploidy b.S-phase index c.Nuclear grade d.Estrogen-receptor assay

ANS: B The S-phase index measures the number of cells in the synthesis phase of cell development. If the number of cells noted is high, then the cancer is growing at a fast rate. In this client's case, her S-phase index is assumed to be low. Tumor ploidy is the amount of deoxyribonucleic acid (DNA) in a tumor cell, compared with that in a normal cell. Nuclear grade describes the degree of abnormalities present in the cancer cell tubules, the nuclei morphologic features, and mitotic rates. Estrogen and progesterone receptors are proteins found in the cell cytoplasm and surface of some breast cancer cells.

. Which description of the phases of the first stage of labor is most accurate? a. Latent: mild, regular contractions; no dilation; bloody show b. Active: moderate, regular contractions; 4 to 7 cm dilation c. Lull: no contractions; dilation stable d. Transition: very strong but irregular contractions; 8 to 10 cm dilation

ANS: B The active phase is characterized by moderate and regular contractions, 4 to 7 cm dilation, and duration of 3 to 6 hours. The latent phase is characterized by mild-to-moderate and irregular contractions, dilation up to 3 cm, brownish-to-pale pink mucus, and duration of 6 to 8 hours. No official "lull" phase exists in the first stage. The transition phase is characterized by strong to very strong and regular contractions, 8 to 10 cm dilation, and duration of 20 to 40 minutes.

A woman who has a history of sexual abuse may have a number of traumatic memories triggered during labor. She may fight the labor process and react with pain or anger. The nurse can implement a number of care measures to help her client view the childbirth experience in a positive manner. Which intervention is key for the nurse to use while providing care? a. Tell the client to relax and that it won't hurt much. b. Limit the number of procedures that invade her body. c. Reassure the client that, as the nurse, you know what is best. d. Allow unlimited care providers to be with the client.

ANS: B The number of invasive procedures such as vaginal examinations, internal monitoring, and IV therapy should be limited as much as possible. The nurse should always avoid words and phrases that may result in the client's recalling the phrases of her abuser (i.e., "Relax, this won't hurt" or "Just open your legs"). The woman's sense of control should be maintained at all times. The nurse should explain procedures at the client's pace and wait for permission to proceed. Protecting the client's environment by providing privacy and limiting the number of staff who observe the client will help to make her feel safe.

A 62-year-old woman has not been to the clinic for an annual examination for 5 years. The recent death of her husband reminded her that she should come for a visit. Her family physician has retired, and she is going to see the womens health nurse practitioner for her visit. What should the nurse do to facilitate a positive health care experience for this client? a. Remind the woman that she is long overdue for her examination and that she should come in annually. b. Carefully listen, and allow extra time for this womans health history interview. c. Reassure the woman that a nurse practitioner is just as good as her old physician. d. Encourage the woman to talk about the death of her husband and her fears about her own death.

ANS: B The nurse has an opportunity to use reflection and empathy while listening, as well as ensure an open and caring communication. Scheduling a longer appointment time may be necessary because older women may have longer histories or may need to talk. A respectful and reassuring approach to caring for women older than age 50 years can help ensure that they continue to seek health care. Reminding the woman about her overdue examination, reassuring the woman that she has a good practitioner, and encouraging conversation about the death of her husband and her own death are not the best approaches.

During the physical examination of a client beginning prenatal care, which initial action is most important for the nurse to perform? a. Only women who show physical signs or meet the sociologic profile should be assessed for physical abuse. b. The client should empty her bladder before the pelvic examination. c. The distribution, amount, and quality of body hair are of no particular importance. d. The size of the uterus is discounted in the initial examination because it will be increasing in size during the second trimester.

ANS: B The nurse should instruct the client to empty her bladder. An empty bladder facilitates the examination and also provides an opportunity to obtain a urine sample for a number of tests. All women should be assessed for a history of physical abuse, particularly because the likelihood of abuse increases during pregnancy. Noting body hair is important because body hair reflects nutritional status, endocrine function, and hygiene. Particular attention is paid to the size of the uterus because it is an indication of the duration of gestation.

Which client might be well advised to continue condom use during intercourse throughout her pregnancy? a. Unmarried pregnant women b. Women at risk for acquiring or transmitting STIs c. All pregnant women d. Women at risk for candidiasis

ANS: B The objective of safer sex is to provide prophylaxis against the acquisition and transmission of STIs. Because these diseases may be transmitted to the woman and then to her fetus, condom use is recommended throughout the pregnancy if the woman is at risk for acquiring an STI. Pregnant women are encouraged to practice safer sex behaviors. An unmarried pregnant woman may be in a monogamous relationship and not require the use of a condom. The client should be educated as to what may place both herself and her fetus at risk. Any pregnant woman can develop candidiasis, which is an infection not related to condom use.

The maternity nurse is cognizant of what important structure and function of the placenta? a. As the placenta widens, it gradually thins to allow easier passage of air and nutrients. b. As one of its early functions, the placenta acts as an endocrine gland. c. The placenta is able to keep out most potentially toxic substances, such as cigarette smoke, to which the mother is exposed. d. Optimal blood circulation is achieved through the placenta when the woman is lying on her back or standing.

ANS: B The placenta produces four hormones necessary to maintain the pregnancy: hCG, hPL, estrogen, and progesterone. The placenta widens until 20 weeks of gestation and continues to grow thicker. Toxic substances such as nicotine and carbon monoxide readily cross the placenta into the fetus. Optimal circulation occurs when the woman is lying on her side.

Of which physiologic alteration of the uterus during pregnancy is it important for the nurse to alert the patient? a. Lightening occurs near the end of the second trimester as the uterus rises into a different position. b. Womans increased urinary frequency in the first trimester is the result of exaggerated uterine antireflexion caused by softening. c. Braxton Hicks contractions become more painful in the third trimester, particularly if the woman tries to exercise. d. Uterine souffle is the movement of the fetus.

ANS: B The softening of the lower uterine segment is called the Hegar sign. In this position, the uterine fundus presses on the bladder, causing urinary frequency that is a normal change of pregnancy. Lightening occurs in the last 2 weeks of pregnancy, when the fetus descends. Braxton Hicks contractions become more defined in the final trimester but are not painful. Walking or exercise usually causes them to stop. The uterine souffle is the sound made by blood in the uterine arteries; it can be heard with a fetal stethoscope.

What is the rationale for the use of a blood patch after spinal anesthesia? a.Hypotension b.Headache c.Neonatal respiratory depression d.Loss of movement

ANS: B The subarachnoid block may cause a postspinal headache resulting from the loss of cerebrospinal fluid from the puncture in the dura. When blood is injected into the epidural space in the area of the dural puncture, it forms a seal over the hole to stop the leaking of cerebrospinal fluid. Hypotension is prevented by increasing fluid volume before the procedure. Neonatal respiratory depression is not an expected outcome with spinal anesthesia. Loss of movement is an expected outcome of spinal anesthesia.

Importantly, the nurse must be aware of which information related to the use of IUDs? a. Return to fertility can take several weeks after the device is removed. b. IUDs containing copper can provide an emergency contraception option if inserted within a few days of unprotected intercourse. c. IUDs offer the same protection against STIs as the diaphragm. d. Consent forms are not needed for IUD insertion

ANS: B The woman has up to 5 days to insert the IUD after unprotected sex. The return to fertility is immediate after the removal of the IUD. IUDs offer no protection against STIs. A consent form is required for insertion, as is a negative pregnancy test.

Which statement by the client indicates that she understands BSE? a. I will examine both breasts in two different positions. b. I will examine my breasts 1 week after my menstrual period starts. c. I will examine only the outer upper area of the breast. d. I will use the palm of the hand to perform the examination.

ANS: B The woman should examine her breasts when hormonal influences are at their lowest level. The client should be instructed to use four positions: standing with arms at her sides, standing with arms raised above her head, standing with hands pressed against hips, and lying down. The entire breast needs to be examined, including the outer upper area. The client should use the sensitive pads of the middle three fingers.

A pregnant woman at 29 weeks of gestation has been diagnosed with preterm labor. Her labor is being controlled with tocolytic medications. She asks when she might be able to go home. Which response by the nurse is most accurate? a."After the baby is born." b."When we can stabilize your preterm labor and arrange home health visits." c."Whenever your physician says that it is okay." d."It depends on what kind of insurance coverage you have."

ANS: B This client's preterm labor is being controlled with tocolytics. Once she is stable, home care may be a viable option for this type of client. Care of a client with preterm labor is multidisciplinary and multifactorial; the goal is to prevent delivery. In many cases, this goal may be achieved at home. Managed care may dictate an earlier hospital discharge or a shift from hospital to home care. Insurance coverage may be one factor in client care, but ultimately, client safety remains the most important factor.

A 26-year-old primigravida has come to the clinic for her regular prenatal visit at 12 weeks. She appears thin and somewhat nervous. She reports that she eats a well-balanced diet, although her weight is 5 pounds less than it was at her last visit. The results of laboratory studies confirm that she has a hyperthyroid condition. Based on the available data, the nurse formulates a plan of care. Which nursing diagnosis is most appropriate for the client at this time? a. Deficient fluid volume b. Imbalanced nutrition: less than body requirements c. Imbalanced nutrition: more than body requirements d. Disturbed sleep pattern

ANS: B This clients clinical cues include weight loss, which supports a nursing diagnosis of Imbalanced nutrition: less than body requirements. No clinical signs or symptoms support a nursing diagnosis of deficient fluid volume. This client reports weight loss, not weight gain. Although the client reports nervousness, the most appropriate nursing diagnosis, based on the clients other clinical symptoms, is Imbalanced nutrition: less than body requirements.

In terms of the incidence and classification of diabetes, which information should the nurse keep in mind when evaluating clients during their ongoing prenatal appointments? a. Type 1 diabetes is most common. b. Type 2 diabetes often goes undiagnosed. c. GDM means that the woman will receive insulin treatment until 6 weeks after birth. d. Type 1 diabetes may become type 2 during pregnancy.

ANS: B Type 2 diabetes often goes undiagnosed because hyperglycemia gradually develops and is often not severe. Type 2, sometimes called adult-onset diabetes, is the most common type of diabetes. GDM refers to any degree of glucose intolerance first recognized during pregnancy; insulin may or may not be needed. People do not go back and forth between type 1 and type 2 diabetes.

A womans obstetric history indicates that she is pregnant for the fourth time, and all of her children from previous pregnancies are living. One was born at 39 weeks of gestation, twins were born at 34 weeks of gestation, and another child was born at 35 weeks of gestation. What is her gravidity and parity using the GTPAL system? a. 3-1-1-1-3 b. 4-1-2-0-4 c. 3-0-3-0-3 d. 4-2-1-0-3

ANS: B Using the GTPAL system, 4-1-2-0-4 is the correct calculation of this womans gravidity and parity. The numbers reflect the womans gravidity and parity information. Her information is calculated as: G reflects the total number of times the woman has been pregnant; she is pregnant for the fourth time. T indicates the number of pregnancies carried to term, not the number of deliveries at term; only one of her pregnancies resulted in a fetus at term. P is the number of pregnancies that resulted in a preterm birth; the woman has had two pregnancies in which she delivered preterm. A signifies whether the woman has had any abortions or miscarriages before the period of viability; she has not. L signifies the number of children born who are currently living; the woman has four children. 3-1-1-1-3 is an incorrect calculation of this womans gravidity and parity; 3-0-3-0-3 is an incorrect calculation of this womans gravidity and parity; and 4-2-1-0-3 is an incorrect calculation of this womans gravidity and parity.

A client in late middle age who is certain she is not pregnant tells the nurse during an office visit that she has urinary problems, as well as sensations of bearing down and of something in her vagina. What condition would the nurse suspect based upon this report? a.Pelvic relaxation b.Cystoceles and/or rectoceles c.Uterine prolapse d.Genital fistulas

ANS: B Uterine displacement can be caused by congenital or acquired weakness of the pelvic support structures and is known as pelvic relaxation. Cystoceles are protrusions of the bladder downward into the vagina; rectoceles are herniations of the anterior rectal wall through a relaxed or ruptured vaginal fascia. Both can produce a bearing-down sensation with urinary dysfunction. They occur more often in older women who have borne children. Uterine prolapse is a more serious type of displacement. In women with a complete prolapse, the cervix and body of the uterus protrude through the vagina. Genital fistulas are perforations between genital tract organs. Most occur between the bladder and the genital tract.

A laboring woman with no known risk factors suddenly experiences spontaneous ROM. The fluid consists of bright red blood. Her contractions are consistent with her current stage of labor. No change in uterine resting tone has occurred. The fetal heart rate (FHR) begins to decline rapidly after the ROM. The nurse should suspect the possibility of what condition? a.Placenta previa b.Vasa previa c.Severe abruptio placentae d.Disseminated intravascular coagulation (DIC)

ANS: B Vasa previa is the result of a velamentous insertion of the umbilical cord. The umbilical vessels are not surrounded by Wharton jelly and have no supportive tissue. The umbilical blood vessels thus are at risk for laceration at any time, but laceration occurs most frequently during ROM. The sudden appearance of bright red blood at the time of ROM and a sudden change in the FHR without other known risk factors should immediately alert the nurse to the possibility of vasa previa. The presence of placenta previa most likely would be ascertained before labor and is considered a risk factor for this pregnancy. In addition, if the woman had a placenta previa, it is unlikely that she would be allowed to pursue labor and a vaginal birth. With the presence of severe abruptio placentae, the uterine tonicity typically is tetanus (i.e., a boardlike uterus). DIC is a pathologic form of diffuse clotting that consumes large amounts of clotting factors, causing widespread external bleeding, internal bleeding, or both. DIC is always a secondary diagnosis, often associated with obstetric risk factors such as the hemolysis, elevated liver enzyme levels, and low platelet levels (HELLP) syndrome. This woman did not have any prior risk factors.

Which information should the nurse take into consideration when planning care for a postpartum client with cardiac disease? a. The plan of care for a postpartum client is the same as the plan for any pregnant woman. b. The plan of care includes rest, stool softeners, and monitoring of the effect of activity. c. The plan of care includes frequent ambulating, alternating with active range-of-motion exercises. d. The plan of care includes limiting visits with the infant to once per day.

ANS: B Bed rest may be ordered, with or without bathroom privileges. Bowel movements without stress or strain for the woman are promoted with stool softeners, diet, and fluids. Care of the woman with cardiac disease in the postpartum period is tailored to the woman's functional capacity. The woman will be on bed rest to conserve energy and to reduce the strain on the heart. Although the woman may need help caring for the infant, breastfeeding and infant visits are not contraindicated.

Which information regarding the care of antepartum women with cardiac conditions is most important for the nurse to understand? a. Stress on the heart is greatest in the first trimester and the last 2 weeks before labor. b. Women with class II cardiac disease should avoid heavy exertion and any activity that causes even minor symptoms. c. Women with class III cardiac disease should get 8 to 10 hours of sleep every day and limit housework, shopping, and exercise. d. Women with class I cardiac disease need bed rest through most of the pregnancy and face the possibility of hospitalization near term.

ANS: B Class II cardiac disease is symptomatic with ordinary activity. Women in this category need to avoid heavy exertion and limit regular activities as symptoms dictate. Stress is greatest between weeks 28 and 32 of gestation, when hemodynamic changes reach their maximum. Class III cardiac disease is symptomatic with less-than-ordinary activity. These women need bed rest most of the day and face the possibility of hospitalization near term. Class I cardiac disease is asymptomatic at normal levels of activity. These women can perform limited normal activities with discretion, although they still need a good amount of sleep.

Which statement is the best rationale for assessing the maternal vital signs between uterine contractions? a. During a contraction, assessing the fetal heart rate is the priority. b. Maternal circulating blood volume temporarily increases during contractions. c. Maternal blood flow to the heart is reduced during contractions. d. Vital signs taken during contractions are not accurate.

ANS: B During uterine contractions, blood flow to the placenta temporarily stops, causing a relative increase in the mother's blood volume, which, in turn, temporarily increases blood pressure and slows the pulse. Monitoring fetal responses to the contractions is important; however, this question concerns the maternal vital signs. Maternal blood flow is increased during a contraction. Vital signs are altered by contractions but are considered accurate for that period.

What is the correct term describing the slight overlapping of cranial bones or shaping of the fetal head during labor? a. Lightening b. Molding c. Ferguson reflex d. Valsalva maneuver

ANS: B Molding also permits adaptation to various diameters of the maternal pelvis. Lightening is the mother's sensation of decreased abdominal distention, which usually occurs the week before labor. The Ferguson reflex is the contraction urge of the uterus after the stimulation of the cervix. The Valsalva maneuver describes conscious pushing during the second stage of labor.

Nurses can help their clients by keeping them informed about the distinctive stages of labor. Which description of the phases of the first stage of labor is accurate? a. Latent: Mild, regular contractions; no dilation; bloody show; duration of 2 to 4 hours b. Active: Moderate, regular contractions; 4- to 7-cm dilation; duration of 3 to 6 hours c. Lull: No contractions; dilation stable; duration of 20 to 60 minutes d. Transition: Very strong but irregular contractions; 8- to 10-cm dilation; duration of 1 to 2 hours

ANS: B The active phase is characterized by moderate, regular contractions; 4- to 7-cm dilation; and a duration of 3 to 6 hours. The latent phase is characterized by mild-to-moderate and irregular contractions; dilation up to 3 cm; brownish-to-pale pink mucus, and a duration of 6 to 8 hours. No official "lull" phase exists in the first stage. The transition phase is characterized by strong- to-very strong and regular contractions; 8- to 10-cm dilation; and a duration of 20 to 40 minutes.

Which statement related to fetal positioning during labor is correct and important for the nurse to understand? a. Position is a measure of the degree of descent of the presenting part of the fetus through the birth canal. b. Birth is imminent when the presenting part is at +4 to +5 cm below the spine. c. The largest transverse diameter of the presenting part is the suboccipitobregmatic diameter. d. Engagement is the term used to describe the beginning of labor.

ANS: B The station of the presenting part should be noted at the beginning of labor to determine the rate of descent. Position is the relationship of the presenting part of the fetus to the four quadrants of the mother's pelvis; station is the measure of degree of descent. The largest diameter is usually the biparietal diameter. The suboccipitobregmatic diameter is the smallest, although one of the most critical. Engagement often occurs in the weeks just before labor in nulliparous women and before or during labor in multiparous women.

The nurse has received a report regarding a client in labor. The woman's last vaginal examination was recorded as 3 cm, 30%, and -2. What is the nurse's interpretation of this assessment? a. Cervix is effaced 3 cm and dilated 30%; the presenting part is 2 cm above the ischial spines. b. Cervix is dilated 3 cm and effaced 30%; the presenting part is 2 cm above the ischial spines. c. Cervix is effaced 3 cm and dilated 30%; the presenting part is 2 cm below the ischial spines. d. Cervix is dilated 3 cm and effaced 30%; the presenting part is 2 cm below the ischial spines.

ANS: B The sterile vaginal examination is recorded as centimeters of cervical dilation, percentage of cervical dilation, and the relationship of the presenting part to the ischial spines (either above or below). For this woman, the cervix is dilated 3 cm and effaced 30%, and the presenting part is 2 cm above the ischial spines. The first interpretation of this vaginal examination is incorrect; the cervix is dilated 3 cm and is 30% effaced. However, the presenting part is correct at 2 cm above the ischial spines. The remaining two interpretations of this vaginal examination are incorrect. Although the dilation and effacement are correct at 3 cm and 30%, the presenting part is actually 2 cm above the ischial spines.

It is extremely rare for a woman to die in childbirth; however, it can happen. In the United States, the annual occurrence of maternal death is 12 per 100,000 cases of live birth. What are the leading causes of maternal death? a. Embolism and preeclampsia b. Trauma and motor vehicle accidents (MVAs) c. Hemorrhage and infection d. Underlying chronic conditions

ANS: B Trauma is the leading cause of obstetric death in women of childbearing age. Most maternal injuries are the result of MVAs and falls. Although preeclampsia and embolism are significant contributors to perinatal morbidity, these are not the leading cause of maternal mortality. Maternal death caused by trauma may occur as the result of hemorrhagic shock or abruptio placentae. In these cases, the hemorrhage is the result of trauma, not childbirth. The wish to become a parent is not eliminated by a chronic health problem, and many women each year risk their lives to have a baby. Because of advanced pediatric care, many women are surviving childhood illnesses and reaching adulthood with chronic health problems such as cystic fibrosis, diabetes, and pulmonary disorders.

Women of all ages will receive substantial and immediate benefits from smoking cessation. The process is not easy, and most people have attempted to quit numerous times before achieving success. Which organizations provide self-help and smoking cessation materials? (Select all that apply.) a. Leukemia and Lymphoma Society b. March of Dimes c. American Cancer Society d. American Lung Association e. Easter Seals

ANS: B, C, D The March of Dimes, the American Lung Association, and the American Cancer Society have self-help materials available. The Leukemia and Lymphoma Society support research for these two types of cancer. Easter Seals is best known for its work with disabled children.

Which FHR decelerations would require the nurse to change the maternal position? (Select all that apply.) a.Early decelerations b.Late decelerations c.Variable decelerations d.Moderate decelerations e.Prolonged decelerations

ANS: B, C, E Early decelerations (and accelerations) do not generally need any nursing intervention. Late decelerations suggest that the nurse should change the maternal position (lateral). Variable decelerations also require a maternal position change (side to side). Moderate decelerations are not an accepted category. Prolonged decelerations are late or variable decelerations that last for a prolonged period (longer than 2 minutes) and require intervention.

Indications for a primary cesarean birth are often nonrecurring. Therefore, a woman who has had a cesarean birth with a low transverse scar may be a candidate for vaginal birth after cesarean (VBAC). Which clients would beless likely to have a successful VBAC? (Select all that apply.) aLengthy interpregnancy interval b.African-American race c.Delivery at a rural hospital d.Estimated fetal weight <4000 g e.Maternal obesity (BMI >30)

ANS: B, C, E Indications for a low success rate for a VBAC delivery include a short interpregnancy interval, non-Caucasian race, gestational age longer than 40 weeks, maternal obesity, preeclampsia, fetal weight greater than 4000 g, and delivery at a rural or private hospital.

Postabortion instructions may differ among providers regarding tampon use and the resumption of intercourse. However, education should be provided regarding serious complications. When should the woman who has undergone an induced abortion be instructed to return to the emergency department? (Select all that apply.) a. Fever higher than 39 C b. Chills c. Foul-smelling vaginal discharge d. Bleeding greater than four pads in 2 hours e. Severe abdominal pain

ANS: B, C, E The client should report to a health care facility for any of the following symptoms: fever higher than 38 C, chills, bleeding more than two saturated pads in 2 hours or heavy bleeding lasting for days, foul-smelling discharge, abdominal tenderness or pain, and cramping or backache.

A tiered system of categorizing FHR has been recommended by professional organizations. Nurses, midwives, and physicians who care for women in labor must have a working knowledge of fetal monitoring standards and understand the significance of each category. What is the correct nomenclature for these categories? (Select all that apply.) a.Reassuring b.Category I c.Category II d.Nonreassuring e.Category III

ANS: B, C, E The three-tiered system of FHR tracings include category I, II, and III. Category I is a normal tracing requiring no action. Category II FHR tracings are indeterminate and includes tracings that do not meet category I or III criteria. Category III tracings are abnormal and require immediate intervention.

The client and her partner are considering male sterilization as a form of permanent birth control. While educating the client regarding the risks and benefits of the procedure, which information should the nurse include? (Select all that apply.) a. Sterilization should be performed under general anesthesia. b. Pain, bleeding, and infection are possible complications. c. Pregnancy may still be possible. d. Vasectomy may affect potency. e. Secondary sex characteristics are unaffected.

ANS: B, C, E Vasectomy is the most commonly used procedure for male sterilization and is performed on an outpatient basis under local anesthesia. Pain, bleeding, swelling, and infection are considered complications. Reversal is generally unsuccessful; however, it may take several weeks to months for all sperm to be cleared from the sperm ducts. Another form of contraception is necessary until the sperm counts are zero. Vasectomy has no effect on potency, and secondary sex characteristics are not affected.

Guidelines for breast cancer screening continue to evolve as new evidence is generated. Which examination or procedure and frequency would be recommended for a 31-year-old asymptomatic client? (Select all that apply.) a.Annual mammography b.Clinical breast examination every 3 years c.Annual MRI d.Breast self-examination e.Mammography every 3 years

ANS: B, D A 31-year-old client with no risk factors and who is asymptomatic should perform breast self-examination on a regular basis and have a clinical breast examination every 3 years. Women who are 40 years of age and older require both mammography and clinical breast examination annually. High-risk women 30 years and older should have an annual MRI and mammogram.

Which information should nurses provide to expectant mothers when teaching them how to evaluate daily fetal movement counts (DFMCs)? a. Alcohol or cigarette smoke can irritate the fetus into greater activity. b. Kick counts should be taken every hour and averaged every 6 hours, with every other 6-hour stretch off. c. The fetal alarm signal should go off when fetal movements stop entirely for 12 hours. d. A count of less than four fetal movements in 1 hour warrants future evaluation.

ANS: C No movement in a 12-hour period is cause for investigation and possibly intervention. Alcohol and cigarette smoke temporarily reduce fetal movement. The mother should count fetal activity (kick counts) two or three times daily for 60 minutes each time. A count of less than 3 in 1 hour warrants further evaluation by a NST.

The nurse recognizes that uterine hyperstimulation with oxytocin requires emergency interventions. What clinical cues alert the nurse that the woman is experiencing uterine hyperstimulation? (Select all that apply.) a.Uterine contractions lasting <90 seconds and occurring >2 minutes in frequency b.Uterine contractions lasting >90 seconds and occurring <2 minutes in frequency c.Uterine tone <20 mm Hg d.Uterine tone >20 mm Hg e.Increased uterine activity accompanied by a nonreassuring FHR and pattern

ANS: B, D, E Uterine contractions that occur less frequently than 2 minutes apart and last longer than 90 seconds, a uterine tone over 20 mm Hg, and a nonreassuring FHR and pattern are indications of uterine hyperstimulation with oxytocin administration. Uterine contractions that occur more frequently than 2 minutes apart and last less than 90 seconds are the expected goal of oxytocin induction. A uterine tone less than 20 mm Hg is normal.

According to the National Institute of Child Health and Human Development (NICHD) Three-Tier System of FHR Classification, category III tracings include all FHR tracings not categorized as category I or II. Which characteristics of the FHR belong in category III? (Select all that apply.) a.Baseline rate of 110 to 160 beats per minute b.Tachycardia c.Absent baseline variability not accompanied by recurrent decelerations d.Variable decelerations with other characteristics such as shoulders or overshoots e.Absent baseline variability with recurrent variable decelerations f.Bradycardia

ANS: B, D, E, F Tachycardia, variable decelerations with other characteristics, absent baseline variability with recurrent variable decelerations, and bradycardia are characteristics that are considered nonreassuring or abnormal and belong in category III. A FHR of 110 to 160 beats per minute is considered normal and belongs in category I. Absent baseline variability not accompanied by recurrent decelerations is a category II characteristic

In the past, factors to determine whether a woman was likely to develop a high-risk pregnancy were primarily evaluated from a medical point of view. A broader, more comprehensive approach to high-risk pregnancy has been adopted today. Four categories have now been established, based on the threats to the health of the woman and the outcome of pregnancy. Which category should not be included in this group? a. Biophysical b. Psychosocial c. Geographic d. Environmental

ANS: C A geographic category is correctly referred to as sociodemographic risk. These factors stem from the mother and her family. Ethnicity may be one of the risks to pregnancy; however, it is not the only factor in this category. Low income, lack of prenatal care, age, parity, and marital status also are included. Biophysical is one of the broad categories used for determining risk. These include genetic considerations, nutritional status, and medical and obstetric disorders. Psychosocial risks include smoking, caffeine, drugs, alcohol, and psychologic status. All of these adverse lifestyles can have a negative effect on the health of the mother or fetus. Environmental risks are risks that can affect both fertility and fetal development. These include infections, chemicals, radiation, pesticides, illicit drugs, and industrial pollutants.

A woman in labor is breathing into a mouthpiece just before the start of her regular contractions. As she inhales, a valve opens and gas is released. She continues to inhale the gas slowly and deeply until the contraction starts to subside. When the inhalation stops, the valve closes. Which statement regarding this procedure is correct? a.The application of nitrous oxide gas is not often used anymore. b.An inhalation of gas is likely to be used in the second stage of labor, not during the first stage. c.An application of nitrous oxide gas is administered for pain relief. d.The application of gas is a prelude to a cesarean birth.

ANS: C A mixture of nitrous oxide with oxygen in a low concentration can be used in combination with other nonpharmacologic and pharmacologic measures for pain relief. This procedure is still commonly used in Canada and in the United Kingdom. Nitrous oxide inhaled in a low concentration will reduce but not eliminate pain during the first and second stages of labor. Nitrous oxide inhalation is not generally used before a caesarean birth. Nitrous oxide does not appear to depress uterine contractions or cause adverse reactions in the newborn.

Which statement regarding the development of the respiratory system is a high priority for the nurse to understand? a. The respiratory system does not begin developing until after the embryonic stage. b. The infants lungs are considered mature when the L/S ratio is 1:1, at approximately 32 weeks of gestation. c. Maternal hypertension can reduce maternal-placental blood flow, accelerating lung maturity. d. Fetal respiratory movements are not visible on ultrasound scans until at least 16 weeks of gestation.

ANS: C A reduction in placental blood flow stresses the fetus, increases blood levels of corticosteroids, and thus accelerates lung maturity. The development of the respiratory system begins during the embryonic phase and continues into childhood. The infants lungs are considered mature when the L/S ratio is 2:1, at approximately 35 weeks of gestation. Lung movements have been visualized on ultrasound scans at 11 weeks of gestation.

Which questionnaire would be best for the nurse to use when screening an adolescent client for an eating disorder? a. Four Cs b. Dietary Guidelines for Americans c. SCOFF screening tool d. Dual-energy x-ray absorptiometry (DEXA) scan

ANS: C A screening tool specifically developed to identify eating disorders uses the acronym SCOFF. Each question scores 1 point. A score of 2 or more indicates that the client may have anorexia nervosa or bulimia. The letters represent the following questions: Do you make yourself Sick because you feel too full? Do you worry about loss of Control over the amount that you eat? Have you recently lost more than One stone (14 pounds) in a 3-month period? Do you think that you are too Fat, even if others think you are thin? Does Food dominate your life? The 4 Cs are used to determine cultural competence. Dietary Guidelines for Americans provide nutritional guidance for all, not only for those with eating disorders. The DEXA scan is used to determine bone density.

A woman arrives at the emergency department with complaints of bleeding and cramping. The initial nursing history is significant for a last menstrual period 6 weeks ago. On sterile speculum examination, the primary care provider finds that the cervix is closed. The anticipated plan of care for this woman would be based on a probable diagnosis of which type of spontaneous abortion? a.Incomplete b.Inevitable c.Threatened d.Septic

ANS: C A woman with a threatened abortion has spotting, mild cramps, and no cervical dilation. A woman with an incomplete abortion would have heavy bleeding, mild-to-severe cramping, and cervical dilation. An inevitable abortion demonstrates the same symptoms as an incomplete abortion: heavy bleeding, mild-to-severe cramping, and cervical dilation. A woman with a septic abortion has malodorous bleeding and typically a dilated cervix.

When caring for clients with neoplasms of the reproductive system, the nurse must begin by assessing the woman's knowledge of the disorder, its management, and prognosis. This assessment should be followed by a nursing diagnosis. Which diagnosis fails to address the psychologic effect of these disorders? a.Anxiety, related to surgical procedures b.Disturbed body image, as a result of changes in anatomy c.Risk for injury, related to lack of skill for self-care d.Interrupted family processes

ANS: C Although risk for injury, related to lack of skill for self-care, is appropriate to this client's condition, this diagnosis is more suited to the client's learning needs than the psychologic effect. Anxiety, related to surgical procedures, is appropriate for addressing psychosocial concerns; the client may also develop anxiety related to the diagnosis and prognosis and whether or not surgery is required. Disturbed body image is an applicable diagnosis; changes in her anatomy and function may also result in low self-esteem and ineffective coping skills. Interrupted family processes is a possible and acceptable diagnosis; functional and anatomic changes may result in the client's inability to fulfill her familial role. Depending on the severity of her condition, interrupted family processes could also lead to social isolation.

Some pregnant clients may complain of changes in their voice and impaired hearing. What should the nurse explain to the client concerning these findings? a. Voice changes are caused by decreased estrogen levels. b. Displacement of the diaphragm results in thoracic breathing. c. Voice changes and impaired hearing are due to the results of congestion and swelling of the upper respiratory tract. d. Increased blood volume causes changes in the voice.

ANS: C Although the diaphragm is displaced and the volume of blood is increased, neither causes changes in the voice nor impairs hearing. The key is that estrogen levels increase, not decrease, which causes the upper respiratory tract to become more vascular, which produces swelling and congestion in the nose and ears and therefore voice changes and impaired hearing.

A woman arrives at the clinic for her annual examination. She tells the nurse that she thinks she has a vaginal infection, and she has been using an over-the-counter cream for the past 2 days to treat it. How should the nurse initially respond? a. Inform the woman that vaginal creams may interfere with the Papanicolaou (Pap) test for which she is scheduled. b. Reassure the woman that using vaginal cream is not a problem for the examination. c. Ask the woman to describe the symptoms that indicate to her that she has a vaginal infection. d. Ask the woman to reschedule the appointment for the examination.

ANS: C An important element of the health history and physical examination is the clients description of any symptoms she may be experiencing. The best response is for the nurse to inquire about the symptoms the woman is experiencing. Women should not douche, use vaginal medications, or have sexual intercourse for 24 to 48 hours before obtaining a Pap test. Although the woman may need to reschedule a visit for her Pap test, her current symptoms should still be addressed.

A laboring woman has received meperidine (Demerol) intravenously (IV), 90 minutes before giving birth. Which medication should be available to reduce the postnatal effects of meperidine on the neonate? a.Fentanyl (Sublimaze) b.Promethazine (Phenergan) c.Naloxone (Narcan) d.Nalbuphine (Nubain)

ANS: C An opioid antagonist can be given to the newborn as one part of the treatment for neonatal narcosis, which is a state of central nervous system (CNS) depression in the newborn produced by an opioid. Opioid antagonists, such as naloxone (Narcan), can promptly reverse the CNS depressant effects, especially respiratory depression. Fentanyl (Sublimaze), promethazine (Phenergan), and nalbuphine (Nubain) do not act as opioid antagonists to reduce the postnatal effects of meperidine on the neonate.

A woman with worsening preeclampsia is admitted to the hospitals labor and birth unit. The physician explains the plan of care for severe preeclampsia, including the induction of labor, to the woman and her husband. Which statement by the husband leads the nurse to believe that the couple needs further information? a. I will help my wife use the breathing techniques that we learned in our childbirth classes. b. I will give my wife ice chips to eat during labor. c. Since we will be here for a while, I will call my mother so she can bring the two boys2 years and 4 years of ageto visit their mother. d. I will stay with my wife during her labor, just as we planned.

ANS: C Arranging a visit with their two children indicates that the husband does not understand the importance of the quiet, subdued environment that is needed to prevent his wifes condition from worsening. Implementing breathing techniques is indicative of adequate knowledge related to pain management during labor. Administering ice chips indicates an understanding of nutritional needs during labor. Staying with his wife during labor demonstrates the husbands support for his wife and is appropriate.

A client in the third trimester has just undergone an amniocentesis to determine fetal lung maturity. Which statement regarding this testing is important for the nurse in formulating a care plan? a. Because of new imaging techniques, an amniocentesis should have been performed in the first trimester. b. Despite the use of ultrasonography, complications still occur in the mother or infant in 5% to 10% of cases. c. Administration of Rho(D) immunoglobulin may be necessary. d. The presence of meconium in the amniotic fluid is always a cause for concern.

ANS: C As a result of the possibility of fetomaternal hemorrhage, administration of Rho(D) immunoglobulin is the standard of practice after amniocentesis for women who are Rh negative. Amniocentesis is possible after the 14th week of pregnancy when the uterus becomes an abdominal organ. Complications occur in less than 1% of cases; many have been minimized or eliminated through the use of ultrasonography. Meconium in the amniotic fluid before the beginning of labor is not usually a problem.

The nurse guides a woman to the examination room and asks her to remove her clothes and put on an examination gown with the front open. The woman replies, I have special undergarments that I do not remove for religious reasons. Which is the most appropriate response from the nurse? a. You cant have an examination without removing all your clothes. b. Ill ask the physician to modify the examination. c. Tell me about your undergarments. Ill explain the examination procedure, and then we can discuss how you can comfortably have your examination. d. I have no idea how we can accommodate your beliefs.

ANS: C Explaining the examination procedure reflects cultural competence by the nurse and shows respect for the womans religious practices. The nurse must respect the rich and unique qualities that cultural diversity brings to individuals. The examination can be modified to ensure that modesty is maintained. In recognizing the value of cultural differences, the nurse can modify the plan of care to meet the needs of each woman. Telling the client that her religious practices are different or strange is inappropriate and disrespectful to the client.

During the initial visit with a client who is beginning prenatal care, which action should be the highest priority for the nurse? a. The first interview is a relaxed, get-acquainted affair during which the nurse gathers some general impressions of his or her new client. b. If the nurse observed handicapping conditions, he or she should be sensitive and not inquire about them because the client will do that in her own time. c. The nurse should be alert to the appearance of potential parenting problems, such as depression or lack of family support. d. Because of legal complications, the nurse should not ask about illegal drug use; that is left to the physician.

ANS: C Besides these potential problems, the nurse needs to be alert to the womans attitude toward keeping regular health care appointments. If the client lacks insurance, then the nurse may be able to direct her to resources that provide assistance for pregnant women (i.e., Women, Infants, and Children [WIC]; Medicaid). The initial interview needs to be planned, purposeful, and focused on specific content. A lot of ground must be covered. The nurse must be sensitive to special problems; he or she should inquire because discovering individual needs is important. A client with a chronic or handicapping condition might forget to mention it because she has adapted to it. Obtaining information on drug use is important and can be confidentially done. Actual testing for drug use requires the clients consent.

Breast pain occurs in many women during their perimenopausal years. Which information is a priority for the nurse to share with the client? a.Breast pain is an early indication of cancer. b.Pain is almost always an indication of a solid mass. c.Distinguishing between cyclical and noncyclical pain is important. d.Breast pain is most often treated with narcotics.

ANS: C Breast pain is unusual in breast cancer. Solid masses are generally benign and described as smooth, round, mobile, and painless. Distinguishing between cyclical and noncyclical pain is important to determine whether the cause is hormonal. Idiopathic pain is most often treated with nonsteroidal antiinflammatory medications.

While working with the pregnant client in her first trimester, what information does the nurse provide regarding when CVS can be performed (in weeks of gestation)? a. 4 b. 8 c. 10 d. 14

ANS: C CVS can be performed in the first or second trimester, ideally between 10 and 13 weeks of gestation. During this procedure, a small piece of tissue is removed from the fetal portion of the placenta. If performed after 9 completed weeks of gestation, then the risk of limb reduction is no greater than in the general population.

A woman arrives for evaluation of signs and symptoms that include a missed period, adnexal fullness, tenderness, and dark red vaginal bleeding. On examination, the nurse notices an ecchymotic blueness around the woman's umbilicus. What does this finding indicate? a.Normal integumentary changes associated with pregnancy b.Turner sign associated with appendicitis c.Cullen sign associated with a ruptured ectopic pregnancy d.Chadwick sign associated with early pregnancy

ANS: C Cullen sign, the blue ecchymosis observed in the umbilical area, indicates hematoperitoneum associated with an undiagnosed ruptured intraabdominal ectopic pregnancy. Linea nigra on the abdomen is the normal integumentary change associated with pregnancy and exhibits a brown pigmented, vertical line on the lower abdomen. Turner sign is ecchymosis in the flank area, often associated with pancreatitis. A Chadwick sign is a blue-purple cervix that may be seen during or around the eighth week of pregnancy.

Which is the initial treatment for the client with vWD who experiences a PPH? a.Cryoprecipitate b.Factor VIII and von Willebrand factor (vWf) c.Desmopressin d.Hemabate

ANS: C Desmopressin is the primary treatment of choice for vWD and can be administered orally, nasally, and intravenously. This medication promotes the release of factor VIII and vWf from storage. Cryoprecipitate may be used; however, because of the risk of possible donor viruses, other modalities are considered safer. Treatment with plasma products such as factor VIII and vWf is an acceptable option for this client. Because of the repeated exposure to donor blood products and possible viruses, this modality is not the initial treatment of choice. Although the administration of the prostaglandin, Hemabate, is known to promote contraction of the uterus during PPH, it is not effective for the client who has a bleeding disorder.

During a prenatal visit, the nurse is explaining dietary management to a woman with pregestational diabetes. Which statement by the client reassures the nurse that teaching has been effective? a. I will need to eat 600 more calories per day because I am pregnant. b. I can continue with the same diet as before pregnancy as long as it is well balanced. c. Diet and insulin needs change during pregnancy. d. I will plan my diet based on the results of urine glucose testing.

ANS: C Diet and insulin needs change during the pregnancy in direct correlation to hormonal changes and energy needs. In the third trimester, insulin needs may double or even quadruple. The diet is individualized to allow for increased fetal and metabolic requirements, with consideration of such factors as prepregnancy weight and dietary habits, overall health, ethnic background, lifestyle, stage of pregnancy, knowledge of nutrition, and insulin therapy. Energy needs are usually calculated on the basis of 30 to 35 calories per kilogram of ideal body weight. Dietary management during a diabetic pregnancy must be based on blood, not urine, glucose changes

A pregnant woman who abuses cocaine admits to exchanging sex to finance her drug habit. This behavior places the client at the greatest risk for what? a. Depression of the CNS b. Hypotension and vasodilation c. Sexually transmitted infections (STIs) d. Postmature birth

ANS: C Exchanging sex acts for drugs places the woman at increased risk for STIs because of multiple partners and the lack of protection. Cocaine is a CNS stimulant that causes hypertension and vasoconstriction. Premature delivery of the infant is one of the more common problems associated with cocaine use during pregnancy.

A nurse is providing breast care education to a client after mammography. Which information regarding fibrocystic changes in the breast is important for the nurse to share? a.Fibrocystic breast disease is a disease of the milk ducts and glands in the breasts. b.It is a premalignant disorder characterized by lumps found in the breast tissue. c.Healthy women with fibrocystic breast disease find lumpiness with pain and tenderness in varying degrees in the breast tissue during menstrual cycles. d.Lumpiness is accompanied by tenderness after menses.

ANS: C Fibrocystic changes are palpable thickenings in the breast usually associated with pain and tenderness. The pain and tenderness fluctuate with the menstrual cycle. Fibrocystic changes are not premalignant changes; this information is inaccurate. Tenderness most often occurs before menses.

Which statement accurately describes the centering model of care? a. Group sessions begin with the first prenatal visit. b. Blood pressure (BP), weight, and urine dipsticks are obtained by the nurse at each visit. c. Approximately 8 to 12 women are placed in each gestational-age cohort group. d. Outcomes are similar to traditional prenatal care.

ANS: C Gestational-age cohorts comprise the groups, with approximately 8 to 12 women in each group. The groups remain intact throughout the pregnancy. Individual follow-up visits are scheduled as needed. Group sessions begin at 12 to 16 weeks of gestation and end with an early postpartum visit. Before the group sessions, the client has an individual assessment, physical examination, and history. At the beginning of each group meeting, clients measure their own BP, weight, and urine dips and enter these findings in their record. Fetal heart rate assessment and fundal height are obtained by the nurse. Results evaluating this approach have been very promising. In a recent study of adolescent clients, the number of LBW infants decreased and breastfeeding rates increased.

The labor of a pregnant woman with preeclampsia is going to be induced. Before initiating the oxytocin (Pitocin) infusion, the nurse reviews the womans latest laboratory test findings, which reveal a platelet count of 90,000 mm3, an elevated aspartate aminotransaminase (AST) level, and a falling hematocrit. The laboratory results are indicative of which condition? a. Eclampsia b. Disseminated intravascular coagulation (DIC) syndrome c. Hemolysis, elevated liver enzyme levels, and low platelet levels (HELLP) syndrome d. Idiopathic thrombocytopenia

ANS: C HELLP syndrome is a laboratory diagnosis for a variant of severe preeclampsia that involves hepatic dysfunction characterized by hemolysis (H), elevated liver (EL) enzymes, and low platelets (LP). Eclampsia is determined by the presence of seizures. DIC is a potential complication associated with HELLP syndrome. Idiopathic thrombocytopenia is the presence of low platelets of unknown cause and is not associated with preeclampsia.

With regard to weight gain during pregnancy, the nurse should be aware of which important information? a. In pregnancy, the womans height is not a factor in determining her target weight. b. Obese women may have their health concerns, but their risk of giving birth to a child with major congenital defects is the same as with women of normal weight. c. Women with inadequate weight gain have an increased risk of delivering a preterm infant with intrauterine growth restriction (IUGR). d. Greater than expected weight gain during pregnancy is almost always attributable to old-fashioned overeating.

ANS: C IUGR is associated with women with inadequate weight gain. The primary factor in making a weight gain recommendation is the appropriateness of the prepregnancy weight for the womans height. Obese women are twice as likely as women of normal weight to give birth to a child with major congenital defects. Overeating is only one of several likely causes.

A woman has arrived for her preoperative testing appointment. She is scheduled for a myomectomy the following day. What condition would require the client to undergo this procedure for symptom relief? a.Numerous small fibroid tumors b.Bartholin cysts c.Fibroid tumors near the outer wall of the uterus with a uterine size no larger than at 12 weeks of gestation d.Leiomyomas (also known as fibroid tumors) in a uterus larger than 14 weeks of gestation

ANS: C If a fibroid tumor lies near the outer wall of the uterus and the uterine size is no larger than at 12 to 14 weeks of gestation and the symptoms are significant, a myomectomy (i.e., removal of the tumor) may be performed. This procedure leaves the uterine walls relatively intact, thereby preserving the uterus for future pregnancies. Laser surgery or electrocauterization can be safely used to destroy numerous small fibroid tumors. Bartholin cysts are benign lesions of the vulva. If the cyst is symptomatic or infected, surgical incision and drainage may provide relief. A hysterectomy (i.e., removal of the entire uterus) is the treatment of choice if bleeding is severe or if the fibroid tumor is obstructing the normal function of other organs.

A woman who has just undergone a first-trimester abortion will be using oral contraceptives. To protect against pregnancy, the client should be advised to do what? a. Avoid sexual contact for at least 10 days after starting the pill. b. Use condoms and foam for the first few weeks as a backup. c. Use another method of contraception for 1 week after starting the pill. d. Begin sexual relations once vaginal bleeding has ended.

ANS: C If oral contraceptives are to be started within 3 weeks after an abortion, additional forms of contraception should be used throughout the first week to avoid the risk of pregnancy.

In the acronym BRAIDED, which letter is used to identify the key components of informed consent that the nurse must document? a. B stands for birth control. b. R stands for reproduction. c. A stands for alternatives. d. I stands for ineffective.

ANS: C In the acronym BRAIDED, A stands for alternatives and information about other viable methods. B stands for benefits and information about the advantages of a particular birth control method and its success rates. Rstands for risks and information about the disadvantages of a particular method and its failure rates. I stands for inquiries and the opportunity to ask questions.

What is the primary rationale for the thorough drying of the infant immediately after birth? a. Stimulates crying and lung expansion b. Removes maternal blood from the skin surface c. Reduces heat loss from evaporation d. Increases blood supply to the hands and feet

ANS: C Infants are wet with amniotic fluid and blood at birth, and this accelerates evaporative heat loss. The primary purpose of drying the infant is to prevent heat loss. Although rubbing the infant stimulates crying, it is not the main reason for drying the infant. This process does not remove all the maternal blood.

Which minerals and vitamins are usually recommended as a supplement in a pregnant clients diet? a. Fat-soluble vitamins A and D b. Water-soluble vitamins C and B6 c. Iron and folate d. Calcium and zinc

ANS: C Iron should generally be supplemented, and folic acid supplements are often needed because folate is so important in pregnancy. Fat-soluble vitamins should be supplemented as a medical prescription, as vitamin D might be for lactose-intolerant women. Water-soluble vitamin C is sometimes naturally consumed in excess; vitamin B6 is prescribed only if the woman has a very poor diet; and zinc is sometimes supplemented. Most women get enough calcium.

The nurse is providing education to a client regarding the normal changes of the breasts during pregnancy. Which statement regarding these changes is correct? a. The visibility of blood vessels that form an intertwining blue network indicates full function of the Montgomery tubercles and possibly an infection of the tubercles. b. The mammary glands do not develop until 2 weeks before labor. c. Lactation is inhibited until the estrogen level declines after birth. d. Colostrum is the yellowish oily substance used to lubricate the nipples for breastfeeding.

ANS: C Lactation is inhibited until after birth. The visible blue network of blood vessels is a normal outgrowth of a richer blood supply. The mammary glands are functionally complete by midpregnancy. Colostrum is a creamy white-to-yellow premilk fluid that can be expressed from the nipples before birth.

What should the nurse be cognizant of concerning the clients reordering of personal relationships during pregnancy? a. Because of the special motherhood bond, a womans relationship with her mother is even more important than with the father of the child. b. Nurses need not get involved in any sexual issues the couple has during pregnancy, particularly if they have trouble communicating them to each other. c. Women usually express two major relationship needs during pregnancy: feeling loved and valued and having the child accepted by the father. d. The womans sexual desire is likely to be highest in the first trimester because of the excitement and because intercourse is physically easier.

ANS: C Love and support help a woman feel better about her pregnancy. The most important person to the pregnant woman is usually the father of the child. Nurses can facilitate communication between partners about sexual matters if, as is common, they are nervous about expressing their worries and feelings to one another. The second trimester is the time when a womans sense of well-being, along with certain physical changes, increases her desire for sex. Sexual desire is down in the first and third trimesters.

The nurse who provides care to clients in labor must have a thorough understanding of the physiologic processes of maternal hypotension. Which outcome might occur if the interventions for maternal hypotension are inadequate? a.Early FHR decelerations b.Fetal arrhythmias c.Uteroplacental insufficiency d.Spontaneous rupture of membranes

ANS: C Low maternal blood pressure reduces placental blood flow during uterine contractions, resulting in fetal hypoxemia. Maternal hypotension does not result in early FHR decelerations nor is it associated with fetal arrhythmias. Spontaneous rupture of membranes is not a result of maternal hypotension.

What is the primary purpose for magnesium sulfate administration for clients with preeclampsia and eclampsia? a. To improve patellar reflexes and increase respiratory efficiency b. To shorten the duration of labor c. To prevent convulsions d. To prevent a boggy uterus and lessen lochial flow

ANS: C Magnesium sulfate is the drug of choice used to prevent convulsions, although it can generate other problems. Loss of patellar reflexes and respiratory depression are signs of magnesium toxicity. Magnesium sulfate can also increase the duration of labor. Women are at risk for a boggy uterus and heavy lochial flow as a result of magnesium sulfate therapy.

The nurse is preparing to administer methotrexate to the client. This hazardous drug is most often used for which obstetric complication? a.Complete hydatidiform mole b.Missed abortion c.Unruptured ectopic pregnancy d.Abruptio placentae

ANS: C Methotrexate is an effective nonsurgical treatment option for a hemodynamically stable woman whose ectopic pregnancy is unruptured and measures less than 4 cm in diameter. Methotrexate is not indicated or recommended as a treatment option for a complete hydatidiform mole, for a missed abortion, or for abruptio placentae.

Management of primary dysmenorrhea often requires a multifaceted approach. Which pharmacologic therapy provides optimal pain relief for this condition? a. Acetaminophen b. Oral contraceptive pills (OCPs) c. Nonsteroidal antiinflammatory drugs (NSAIDs) d. Aspirin

ANS: C NSAIDs have the strongest research results for pain relief. If one NSAID is not effective, then another one may provide relief. Approximately 80% of women find relief from these prostaglandin inhibitors. Preparations containing acetaminophen are less effective for dysmenorrhea because they lack the antiprostaglandin properties of NSAIDs. OCPs are a reasonable choice for women who also want birth control. The benefit of OCPs is the reduction of menstrual flow and irregularities. OCPs may be contraindicated for some women and have a number of potential side effects. NSAIDs are the drug of choice. However, if a woman is taking an NSAID, she should avoid taking aspirin as well.

The nurse observes a sudden increase in variability on the ERM tracing. Which class of medications may cause this finding? a.Narcotics b.Barbiturates c.Methamphetamines d.Tranquilizers

ANS: C Narcotics, barbiturates, and tranquilizers may be causes of decreased variability; whereas methamphetamines may cause increased variability.

The nurse is providing contraceptive instruction to a young couple who are eager to learn. The nurse should be cognizant of which information regarding the natural family planning method? a. The natural family planning method is the same as coitus interruptus or pulling out. b. This contraception method uses the calendar method to align the womans cycle with the natural phases of the moon. c. This practice is the only contraceptive method acceptable to the Roman Catholic Church. d. The natural family planning method relies on barrier methods during the fertility phases.

ANS: C Natural family planning is the only contraceptive practice acceptable to the Roman Catholic Church. Pulling out is not the same as periodic abstinence, another name for natural family planning. The phases of the moon are not part of the calendar method or any method. Natural family planning is another name for periodic abstinence, which is the accepted way to pass safely through the fertility phases without relying on chemical or physical barriers.

Women with mild gestational hypertension and mild preeclampsia can be safely managed at home with frequent maternal and fetal evaluation. Complete or partial bed rest is still frequently ordered by some providers. Which complication is rarely the result of prolonged bed rest? a. Thrombophlebitis b. Psychologic stress c. Fluid retention d. Cardiovascular deconditioning

ANS: C No evidence has been found that supports the practice of bed rest to improve pregnancy outcome. Fluid retention is not an adverse outcome of prolonged bed rest. The woman is more likely to experience diuresis with accompanying fluid and electrolyte imbalance and weight loss. Prolonged bed rest is known to increase the risk for thrombophlebitis. Psychologic stress is known to begin on the first day of bed rest and continue for the duration of the therapy. Therefore, restricted activity, rather than complete bed rest, is recommended. Cardiovascular deconditioning is a known complication of bed rest.

A client at 24 weeks of gestation says she has a glass of wine with dinner every evening. Why should the nurse counsel her to eliminate all alcohol intake? a. Daily consumption of alcohol indicates a risk for alcoholism. b. She is at risk for abusing other substances as well. c. Alcohol places the fetus at risk for altered brain growth. d. Alcohol places the fetus at risk for multiple organ anomalies.

ANS: C No period during pregnancy is safe to consume alcohol. The documented effects of alcohol consumption during pregnancy include fetal mental retardation, learning disabilities, high activity level, and short attention span. The fetal brain grows most rapidly in the third trimester and is vulnerable to alcohol exposure during this time. Abuse of other substances has not been linked to alcohol use.

A woman at 24 weeks of gestation states that she has a glass of wine with dinner every evening. Why would the nurse counsel the client to eliminate all alcohol? a. Daily consumption of alcohol indicates a risk for alcoholism. b. She will be at risk for abusing other substances as well. c. The fetus is placed at risk for altered brain growth. d. The fetus is at risk for multiple organ anomalies.

ANS: C No period exists when consuming alcohol during pregnancy is safe. The documented effects of alcohol consumption during pregnancy include mental retardation, learning disabilities, high activity level, and short attention span. The brain grows most rapidly in the third trimester and is vulnerable to alcohol exposure during this time. Abuse of other substances has not been linked to alcohol use.

Where is the point of maximal intensity (PMI) of the FHR located? a. Usually directly over the fetal abdomen b. In a vertex position, heard above the mother's umbilicus c. Heard lower and closer to the midline of the mother's abdomen as the fetus descends and internally rotates d. In a breech position, heard below the mother's umbilicus

ANS: C Nurses should be prepared for the shift. The PMI of the FHR is usually directly over the fetal back. In a vertex position, the PMI of the FHR is heard below the mother's umbilicus. In a breech position, it is heard above the mother's umbilicus.

The labor and delivery nurse is preparing a client who is severely obese (bariatric) for an elective cesarean birth. Which piece of specialized equipment will not likely be needed when providing care for this pregnant woman? a. Extra-long surgical instruments b. Wide surgical table c. Temporal thermometer d. Increased diameter blood pressure cuff

ANS: C Obstetricians today are seeing an increasing number of morbidly obese pregnant women weighing 400, 500, and 600 pounds. To manage their conditions and to meet their logistical needs, a new medical subspecialty,bariatric obstetrics, has arisen. Extra-wide blood pressure cuffs, scales that can accommodate up to 880 pounds, and extra-wide surgical tables designed to hold the weight of these women are used. Special techniques for ultrasound examination and longer surgical instruments for cesarean birth are also required. A temporal thermometer can be used for a pregnant client of any size.

What is the primary reason why a woman who is older than 35 years may have difficulty achieving pregnancy? a. Personal risk behaviors influence fertility. b. Mature women have often used contraceptives for an extended time. c. Her ovaries may be affected by the aging process. d. Prepregnancy medical attention is lacking.

ANS: C Once the mature woman decides to conceive, a delay in becoming pregnant may occur because of the normal aging of the ovaries. Older adults participate in fewer risk behaviors than younger adults. The past use of contraceptives is not the problem. Prepregnancy medical care is both available and encouraged.

The management of the pregnant client who has experienced a pregnancy loss depends on the type of miscarriage and the signs and symptoms. While planning care for a client who desires outpatient management after a first-trimester loss, what would the nurse expect the plan to include? a.Dilation and curettage (D&C) b.Dilation and evacuation (D&E) c.Misoprostol d.Ergot products

ANS: C Outpatient management of a first-trimester loss is safely accomplished by the intravaginal use of misoprostol for up to 2 days. If the bleeding is uncontrollable, vital signs are unstable, or signs of infection are present, then a surgical evacuation should be performed. D&C is a surgical procedure that requires dilation of the cervix and scraping of the uterine walls to remove the contents of pregnancy. This procedure is commonly performed to treat inevitable or incomplete abortion and should be performed in a hospital. D&E is usually performed after 16 weeks of pregnancy. The cervix is widely dilated, followed by removal of the contents of the uterus. Ergot products such as Methergine or Hemabate may be administered for excessive bleeding after miscarriage.

While providing care to the maternity client, the nurse should be aware that one of these anxiety disorders is likely to be triggered by the process of labor and birth. Which disorder fits this criterion? a. Phobias b. Panic disorder c. Posttraumatic stress disorder (PTSD) d. Obsessive-compulsive disorder (OCD)

ANS: C PTSD can occur as the result of a past trauma such as rape. Symptoms of PTSD include re-experiencing the event, numbing, irritability, angry outbursts, and exaggerated startle reflex. With the increased bodily touch and vaginal examinations that occur during labor, the client may have memories of the original trauma. The process of giving birth may result in her feeling out of control. The nurse should verbalize an understanding and reassure the client as necessary. Phobias are irrational fears that may lead a person to avoid certain events or situations. Panic disorders may occur in as many as 3% to 5% of women in the postpartum period and are described as episodes of intense apprehension, fear, and terror. Symptoms of a panic disorder may include palpitations, chest pain, choking, or smothering. OCD symptoms include recurrent, persistent, and intrusive thoughts. The mother may repeatedly check and recheck her infant once he or she is born, although she realizes that this behavior is irrational. OCD is optimally treated with medications.

Which statement regarding female sexual response is inaccurate? a. Women and men are more alike than different in their physiologic response to sexual arousal and orgasm. b. Vasocongestion is the congestion of blood vessels. c. Orgasmic phase is the final state of the sexual response cycle. d. Facial grimaces and spasms of the hands and feet are often part of arousal.

ANS: C The final state of the sexual response cycle is the resolution phase after orgasm. Men and women are surprisingly alike. Vasocongestion causes vaginal lubrication and engorgement of the genitals. Arousal is characterized by increased muscular tension (myotonia).

A woman who has recently given birth complains of pain and tenderness in her leg. On physical examination, the nurse notices warmth and redness over an enlarged, hardened area. Which condition should the nurse suspect, and how will it be confirmed? a.Disseminated intravascular coagulation (DIC); asking for laboratory tests b.von Willebrand disease (vWD); noting whether bleeding times have been extended c.Thrombophlebitis; using real-time and color Doppler ultrasound d.Idiopathic or immune thrombocytopenic purpura (ITP); drawing blood for laboratory analysis

ANS: C Pain and tenderness in the extremities, which show warmth, redness, and hardness, is likely thrombophlebitis. A Doppler ultrasound examination is a common noninvasive way to confirm the diagnosis. A diagnosis of DIC is made according to clinical findings and laboratory markers. With DIC, a physical examination will reveal symptoms that may include unusual bleeding, petechiae around a blood pressure cuff on the woman's arm, and/or excessive bleeding from the site of a slight trauma such as a venipuncture site. Symptoms of vWD, a type of hemophilia, include recurrent bleeding episodes, prolonged bleeding time, and factor VIII deficiency. A risk for PPH exists with vWD but does not exhibit a warm or reddened area in an extremity. ITP is an autoimmune disorder in which the life span of antiplatelet antibodies is decreased. Increased bleeding time is a diagnostic finding, and the risk of postpartum uterine bleeding is increased.

Which clinical finding in a primiparous client at 32 weeks of gestation might be an indication of anemia? a. Ptyalism b. Pyrosis c. Pica d. Decreased peristalsis

ANS: C Pica (a desire to eat nonfood substances) is an indication of iron deficiency and should be evaluated. Cravings include ice, clay, and laundry starch. Ptyalism (excessive salivation), pyrosis (heartburn), and decreased peristalsis are normal findings.

Preconception and prenatal care have become important components of womens health. What is the guiding principal of preconception care? a. Ensure that pregnancy complications do not occur. b. Identify the woman who should not become pregnant. c. Encourage healthy lifestyles for families desiring pregnancy. d. Ensure that women know about prenatal care.

ANS: C Preconception counseling guides couples in how to avoid unintended pregnancies, how to identify and manage risk factors in their lives and in their environment, and how to identify healthy behaviors that promote the well-being of the woman and her potential fetus. Preconception care does not ensure that pregnancy complications will not occur. In many cases, problems can be identified and treated and may not recur in subsequent pregnancies. For many women, counseling can allow behavior modification before any damage is done, or a woman can make an informed decision about her willingness to accept potential hazards. If a woman is seeking preconception care, then she is likely aware of prenatal care.

A number of metabolic changes occur throughout pregnancy. Which physiologic adaptation of pregnancy will influence the nurses plan of care? a. Insulin crosses the placenta to the fetus only in the first trimester, after which the fetus secretes its own. b. Women with insulin-dependent diabetes are prone to hyperglycemia during the first trimester because they are consuming more sugar. c. During the second and third trimesters, pregnancy exerts a diabetogenic effect that ensures an abundant supply of glucose for the fetus. d. Maternal insulin requirements steadily decline during pregnancy.

ANS: C Pregnant women develop increased insulin resistance during the second and third trimesters. Insulin never crosses the placenta; the fetus starts making its own around the 10th week. As a result of normal metabolic changes during pregnancy, insulin-dependent women are prone to hypoglycemia (low levels). Maternal insulin requirements may double or quadruple by the end of pregnancy.

Prostaglandin gel has been ordered for a pregnant woman at 43 weeks of gestation. What is the primary purpose of prostaglandin administration? a.To enhance uteroplacental perfusion in an aging placenta b.To increase amniotic fluid volume c.To ripen the cervix in preparation for labor induction d.To stimulate the amniotic membranes to rupture

ANS: C Preparations of prostaglandin E1 and E2 are effective when used before labor induction to ripen (i.e., soften and thin) the cervix. Uteroplacental perfusion is not altered by the use of prostaglandins. The insertion of prostaglandin gel has no effect on the level of amniotic fluid. In some cases, women will spontaneously begin laboring after the administration of prostaglandins, thereby eliminating the need for oxytocin. It is not common for a woman's membranes to rupture as a result of prostaglandin use.

In planning for home care of a woman with preterm labor, which concern should the nurse need to address? a.Nursing assessments are different from those performed in the hospital setting. b.Restricted activity and medications are necessary to prevent a recurrence of preterm labor. c.Prolonged bed rest may cause negative physiologic effects. d.Home health care providers are necessary

ANS: C Prolonged bed rest may cause adverse effects such as weight loss, loss of appetite, muscle wasting, weakness, bone demineralization, decreased cardiac output, risk for thrombophlebitis, alteration in bowel functions, sleep disturbance, and prolonged postpartum recovery. Nursing assessments differ somewhat from those performed in the acute care setting, but this concern does not need to be addressed. Restricted activity and medications may prevent preterm labor but not in all women. In addition, the plan of care is individualized to meet the needs of each client. Many women receive home health nurse visits, but care is individualized for each woman

Which information related to a prolonged deceleration is important for the labor nurse to understand? a.Prolonged decelerations present a continuing pattern of benign decelerations that do not require intervention. b.Prolonged decelerations constitute a baseline change when they last longer than 5 minutes. c.A disruption to the fetal oxygen supply causes prolonged decelerations. d.Prolonged decelerations require the customary fetal monitoring by the nurse.

ANS: C Prolonged decelerations are caused by a disruption in the fetal oxygen supply. They usually begin as a reflex response to hypoxia. If the disruption continues, then the fetal cardiac tissue, itself, will become hypoxic, resulting in direct myocardial depression of the FHR. Prolonged decelerations can be caused by prolonged cord compression, uteroplacental insufficiency, or perhaps sustained head compression. Prolonged decelerations lasting longer than 10 minutes are considered a baseline change that may require intervention. A prolonged deceleration is a visually apparent decrease (may be either gradual or abrupt) in the FHR of at least 15 beats per minute below the baseline and lasting longer than 2 minutes but shorter than 10 minutes. Nurses should immediately notify the physician or nurse-midwife and initiate appropriate treatment of abnormal patterns when they see prolonged decelerations.

A pregnant woman has been receiving a magnesium sulfate infusion for treatment of severe preeclampsia for 24 hours. On assessment, the nurse finds the following vital signs: temperature 37.3 C, pulse rate 88 beats per minute, respiratory rate 10 breaths per minute, BP 148/90 mm Hg, absent deep tendon reflexes (DTRs), and no ankle clonus. The client complains, Im so thirsty and warm. What is the nurses immediate action? a. To call for an immediate magnesium sulfate level b. To administer oxygen c. To discontinue the magnesium sulfate infusion d. To prepare to administer hydralazine

ANS: C Regardless of the magnesium level, the client is displaying the clinical signs and symptoms of magnesium toxicity. The first action by the nurse should be to discontinue the infusion of magnesium sulfate. In addition, calcium gluconate, the antidote for magnesium, may be administered. Hydralazine is an antihypertensive drug commonly used to treat hypertension in severe preeclampsia. Typically, hydralazine is administered for a systolic BP higher than 160 mm Hg or a diastolic BP higher than 110 mm Hg.

1. Which statement by the client will assist the nurse in determining whether she is in true labor as opposed to false labor? a. "I passed some thick, pink mucus when I urinated this morning." b. "My bag of waters just broke." c. "The contractions in my uterus are getting stronger and closer together." d. "My baby dropped, and I have to urinate more frequently now."

ANS: C Regular, strong contractions with the presence of cervical change indicate that the woman is experiencing true labor. The loss of the mucous plug (operculum) often occurs during the first stage of labor or before the onset of labor, but it is not the indicator of true labor. Spontaneous rupture of membranes (ROM) often occurs during the first stage of labor, but it is not the indicator of true labor. The presenting part of the fetus typically becomes engaged in the pelvis at the onset of labor, but this is not the indicator of true labor.

Which substance used during pregnancy causes vasoconstriction and decreased placental perfusion, resulting in maternal and neonatal complications? a. Alcohol b. Caffeine c. Tobacco d. Chocolate

ANS: C Smoking in pregnancy is known to cause a decrease in placental perfusion and is the cause of low-birth-weight infants. Prenatal alcohol exposure is the single greatest preventable cause of mental retardation. Alcohol use during pregnancy can cause high blood pressure, miscarriage, premature birth, stillbirth, and anemia. Caffeine may interfere with certain medications and worsen arrhythmias. Chocolate, particularly dark chocolate, contains caffeine that may interfere with certain medications.

A woman who is 30 weeks of gestation arrives at the hospital with bleeding. Which differential diagnosis would not be applicable for this client? a.Placenta previa b.Abruptio placentae c.Spontaneous abortion d.Cord insertion

ANS: C Spontaneous abortion is another name for miscarriage; it occurs, by definition, early in pregnancy. Placenta previa is a well-known reason for bleeding late in pregnancy. The premature separation of the placenta (abruptio placentae) is a bleeding disorder that can occur late in pregnancy. Cord insertion may cause a bleeding disorder that can also occur late in pregnancy.

The most effective and least expensive treatment of puerperal infection is prevention. What is the most important strategy for the nurse to adopt? a.Large doses of vitamin C during pregnancy b.Prophylactic antibiotics c.Strict aseptic technique, including hand washing, by all health care personnel d.Limited protein and fat intake

ANS: C Strict adherence by all health care personnel to aseptic techniques during childbirth and the postpartum period is extremely important and the least expensive measure to prevent infection. Good nutrition to control anemia is a preventive measure. Increased iron intake assists in preventing anemia. Antibiotics may be administered to manage infections; they are not a cost-effective measure to prevent postpartum infection. Limiting protein and fat intake does not help prevent anemia or prevent infection.

A 30-year-old gravida 3, para 2-0-0-2 is at 18 weeks of gestation. Which screening test should the nurse recommend be ordered for this client? a. BPP b. Chorionic villi sampling c. MSAFP screening d. Screening for diabetes mellitus

ANS: C The biochemical assessment MSAFP test is performed from week 15 to week 20 of gestation (weeks 16 to 18 are ideal). A BPP is a method of biophysical assessment of fetal well-being in the third trimester. Chorionic villi sampling is a biochemical assessment of the fetus that should be performed from the 10th to 12th weeks of gestation. Screening for diabetes mellitus begins with the first prenatal visit.

A womans cousin gave birth to an infant with a congenital heart anomaly. The woman asks the nurse when such anomalies occur during development. Which response by the nurse is most accurate? a. We dont really know when such defects occur. b. It depends on what caused the defect. c. Defects occur between the third and fifth weeks of development. d. They usually occur in the first 2 weeks of development.

ANS: C The cardiovascular system is the first organ system to function in the developing human. Blood vessel and blood formation begins in the third week, and the heart is developmentally complete in the fifth week. We dont really know when such defects occur is an inaccurate statement. Regardless of the cause, the heart is vulnerable during its period of developmentin the third to fifth weeks; therefore, the statement, They usually occur in the first 2 weeks of development is inaccurate.

A primigravida at 40 weeks of gestation is having uterine contractions every to 2 minutes and states that they are very painful. Her cervix is dilated 2 cm and has not changed in 3 hours. The woman is crying and wants an epidural. What is the likely status of this woman's labor? a.She is exhibiting hypotonic uterine dysfunction. b.She is experiencing a normal latent stage. c.She is exhibiting hypertonic uterine dysfunction. d.She is experiencing precipitous labor.

ANS: C The contraction pattern observed in this woman signifies hypertonic uterine activity. Typically, uterine activity in this phase occurs at 4- to 5-minute intervals lasting 30 to 45 seconds. Women who experience hypertonic uterine dysfunction, or primary dysfunctional labor, are often anxious first-time mothers who are having painful and frequent contractions that are ineffective at causing cervical dilation or effacement to progress. With hypotonic uterine dysfunction, the woman initially makes normal progress into the active stage of labor; then the contractions become weak and inefficient or stop altogether. Precipitous labor is one that lasts less than 3 hours from the onset of contractions until time of birth.

A woman in labor has just received an epidural block. What is the most important nursing intervention at this time? a.Limit parenteral fluids. b.Monitor the fetus for possible tachycardia. c.Monitor the maternal blood pressure for possible hypotension. d.Monitor the maternal pulse for possible bradycardia.

ANS: C The most important nursing intervention for a woman who has received an epidural block is for the nurse to monitor the maternal blood pressure frequently for signs of hypotension. IV fluids are increased for a woman receiving an epidural to prevent hypotension. The nurse also observes for signs of fetal bradycardia and monitors for signs of maternal tachycardia, secondary to hypotension.

The uterine contractions of a woman early in the active phase of labor are assessed by an internal uterine pressure catheter (IUPC). The uterine contractions occur every 3 to 4 minutes and last an average of 55 to 60 seconds. They are becoming more regular and are moderate to strong. Based on this information, what would a prudent nurse do next? a. Immediately notify the woman's primary health care provider. b. Prepare to administer an oxytocic to stimulate uterine activity. c. Document the findings because they reflect the expected contraction pattern for the active phase of labor. d. Prepare the woman for the onset of the second stage of labor.

ANS: C The nurse is responsible for monitoring the uterine contractions to ascertain whether they are powerful and frequent enough to accomplish the work of expelling the fetus and the placenta. In addition, the nurse documents these findings in the client's medical record. This labor pattern indicates that the client is in the active phase of the first stage of labor. Nothing indicates a need to notify the primary health care provider at this time. Oxytocin augmentation is not needed for this labor pattern; this contraction pattern indicates that the woman is in active labor. Her contractions will eventually become stronger, last longer, and come closer together during the transition phase of the first stage of labor. The transition phase precedes the second stage of labor, or delivery of the fetus.

Which action is correct when palpation is used to assess the characteristics and pattern of uterine contractions? a. Placing the hand on the abdomen below the umbilicus and palpating uterine tone with the fingertips b. Determining the frequency by timing from the end of one contraction to the end of the next contraction c. Evaluating the intensity by pressing the fingertips into the uterine fundus d. Assessing uterine contractions every 30 minutes throughout the first stage of labor

ANS: C The nurse or primary health care provider may assess uterine activity by palpating the fundal section of the uterus using the fingertips. Many women may experience labor pain in the lower segment of the uterus, which may be unrelated to the firmness of the contraction detectable in the uterine fundus. The frequency of uterine contractions is determined by palpating from the beginning of one contraction to the beginning of the next contraction. Assessment of uterine activity is performed in intervals based on the stage of labor. As labor progresses, this assessment is performed more frequently.

A client arrives for her initial prenatal examination. This is her first child. She asks the nurse, How does my baby get air inside my uterus? What is the correct response by the nurse? a. The babys lungs work in utero to exchange oxygen and carbon dioxide. b. The baby absorbs oxygen from your blood system. c. The placenta provides oxygen to the baby and excretes carbon dioxide into your bloodstream. d. The placenta delivers oxygen-rich blood through the umbilical artery to the babys abdomen.

ANS: C The placenta delivers oxygen-rich blood through the umbilical vein, not the artery, to the fetus and excretes carbon dioxide into the maternal bloodstream. The fetal lungs do not function as respiratory gas exchange in utero. The baby does not simply absorb oxygen from a womans blood system; rather, blood and gas transport occur through the placenta.

A woman has requested an epidural for her pain. She is 5 cm dilated and 100% effaced. The baby is in a vertex position and is engaged. The nurse increases the woman's IV fluid for a preprocedural bolus. The nurse reviews her laboratory values and notes that the woman's hemoglobin is 12 g/dl, hematocrit is 38%, platelets are 67,000, and white blood cells (WBCs) are 12,000/mm3. Which factor would contraindicate an epidural for this woman? a.She is too far dilated. b.She is anemic. c.She has thrombocytopenia. d.She is septic.

ANS: C The platelet count indicates a coagulopathy, specifically, thrombocytopenia (low platelets), which is a contraindication to epidural analgesia and anesthesia. Typically, epidural analgesia and anesthesia are used in the laboring woman when a regular labor pattern has been achieved, as evidenced by progressive cervical change. The laboratory values show that the woman's hemoglobin and hematocrit levels are in the normal range and show a slight increase in the WBC count that is not uncommon in laboring women.

Dental care during pregnancy is an important component of good prenatal care. Which instruction regarding dental health should the nurse provide? a. Regular brushing and flossing may not be necessary during early pregnancy because it may stimulate the woman who is already nauseated to vomit. A cleaning is all that is necessary. b. Dental surgery, in particular, is contraindicated during pregnancy and should be delayed until after delivery. c. If dental treatment is necessary, then the woman will be most comfortable with it in the second trimester. d. If a woman has dental anxiety, then dental care may interfere with the expectant mothers need to practice conscious relaxation and to prepare for labor.

ANS: C The second trimester is the best time for dental treatment because the woman will be able to sit most comfortably in the dental chair. Dental care, such as brushing with a fluoride toothpaste, is especially important during pregnancy. Periodontal disease has been linked to both preterm labor and low-birth-weight (LBW) infants. Emergency dental surgery is permissible; however, the mother must clearly understand the risks and benefits. Conscious relaxation is useful and may even help the woman get through any dental appointments, but it is not a reason to avoid them.

Which phase of the endometrial cycle best describes a heavy, velvety soft, fully matured endometrium? a. Menstrual b. Proliferative c. Secretory d. Ischemic

ANS: C The secretory phase extends from the day of ovulation to approximately 3 days before the next menstrual cycle. During this secretory phase, the endometrium becomes fully mature again. During the menstrual phase, the endometrium is shed. The proliferative phase is a period of rapid growth. During the ischemic phase, the blood supply is blocked and necrosis develops.

What is a distinct advantage of external EFM? a.The ultrasound transducer can accurately measure short-term variability and beat-to-beat changes in the FHR. b.The tocotransducer can measure and record the frequency, regularity, intensity, and approximate duration of uterine contractions. c.The tocotransducer is especially valuable for measuring uterine activity during the first stage of labor. d.Once correctly applied by the nurse, the transducer need not be repositioned even when the woman changes positions.

ANS: C The tocotransducer is valuable for measuring uterine activity during the first stage of labor and is especially true when the membranes are intact. Short-term variability and beat-to-beat changes cannot be measured with this technology. The tocotransducer cannot measure and record the intensity of uterine contractions. The transducer must be repositioned when the woman or the fetus changes position.

Which finding on a prenatal visit at 10 weeks of gestation might suggest a hydatidiform mole? a.Complaint of frequent mild nausea b.Blood pressure of 120/80 mm Hg c.Fundal height measurement of 18 cm d.History of bright red spotting for 1 day, weeks ago

ANS: C The uterus in a hydatidiform molar pregnancy is often larger than would be expected on the basis of the duration of the pregnancy. Nausea increases in a molar pregnancy because of the increased production of hCG. A woman with a molar pregnancy may have early-onset pregnancy-induced hypertension. In the client's history, bleeding is normally described as brownish.

10. A nulliparous woman has just begun the latent phase of the second stage of her labor. The nurse should anticipate which behavior? a. A nulliparous woman will experience a strong urge to bear down. b. Perineal bulging will show. c. A nulliparous woman will remain quiet with her eyes closed between contractions. d. The amount of bright red bloody show will increase.

ANS: C The woman is able to relax and close her eyes between contractions as the fetus passively descends. The woman may be very quiet during this phase. During the latent phase of the second stage of labor, the urge to bear down is often absent or only slight during the acme of the contractions. Perineal bulging occurs during the transition phase of the second stage of labor, not at the beginning of the second stage. An increase in bright red bloody show occurs during the descent phase of the second stage of labor.

After a mastectomy, which activity should the client be instructed to avoid? a.Emptying surgical drains twice a day and as needed b.Lifting more than 4.5 kg (10 lb) or reaching above her head until given permission by her surgeon c.Wearing clothing with snug sleeves to support the tissue of the arm on the operative side d.Immediately reporting inflammation that develops at the incision site or in the affected arm

ANS: C The woman should not be advised to wear snug clothing. She should be advised to avoid tight clothing, tight jewelry, and other apparel that might cause decreased circulation in the affected arm. As part of the teaching plan, the woman should be instructed to empty the surgical drains twice a day, to avoid lifting more than 4.5 kg (10 lb) or reaching above her head until given permission by her surgeon, and to report immediately any inflammation that develops at the incision site or in the affected arm.

A 27-year-old pregnant woman had a preconceptual body mass index (BMI) of 19. What is this clients total recommended weight gain during pregnancy? a. 20 kg (44 lb) b. 16 kg (35 lb) c. 12.5 kg (27.5 lb) d. 10 kg (22 lb)

ANS: C This woman has a normal BMI and should gain 11.5 to 16 kg during her pregnancy. A weight gain of 20 kg (44 lb) is unhealthy for most women; a weight gain of 16 kg (35 lb) is at the high end of the range of weight this woman should gain in her pregnancy; and a weight gain of 10 kg (22 lb) is appropriate for an obese woman. This woman has a normal BMI, which indicates that her weight is average.

When a woman is diagnosed with postpartum depression (PPD) with psychotic features, what is the nurses primary concern in planning the clients care? a. Displaying outbursts of anger b. Neglecting her hygiene c. Harming her infant d. Losing interest in her husband

ANS: C Thoughts of harm to herself or to the infant are among the most serious symptoms of PPD and require immediate assessment and intervention. Although outbursts of anger and neglecting personal hygiene are symptoms attributable to PPD, the major concern remains the potential of harm to herself or her infant. Although this client is likely to lose interest in her spouse, it is not the nurses primary concern.

The American College of Obstetricians and Gynecologists (ACOG) has developed a comprehensive list of risk factors associated with the development of preeclampsia. Which client exhibits the greatest number of these risk factors? a. 30-year-old obese Caucasian with her third pregnancy b. 41-year-old Caucasian primigravida c. 19-year-old African American who is pregnant with twins d. 25-year-old Asian American whose pregnancy is the result of donor insemination

ANS: C Three risk factors are present in the 19-year-old African-American client. She has African-American ethnicity, is at the young end of the age distribution, and has a multiple pregnancy. In planning care for this client, the nurse must frequently monitor her BP and teach her to recognize the early warning signs of preeclampsia. The 30-year-old obese Caucasian client has only has one known risk factor: obesity. Age distribution appears to be U-shaped, with women younger than 20 years of age and women older than 40 years of age being at greatest risk. Preeclampsia continues to be more frequently observed in primigravidas; this client is a multigravida woman. Two risk factors are present for the 41-year-old Caucasian primigravida client. Her age and status as a primigravida place her at increased risk for preeclampsia. Caucasian women are at a lower risk than are African-American women. The 25-year-old Asian-American client exhibits only one risk factor. Pregnancies that result from donor insemination, oocyte donation, and embryo donation are at an increased risk of developing preeclampsia.

Which adaptation of the maternal-fetal exchange of oxygen occurs in response to uterine contraction? a. The maternal-fetal exchange of oxygen and waste products continues except when placental functions are reduced. b. This maternal-fetal exchange increases as the blood pressure decreases. c. It diminishes as the spiral arteries are compressed. d. This exchange of oxygen and waste products is not significantly affected by contractions.

ANS: C Uterine contractions during labor tend to decrease circulation through the spiral electrodes and subsequent perfusion through the intervillous space. The maternal blood supply to the placenta gradually stops with contractions. The exchange of oxygen and waste products decreases. The exchange of oxygen and waste products is affected by contractions.

A woman will be taking oral contraceptives using a 28-day pack. What advice should the nurse provide to protect this client from an unintended pregnancy? a. Limit sexual contact for one cycle after starting the pill. b. Use condoms and foam instead of the pill for as long as the client takes an antibiotic. c. Take one pill at the same time every day. d. Throw away the pack and use a backup method if two pills are missed during week 1 of her cycle.

ANS: C To maintain adequate hormone levels for contraception and to enhance compliance, clients should take oral contraceptives at the same time each day. If contraceptives are to be started at any time other than during normal menses or within 3 weeks after birth or an abortion, then another method of contraception should be used through the first week to prevent the risk of pregnancy. Taken exactly as directed, oral contraceptives prevent ovulation, and pregnancy cannot occur. No strong pharmacokinetic evidence indicates a link between the use of broad-spectrum antibiotics and altered hormonal levels in oral contraceptive users. If the client misses two pills during week 1, then she should take two pills a day for 2 days and finish the package and use a backup contraceptive method for the next 7 consecutive days.

The nurse provides education to a client about to undergo external radiation therapy. Which statement by the client reassures the nurse that the teaching has been effective? a."I am using ointment to keep my skin from drying out." b."I wash the irradiated area with deodorant soap." c."My diet is high in protein, and I drink at least 2000 ml of fluid a day." d."I wash off the markings for the radiation site after each treatment."

ANS: C To maintain good nutrition, the woman should eat high-protein meals or use protein supplements and should have a high daily fluid intake of 2 to 3 L. The woman is counseled about good skin care and taught to avoid soaps, ointments, cosmetics, and deodorants because these may contain metals that would alter the radiation dose she receives. Markings may be made to indicate the exact location needed for irradiation and should remain until the treatment is complete.

A pregnant woman at 10 weeks of gestation jogs three or four times per week. She is concerned about the effect of the exercise on the fetus. Which guidance should the nurse provide? a. You dont need to modify your exercising any time during your pregnancy. b. Stop exercising because it will harm the fetus. c. You may find that you need to modify your exercise to walking later in your pregnancy, around the seventh month. d. Jogging is too hard on your joints; switch to walking now.

ANS: C Typically, running should be replaced with walking around the seventh month of pregnancy. The nurse should inform the woman that she may need to reduce her exercise level as the pregnancy progresses. Physical activity promotes a feeling of well-being in pregnant women. It improves circulation, promotes relaxation and rest, and counteracts boredom. Simple measures should be initiated to prevent injuries, such as warm-up and stretching exercises to prepare the joints for more strenuous exercise.

Which statement related to cephalopelvic disproportion (CPD) is the least accurate? a.CPD can be related to either fetal size or fetal position. b.The fetus cannot be born vaginally. c.CPD can be accurately predicted. d.Causes of CPD may have maternal or fetal origins.

ANS: C Unfortunately, accurately predicting CPD is not possible. Although CPD is often related to excessive fetal size (macrosomia), malposition of the fetal presenting part is the problem in many cases, not true CPD. When CPD is present, the fetus cannot fit through the maternal pelvis to be born vaginally. CPD may be related to either fetal origins such as macrosomia or malposition or maternal origins such as a too small or malformed pelvis.

Which diagnostic test is used to confirm a suspected diagnosis of breast cancer? a.Mammogram b.Ultrasound c.Needle-localization biopsy d.Magnetic resonance imaging (MRI)

ANS: C When a suspicious mammogram is noted or a lump is detected, diagnosis is confirmed by either a core-needle biopsy or a needle-localization biopsy. Mammography is a clinical screening tool that may aid in the early detection of breast cancers. Transillumination, thermography, and ultrasound breast imaging are being explored as methods for detecting early breast carcinoma. An MRI is useful in women with masses that are difficult to find (occult breast cancer).

A woman is having her first child. She has been in labor for 15 hours. A vaginal examination performed 2 hours earlier revealed the cervix to be dilated to 5 cm and 100% effaced, and the presenting part of the fetus was at station 0; however, another vaginal examination performed 5 minutes ago indicated no changes. What abnormal labor pattern is associated with this description? a.Prolonged latent phase b.Protracted active phase c.Secondary arrest d.Protracted descent

ANS: C With a secondary arrest of the active phase, the progress of labor has stopped. This client has not had any anticipated cervical change, indicating an arrest of labor. In the nulliparous woman, a prolonged latent phase typically lasts longer than 20 hours. A protracted active phase, the first or second stage of labor, is prolonged (slow dilation). With a protracted descent, the fetus fails to descend at an anticipated rate during the deceleration phase and second stage of labor.

Which nursing assessment indicates that a woman who is in second-stage labor is almost ready to give birth? a. Fetal head is felt at 0 station during the vaginal examination. b. Bloody mucous discharge increases. c. Vulva bulges and encircles the fetal head. d. Membranes rupture during a contraction.

ANS: C During the active pushing (descent) phase, the woman has strong urges to bear down as the presenting part of the fetus descends and presses on the stretch receptors of the pelvic floor. The vulva stretches and begins to bulge, encircling the fetal head. Birth of the head occurs when the station is +4. A 0 station indicates engagement. Bloody show occurs throughout the labor process and is not an indication of an imminent birth. Rupture of membranes can occur at any time during the labor process and does not indicate an imminent birth.

Which important component of nutritional counseling should the nurse include in health teaching for a pregnant woman who is experiencing cholecystitis? a. Assess the woman's dietary history for adequate calories and proteins. b. Teach the woman that the bulk of calories should come from proteins. c. Instruct the woman to eat a low-fat diet and to avoid fried foods. d. Instruct the woman to eat a low-cholesterol, low-salt diet.

ANS: C Eating a low-fat diet and avoiding fried foods is appropriate nutritional counseling for this client. Caloric and protein intake do not predispose a woman to the development of cholecystitis. The woman should be instructed to limit protein intake and choose foods that are high in carbohydrates. A low-cholesterol diet may be the result of limiting fats. However, a low-salt diet is not indicated.

A woman at 28 weeks of gestation experiences blunt abdominal trauma as the result of a fall. The nurse must closely observe the client for what? a. Alteration in maternal vital signs, especially blood pressure b. Complaints of abdominal pain c. Placental absorption d. Hemorrhage

ANS: C Electronic fetal monitoring (EFM) tracings can help evaluate maternal status after trauma and can reflect fetal cardiac responses to hypoxia and hypoperfusion. Signs and symptoms of placental absorption include uterine irritability, contractions, vaginal bleeding, and changes in FHR characteristics. Hypoperfusion may be present in the pregnant woman before the onset of clinical signs of shock. EFM tracings show the first signs of maternal compromise, such as when the maternal heart rate, blood pressure, and color appear normal, yet the EFM printout shows signs of fetal hypoxia. Abdominal pain, in and of itself, is not the most important symptom. However, if it is accompanied by contractions, changes in the FHR, rupture of membranes, or vaginal bleeding, then the client should be evaluated for abruptio placentae. Clinical signs of hemorrhage do not appear until after a 30% loss of circulating volume occurs. Careful monitoring of fetal status significantly assists in maternal assessment, because the fetal monitor tracing works as an oximeter of internal well-being.

When assessing the fetus using Leopold's maneuvers, the nurse feels a round, firm, and movable fetal part in the fundal portion of the uterus and a long, smooth surface in the mother's right side close to midline. What is the position of the fetus? a. ROA b. LSP c. RSA d. LOA

ANS: C Fetal position is denoted with a three-letter abbreviation. The first letter indicates the presenting part in either the right or the left side of the maternal pelvis. The second letter indicates the anatomic presenting part of the fetus. The third letter stands for the location of the presenting part in relationship to the anterior, posterior, or transverse portion of the maternal pelvis. Palpation of a round, firm fetal part in the fundal portion of the uterus would be the fetal head, indicating that the fetus is in a breech position with the sacrum as the presenting part in the maternal pelvis. Palpation of the fetal spine along the mother's right side denotes the location of the presenting part in the mother's pelvis. The ability to palpate the fetal spine indicates that the fetus is anteriorly positioned in the maternal pelvis. This fetus is anteriorly positioned in the right side of the maternal pelvis with the sacrum as the presenting part. RSA is the correct three-letter abbreviation to indicate this fetal position. ROA denotes a fetus that is anteriorly positioned in the right side of the maternal pelvis with the occiput as the presenting part. LSP describes a fetus that is posteriorly positioned in the left side of the pelvis with the sacrum as the presenting part. A fetus that is LOA would be anteriorly positioned in the left side of the pelvis with the occiput as the presenting part.

A pregnant woman at term is transported to the emergency department (ED) after a severe vehicular accident. The obstetric nurse responds and rushes to the ED with a fetal monitor. Cardiopulmonary arrest occurs as the obstetric nurse arrives. What is the highest priority for the trauma team? a. Obtaining IV access, and starting aggressive fluid resuscitation b. Quickly applying the fetal monitor to determine whether the fetus viability c. Starting cardiopulmonary resuscitation (CPR) d. Transferring the woman to the surgical unit for an emergency cesarean delivery in case the fetus is still alive

ANS: C In a situation of severe maternal trauma, the systematic evaluation begins with a primary survey and the initial ABCs (airway, breathing, and circulation) of resuscitation. CPR is initiated first, followed by intravenous (IV) replacement fluid. After immediate resuscitation and successful stabilization measures, a more detailed secondary survey of the mother and fetus should be accomplished. Attempts at maternal resuscitation are made, followed by a secondary survey of the fetus. In the presence of multisystem trauma, a cesarean delivery may be indicated to increase the chance for maternal survival.

Which statement by the client would lead the nurse to believe that labor has been established? a. "I passed some thick, pink mucus when I urinated this morning." b. "My bag of waters just broke." c. "The contractions in my uterus are getting stronger and closer together." d. "My baby dropped, and I have to urinate more frequently now."

ANS: C Regular, strong contractions with the presence of cervical change indicate that the woman is experiencing true labor. Although the loss of the mucous plug (operculum) often occurs during the first stage of labor or before the onset of labor, it is not the indicator of true labor. Spontaneous rupture of membranes often occurs during the first stage of labor; however, it is not an indicator of true labor. The presenting part of the fetus typically

A woman has experienced iron deficiency anemia during her pregnancy. She had been taking iron for 3 months before the birth. The client gave birth by cesarean 2 days earlier and has been having problems with constipation. After assisting her back to bed from the bathroom, the nurse notes that the woman's stools are dark (greenish-black). What should the nurse's initial action be? a. Perform a guaiac test, and record the results. b. Recognize the finding as abnormal, and report it to the primary health care provider. c. Recognize the finding as a normal result of iron therapy. d. Check the woman's next stool to validate the observation.

ANS: C The nurse should recognize that dark stools are a common side effect in clients who are taking iron replacement therapy. A guaiac test would be indicated if gastrointestinal (GI) bleeding was suspected. GI irritation, including dark stools, is also a common side effect of iron therapy. Observation of stool formation is a normal nursing activity.

Nurses should be cognizant of what regarding the mechanism of labor? a. Seven critical movements must progress in a more or less orderly sequence. b. Asynclitism is sometimes achieved by means of the Leopold's maneuver. c. Effects of the forces determining descent are modified by the shape of the woman's pelvis and the size of the fetal head. d. At birth, the baby is said to achieve "restitution"; that is, a return to the C-shape of the womb.

ANS: C The size of the maternal pelvis and the ability of the fetal head to mold also affect the process. The seven identifiable movements of the mechanism of labor simultaneously occur in combinations, not in precise sequences. Asynclitism is the deflection of the baby's head; the Leopold's maneuver is a means of judging descent by palpating the mother's abdomen. Restitution is the rotation of the baby's head after the infant is born.

Reports have linked third trimester use of selective serotonin uptake inhibitors (SSRIs) with a constellation of neonatal signs. The nurse is about to perform an assessment on the infant of a mother with a history of a mood disorder. Which signs and symptoms in the neonate may be the result of maternal SSRI use? (Select all that apply.) a. Hypotonia b. Hyperglycemia c. Shivering d. Fever e. Irritability

ANS: C, D, E Neonatal signs of maternal SSRI use include continuous crying, irritability, jitteriness, shivering, fever, hypertonia, respiratory distress, feeding difficulty, hypoglycemia, and seizures. The onset of signs and symptoms ranges from several hours to several days after birth, but the signs generally resolve within 2 weeks.

During pregnancy, many changes occur as a direct result of the presence of the fetus. Which of these adaptations meet this criterion? (Select all that apply.) a. Leukorrhea b. Development of the operculum c. Quickening d. Ballottement e. Lightening

ANS: C, D, E Quickening is the first recognition of fetal movements or feeling life. Quickening is often described as a flutter and is felt earlier in the multiparous woman than in the primiparous woman. Passive movement of the unengaged fetus is referred to as ballottement. Lightening occurs when the fetus begins to descend into the pelvis and occurs 2 weeks before labor in the nulliparous woman and at the start of labor in the multiparous woman. Leukorrhea is a white or slightly gray vaginal discharge that develops in response to cervical stimulation by estrogen and progesterone. Mucus fills the cervical canal creating a plug otherwise known as the operculum. The operculum acts as a barrier against bacterial invasion during the pregnancy.

Hypothyroidism occurs in 2 to 3 pregnancies per 1000. Because severe hypothyroidism is associated with infertility and miscarriage, it is not often seen in pregnancy. Regardless of this fact, the nurse should be aware of the characteristic symptoms of hypothyroidism. Which do they include? (Select all that apply.) a. Hot flashes b. Weight loss c. Lethargy d. Decrease in exercise capacity e. Cold intolerance

ANS: C, D, E Symptoms include weight gain, lethargy, decrease in exercise capacity, and intolerance to cold. Other presentations might include constipation, hoarseness, hair loss, and dry skin. Thyroid supplements are used to treat hyperthyroidism in pregnancy.

The baseline FHR is the average rate during a 10-minute segment. Changes in FHR are categorized as periodic or episodic. These patterns include both accelerations and decelerations. The labor nurse is evaluating the client's most recent 10-minute segment on the monitor strip and notes a late deceleration. Which is likely to have caused this change? (Select all that apply.) a.Spontaneous fetal movement b.Compression of the fetal head c.Placental abruption d.Cord around the baby's neck e.Maternal supine hypotension

ANS: C, E Late decelerations are almost always caused by uteroplacental insufficiency. Insufficiency is caused by uterine tachysystole, maternal hypotension, epidural or spinal anesthesia, IUGR, intraamniotic infection, or placental abruption. Spontaneous fetal movement, vaginal examination, fetal scalp stimulation, fetal reaction to external sounds, uterine contractions, fundal pressure, and abdominal palpation are all likely to cause accelerations of the FHR. Early decelerations are most often the result of fetal head compression and may be caused by uterine contractions, fundal pressure, vaginal examination, and the placement of an internal electrode. A variable deceleration is likely caused by umbilical cord compression, which may happen when the umbilical cord is around the baby's neck, arm, leg, or other body part or when a short cord, a knot in the cord, or a prolapsed cord is present.

A woman has come to the clinic for preconception counseling because she wants to start trying to get pregnant. Which guidance should she expect to receive? a. Discontinue all contraception now. b. Lose weight so that you can gain more during pregnancy. c. You may take any medications you have been regularly taking. d. Make sure you include adequate folic acid in your diet.

ANS: D A healthy diet before conception is the best way to ensure that adequate nutrients are available for the developing fetus. A womans folate or folic acid intake is of particular concern in the periconception period. Neural tube defects are more common in infants of women with a poor folic acid intake. Depending on the type of contraception that she has been using, discontinuing all contraception at this time may not be appropriate. Advising this client to lose weight now so that she can gain more during pregnancy is also not appropriate advice. Depending on the type of medications the woman is taking, continuing to take them regularly may not be appropriate.

Which pregnant woman should strictly follow weight gain recommendations during pregnancy? a. Pregnant with twins b. In early adolescence c. Shorter than 62 inches or 157 cm d. Was 20 pounds overweight before pregnancy

ANS: D A weight gain of 5 to 9 kg will provide sufficient nutrients for the fetus. Overweight and obese women should be advised to lose weight before conception to achieve the best pregnancy outcomes. A higher weight gain in twin gestations may help prevent low birth weights. Adolescents need to gain weight toward the higher acceptable range, which provides for their own growth, as well as for fetal growth. In the past, women of short stature were advised to restrict their weight gain; however, evidence to support these guidelines has not been found.

What is the most basic information that a nurse should be able to share with a client who asks about the process of conception? a. Ova are considered fertile 48 to 72 hours after ovulation. b. Sperm remain viable in the womans reproductive system for an average of 12 to 24 hours. c. Conception is achieved when a sperm successfully penetrates the membrane surrounding the ovum. d. Implantation in the endometrium occurs 6 to 10 days after conception.

ANS: D After implantation, the endometrium is called the decidua. Ova are considered fertile for approximately 24 hours after ovulation. Sperm remain viable in the womans reproductive system for an average of 2 to 3 days. Penetration of the ovum by the sperm is called fertilization. Conception occurs when the zygote, the first cell of the new individual, is formed.

If nonsurgical treatment for late PPH is ineffective, which surgical procedure would be appropriate to correct the cause of this condition? a.Hysterectomy b.Laparoscopy c.Laparotomy d.Dilation and curettage (D&C)

ANS: D D&C allows the examination of the uterine contents and the removal of any retained placental fragments or blood clots. Hysterectomy is the removal of the uterus and is not the appropriate treatment for late PPH. A laparoscopy is the insertion of an endoscope through the abdominal wall to examine the peritoneal cavity, but it, too, is not the appropriate treatment for this condition. A laparotomy is the surgical incision into the peritoneal cavity to explore it but is also not the appropriate treatment for late PPH.

In her work with pregnant women of different cultures, a nurse practitioner has observed various practices that seemed unfamiliar. The nurse practitioner has learned that cultural rituals and practices during pregnancy seem to have one purpose in common. Which statement best describes that purpose? a. To promote family unity b. To ward off the evil eye c. To appease the gods of fertility d. To protect the mother and fetus during pregnancy

ANS: D Although many cultures consider pregnancy normal, certain practices are expected of women of all cultures to ensure a good outcome. Cultural prescriptions tell women what to do, and cultural proscriptions establish taboos. The purposes of these practices are to prevent maternal illness resulting from a pregnancy-induced imbalanced state and to protect the vulnerable fetus. Promoting family unity is important, although not usually the premise for cultural rituals and practices. Warding off the evil eye may be specific to one particular culture; however, it is not the primary purpose of these practices. Appeasing the gods of fertility is not the impetus behind cultural rituals.

Who is most likely to experience the phenomenon of someone other than the mother-to-be having pregnancy-like symptoms such as nausea and weight gain? a. Mother of the pregnant woman b. Couples teenage daughter c. Sister of the pregnant woman d. Expectant father

ANS: D An expectant fathers experiencing of his partners pregnancy-like symptoms is called the couvade syndrome. The mother of the pregnant woman is unlikely to experience this phenomenon. She may be excited about becoming a grandmother or see her daughters pregnancy as a reminder that she is getting old. A couples teenage daughter is usually preoccupied with her own sexual development and may have difficulty accepting the overwhelming evidence of her parents sexual activity. It is the father of the pregnant woman, not the sister, who experiences these symptoms.

The obstetric nurse is preparing the client for an emergency cesarean birth, with no time to administer spinal anesthesia. The nurse is aware of and prepared for the greatest risk of administering general anesthesia to the client. What is this risk? a.Respiratory depression b.Uterine relaxation c.Inadequate muscle relaxation d.Aspiration of stomach contents

ANS: D Aspiration of acidic gastric contents with possible airway obstruction is a potentially fatal complication of general anesthesia. Respirations can be altered during general anesthesia, and the anesthesiologist will take precautions to maintain proper oxygenation. Uterine relaxation can occur with some anesthesia but can be monitored and prevented. Inadequate muscle relaxation can be improved with medication.

When a nulliparous woman telephones the hospital to report that she is in labor, what guidance should the nurse provide or information should the nurse obtain? a. Tell the woman to stay home until her membranes rupture. b. Emphasize that food and fluid intake should stop. c. Arrange for the woman to come to the hospital for labor evaluation. d. Ask the woman to describe why she believes she is in labor.

ANS: D Assessment begins at the first contact with the woman, whether by telephone or in person. By asking the woman to describe her signs and symptoms, the nurse can begin her assessment and gather data. The initial nursing activity should be to gather data about the woman's status. The amniotic membranes may or may not spontaneously rupture during labor. The client may be instructed to stay home until the uterine contractions become strong and regular. Before instructing the woman to come to the hospital, the nurse should initiate her assessment during the telephone interview. After this assessment has been made, the nurse may want to discuss the appropriate oral intake for early labor, such as light foods or clear liquids, depending on the preference of the client or her primary health care provider.

While assessing the vital signs of a pregnant woman in her third trimester, the client complains of feeling faint, dizzy, and agitated. Which nursing intervention is appropriate? a. Have the patient stand up, and then retake her BP. b. Have the patient sit down, and then hold her arm in a dependent position. c. Have the patient lie supine for 5 minutes, and then recheck her BP on both arms. d. Have the patient turn to her left side, and then recheck her BP in 5 minutes.

ANS: D BP is affected by maternal position during pregnancy. The supine position may cause occlusion of the vena cava and descending aorta. Turning the pregnant woman to a lateral recumbent position alleviates pressure on the blood vessels and quickly corrects supine hypotension. Pressures are significantly higher when the client is standing. This option causes an increase in systolic and diastolic pressures. The arm should be supported at the same level of the heart. The supine position may cause occlusion of the vena cava and descending aorta, creating hypotension.

A pregnant couple has formulated a birth plan and is reviewing it with the nurse at an expectant parents class. Which aspect of their birth plan should be considered potentially unrealistic and require further discussion with the nurse? a. My husband and I have agreed that my sister will be my coach because he becomes anxious with regard to medical procedures and blood. He will be nearby and check on me every so often to make sure everything is okay. b. We plan to use the techniques taught in the Lamaze classes to reduce the pain experienced during labor. c. We want the labor and birth to take place in a birthing room. My husband will come in the minute the baby is born. d. Regardless of the circumstances, we do not want the fetal monitor used during labor because it will interfere with movement and doing effleurage.

ANS: D Because monitoring is essential to assess fetal well-being, fetal monitoring is not a factor that can be determined by the couple. The nurse should fully explain its importance. The option for intermittent electronic monitoring could be explored if this is a low-risk pregnancy and as long as labor is normally progressing. The birth plan is a tool with which parents can explore their childbirth options; however, the plan must be viewed as tentative. Having the womans sister as her coach with her husband nearby is an acceptable request for a laboring woman. Using breathing techniques to alleviate pain is a realistic part of a birth plan. Not all fathers are able to be present during the birth; however, this couple has made a realistic plan that works for their specific situation.

Nurses should be aware of the strengths and limitations of various biochemical assessments during pregnancy. Which statement regarding monitoring techniques is the most accurate? a. Chorionic villus sampling (CVS) is becoming more popular because it provides early diagnosis. b. MSAFP screening is recommended only for women at risk for NTDs. c. PUBS is one of the triple-marker tests for Down syndrome. d. MSAFP is a screening tool only; it identifies candidates for more definitive diagnostic procedures.

ANS: D MSAFP is a screening tool, not a diagnostic tool. CVS provides a rapid result, but it is declining in popularity because of advances in noninvasive screening techniques. An MSAFP screening is recommended for all pregnant women. MSAFP screening, not PUBS, is part of the triple-marker tests for Down syndrome.

The nurse who elects to practice in the area of women's health must have a thorough understanding of miscarriage. Which statement regarding this condition is most accurate? a.A miscarriage is a natural pregnancy loss before labor begins. b.It occurs in fewer than 5% of all clinically recognized pregnancies. c.Careless maternal behavior, such as poor nutrition or excessive exercise, can be a factor in causing a miscarriage. d.If a miscarriage occurs before the 12th week of pregnancy, then it may be observed only as moderate discomfort and blood loss.

ANS: D Before the sixth week, the only evidence might be a heavy menstrual flow. After the 12th week, more severe pain, similar to that of labor, is likely. Miscarriage is a natural pregnancy loss, but it occurs, by definition, before 20 weeks of gestation, before the fetus is viable. Miscarriages occur in approximately 10% to 15% of all clinically recognized pregnancies. Miscarriages can be caused by a number of disorders or illnesses outside the mother's control or knowledge.

As part of their participation in the gynecologic portion of the physical examination, which approach should the nurse take? a. Take a firm approach that encourages the client to facilitate the examination by following the physicians instructions exactly. b. Explain the procedure as it unfolds, and continue to question the client to get information in a timely manner. c. Take the opportunity to explain that the trendy vulvar self-examination is only for women at risk for developing cancer. d. Help the woman relax through the proper placement of her hands and proper breathing during the examination.

ANS: D Breathing techniques are important relaxation techniques that can help the client during the examination. The nurse should encourage the client to participate in an active partnership with the health care provider. Explanations during the procedure are fine, but many women are uncomfortable answering questions in the exposed and awkward position of the examination. Vulvar self-examination on a regular basis should be encouraged and taught during the examination.

A woman who is pregnant for the first time is dilated 3 cm and having contractions every 5 minutes. She is groaning and perspiring excessively; she states that she did not attend childbirth classes. What is the optimal intervention for the nurse to provide at this time? a.Notify the woman's health care provider. b.Administer the prescribed narcotic analgesic. c.Assure her that her labor will be over soon. d.Assist her with simple breathing and relaxation instructions.

ANS: D By reducing tension and stress, both focusing and relaxation techniques will allow the woman in labor to rest and conserve energy for the task of giving birth. For those who have had no preparation, instruction in simple breathing and relaxation can be given in early labor and is often successful. The nurse can independently perform many functions in labor and birth, such as teaching and support. Pain medication may be an option for this client. However, the initial response of the nurse should include teaching the client about her options. The length of labor varies among individuals, but the first stage of labor is the longest. At 3 cm of dilation with contractions every 5 minutes, this woman has a significant amount of labor yet to experience.

A woman at 26 weeks of gestation is being assessed to determine whether she is experiencing preterm labor. Which finding indicates that preterm labor is occurring? a.Estriol is not found in maternal saliva. b.Irregular, mild uterine contractions are occurring every 12 to 15 minutes. c.Fetal fibronectin is present in vaginal secretions. d.The cervix is effacing and dilated to 2 cm.

ANS: D Cervical changes such as shortened endocervical length, effacement, and dilation are predictors of imminent preterm labor. Changes in the cervix accompanied by regular contractions indicate labor at any gestation. Estriol is a form of estrogen produced by the fetus that is present in plasma at 9 weeks of gestation. Levels of salivary estriol have been shown to increase before preterm birth. Irregular, mild contractions that do not cause cervical change are not considered a threat. The presence of fetal fibronectin in vaginal secretions between 24 and 36 weeks of gestation could predict preterm labor, but it has only a 20% to 40% positive predictive value. Of more importance are other physiologic clues of preterm labor such as cervical changes.

Which structure is responsible for oxygen and carbon dioxide transport to and from the maternal bloodstream? a. Decidua basalis b. Blastocyst c. Germ layer d. Chorionic villi

ANS: D Chorionic villi are fingerlike projections that develop out of the trophoblast and extend into the blood-filled spaces of the endometrium. The villi obtain oxygen and nutrients from the maternal bloodstream and dispose carbon dioxide and waste products into the maternal blood. The decidua basalis is the portion of the decidua (endometrium) under the blastocyst where the villi attach. The blastocyst is the embryonic development stage after the morula; implantation occurs at this stage. The germ layer is a layer of the blastocyst.

As a powerful central nervous system (CNS) stimulant, which of these substances can lead to miscarriage, preterm labor, placental separation (abruption), and stillbirth? a. Heroin b. Alcohol c. Phencyclidine (1-phenylcyclohexylpiperidine; PCP) d. Cocaine

ANS: D Cocaine is a powerful CNS stimulant. Effects on pregnancy associated with cocaine use include abruptio placentae, preterm labor, precipitous birth, and stillbirth. Heroin is an opiate; its use in pregnancy is associated with preeclampsia, intrauterine growth restriction, miscarriage, premature rupture of membranes, infections, breech presentation, and preterm labor. The most serious effect of alcohol use in pregnancy is FAS. The major concern regarding PCP use in pregnant women is its association with polydrug abuse and its neurobehavioral effects on the neonate.

To reassure and educate their pregnant clients regarding changes in their blood pressure, nurses should be cognizant of what? a. A blood pressure cuff that is too small produces a reading that is too low; a cuff that is too large produces a reading that is too high. b. Shifting the clients position and changing from arm to arm for different measurements produces the most accurate composite blood pressure reading at each visit. c. Systolic blood pressure slightly increases as the pregnancy advances; diastolic pressure remains constant. d. Compression of the iliac veins and inferior vena cava by the uterus contributes to hemorrhoids in the later stage of a term pregnancy.

ANS: D Compression of the iliac veins and inferior vena cava by the uterus contributes to hemorrhoids in the later stage of a term pregnancy. This compression also leads to varicose veins in the legs and vulva. The tightness of a blood pressure cuff that is too small produces a reading that is too high; similarly, the looseness of a cuff that is too large results in a reading that is too low. Because maternal positioning affects readings, blood pressure measurements should be obtained in the same arm and with the woman in the same position. The systolic blood pressure generally remains constant but may decline slightly as the pregnancy advances. The diastolic blood pressure first decreases and then gradually increases.

An 18-year-old pregnant woman, gravida 1, para 0, is admitted to the labor and birth unit with moderate contractions every 5 minutes that last 40 seconds. The client states, "My contractions are so strong, I don't know what to do." Before making a plan of care, what should the nurse's first action be? a.Assess for fetal well-being. b.Encourage the woman to lie on her side. c.Disturb the woman as little as possible. d.Recognize that pain is personalized for each individual.

ANS: D Each woman's pain during childbirth is unique and is influenced by a variety of physiologic, psychosocial, and environmental factors. A critical issue for the nurse is how support can make a difference in the pain of the woman during labor and birth. This scenario includes no information that would indicate fetal distress or a logical reason to be overly concerned about the well-being of the fetus. The left lateral position is used to alleviate fetal distress, not maternal stress. The nurse has an obligation to provide physical, emotional, and psychosocial care and support to the laboring woman. This client clearly needs support.

Nurses need to understand the basic definitions and incidence data regarding PPH. Which statement regarding this condition is most accurate? a.PPH is easy to recognize early; after all, the woman is bleeding. b.Traditionally, it takes more than 1000 ml of blood after vaginal birth and 2500 ml after cesarean birth to define the condition as PPH. c.If anything, nurses and physicians tend to overestimate the amount of blood loss. d.Traditionally, PPH has been classified as early PPH or late PPH with respect to birth.

ANS: D Early PPH is also known as primary, or acute, PPH; late PPH is known as secondary PPH. Unfortunately, PPH can occur with little warning and is often recognized only after the mother has profound symptoms. Traditionally, a 500-ml blood loss after a vaginal birth and a 1000-ml blood loss after a cesarean birth constitute PPH. Medical personnel tend to underestimate blood loss by as much as 50% in their subjective observations.

A woman in the 34th week of pregnancy reports that she is very uncomfortable because of heartburn. Which recommendation would be appropriate for this client? a. Substitute other calcium sources for milk in her diet. b. Lie down after each meal. c. Reduce the amount of fiber she consumes. d. Eat five small meals daily.

ANS: D Eating small, frequent meals may help with heartburn, nausea, and vomiting. Substituting other calcium sources for milk, lying down after eating, and reducing fiber intake are inappropriate dietary suggestions for all pregnant women and do not alleviate heartburn.

A woman is 3 months pregnant. At her prenatal visit she tells the nurse that she does not know what is happening; one minute she is happy that she is pregnant and the next minute she cries for no reason. Which response by the nurse is most appropriate? a. Dont worry about it; youll feel better in a month or so. b. Have you talked to your husband about how you feel? c. Perhaps you really dont want to be pregnant. d. Hormone changes during pregnancy commonly result in mood swings.

ANS: D Explaining that hormone changes can result in mood swings is an accurate statement and the most appropriate response by the nurse. Telling the woman not to worry dismisses her concerns and is not the most appropriate response. Although the woman should be encouraged to share her feelings, asking if she has spoken to her husband about them is not the most appropriate response and does not provide her with a rationale for the psychosocial dynamics of her pregnancy. Suggesting that the woman does not want to be pregnant is completely inappropriate and deleterious to the psychologic well-being of the woman. Hormonal and metabolic adaptations often cause mood swings in pregnancy. The womans responses are normal. She should be reassured about her feelings.

A health care provider performs a clinical breast examination on a woman diagnosed with fibroadenoma. How would the nurse explain the defining characteristics of a fibroadenoma? a.Inflammation of the milk ducts and glands behind the nipples b.Thick, sticky discharge from the nipple of the affected breast c.Lumpiness in both breasts that develops 1 week before menstruation d.Single lump in one breast that can be expected to shrink as the woman ages

ANS: D Fibroadenomas are characterized by discrete, usually solitary lumps smaller than 3 cm in diameter. Fibroadenomas increase in size during pregnancy and shrink as the woman ages. Inflammation of the milk ducts and glands behind the nipples is associated with mammary duct ectasia, not fibroadenoma. Thick, sticky discharge from the nipple of the affected breast is associated with galactorrhea, not fibroadenoma. Lumpiness in both breasts that develops 1 week before menstruation is associated with fibrocystic changes of the breast, not fibroadenoma.

What physiologic change occurs as the result of increasing the infusion rate of nonadditive IV fluids? a.Maintaining normal maternal temperature b.Preventing normal maternal hypoglycemia c.Increasing the oxygen-carrying capacity of the maternal blood d.Expanding maternal blood volume

ANS: D Filling the mother's vascular system increases the amount of blood available to perfuse the placenta and may correct hypotension. Increasing fluid volume may alter the maternal temperature only if she is dehydrated. Most IV fluids for laboring women are isotonic and do not provide extra glucose. Oxygen-carrying capacity is increased by adding more red blood cells.

The nurse is performing an assessment on a client who thinks she may be experiencing preterm labor. Which information is the most important for the nurse to understand and share with the client? a.Because all women must be considered at risk for preterm labor and prediction is so variable, teaching pregnant women the symptoms of preterm labor probably causes more harm through false alarms. b.Braxton Hicks contractions often signal the onset of preterm labor. c.Because preterm labor is likely to be the start of an extended labor, a woman with symptoms can wait several hours before contacting the primary caregiver. d.Diagnosis of preterm labor is based on gestational age, uterine activity, and progressive cervical change.

ANS: D Gestational age of 20 to 37 weeks, uterine contractions, and a cervix that is 80% effaced or dilated 2 cm indicates preterm labor. It is essential that nurses teach women how to detect the early symptoms of preterm labor. Braxton Hicks contractions resemble preterm labor contractions, but they are not true labor. Waiting too long to see a health care provider could result in essential medications failing to be administered. Preterm labor is not necessarily long-term labor.

Which system responses would the nurse recognize as being unrelated to prostaglandin (PGF2) release? a. Systemic responses b. Gastrointestinal system c. Central nervous system d. Genitourinary system

ANS: D Systemic responses to PGF2 include backache, weakness, and sweating. Gastrointestinal system changes include nausea, vomiting, anorexia, and diarrhea. Central nervous system changes manifest themselves as dizziness, syncope, headache, and poor concentration; they usually begin at the onset of menstruation and last 8 to 48 hours.

When a nurse is counseling a woman for primary dysmenorrhea, which nonpharmacologic intervention might be recommended? a. Increasing the intake of red meat to replace blood loss b. Reducing the intake of diuretic foods, such as peaches and asparagus c. Temporarily substituting physical activity for a sedentary lifestyle d. Using a heating pad on the abdomen to relieve cramping

ANS: D Heat minimizes cramping by increasing vasodilation and muscle relaxation and minimizing uterine ischemia. Dietary changes such as a lowfat vegetarian diet may be recommended for women experiencing dysmenorrhea. Increasing the intake of diuretics, including natural diuretics such as asparagus, cranberry juice, peaches, parsley, and watermelon, may help ease the symptoms associated with dysmenorrhea. Exercise has been found to help relieve menstrual discomfort through increased vasodilation and subsequent decreased ischemia.

The nurse suspects that her postpartum client is experiencing hemorrhagic shock. Which observation indicates or would confirm this diagnosis? a.Absence of cyanosis in the buccal mucosa b.Cool, dry skin c.Calm mental status d.Urinary output of at least 30 ml/hr

ANS: D Hemorrhage may result in hemorrhagic shock. Shock is an emergency situation during which the perfusion of body organs may become severely compromised, and death may occur. The presence of adequate urinary output indicates adequate tissue perfusion. The assessment of the buccal mucosa for cyanosis can be subjective. The presence of cool, pale, clammy skin is associated with hemorrhagic shock. Hemorrhagic shock is associated with lethargy, not restlessness.

The client has been on magnesium sulfate for 20 hours for the treatment of preeclampsia. She just delivered a viable infant girl 30 minutes ago. What uterine findings does the nurse expect to observe or assess in this client? a. Absence of uterine bleeding in the postpartum period b. Fundus firm below the level of the umbilicus c. Scant lochia flow d. Boggy uterus with heavy lochia flow

ANS: D High serum levels of magnesium can cause a relaxation of smooth muscle such as the uterus. Because of this tocolytic effect, the client will most likely have a boggy uterus with increased amounts of bleeding. All women experience uterine bleeding in the postpartum period, especially those who have received magnesium therapy. Rather than scant lochial flow, however, this client will most likely have a heavy flow attributable to the relaxation of the uterine wall caused by magnesium administration.

hCG is an important biochemical marker for pregnancy and therefore the basis for many tests. Which statement regarding hCG is true? a. hCG can be detected as early as weeks after conception. b. hCG levels gradually and uniformly increase throughout pregnancy. c. Significantly lower-than-normal increases in the levels of hCG may indicate a postdate pregnancy. d. Higher-than-normal levels of hCG may indicate an ectopic pregnancy or Down syndrome.

ANS: D Higher hCG levels also could be a sign of a multiple gestation. hCG can be detected as early as 7 to 10 days after conception. The hCG levels fluctuate during pregnancy, peaking, declining, stabilizing, and then increasing again. Abnormally slow increases may indicate impending miscarriage.

Pregnancy hormones prepare the vagina for stretching during labor and birth. Which change related to the pelvic viscera should the nurse share with the client? a. Because of a number of changes in the cervix, abnormal Papanicolaou (Pap) tests are easier to evaluate. b. Quickening is a technique of palpating the fetus to engage it in passive movement. c. The deepening color of the vaginal mucosa and cervix (Chadwick sign) usually appears in the second trimester or later as the vagina prepares to stretch during labor. d. Increased vascularity of the vagina increases sensitivity and may lead to a high degree of arousal, especially in the second trimester.

ANS: D Increased sensitivity and an increased interest in sex sometimes go together and frequently occur during the second trimester. These cervical changes make evaluation of abnormal Pap tests more difficult. Quickening is the first recognition of fetal movements by the mother. Ballottement is a technique used to palpate the fetus. The Chadwick sign appears from the 6 to 8 weeks of gestation.

Which statement related to the induction of labor is most accurate? a.Can be achieved by external and internal version techniques b.Is also known as a trial of labor (TOL) c.Is almost always performed for medical reasons d.Is rated for viability by a Bishop score

ANS: D Induction of labor is likely to be more successful with a Bishop score of 9 or higher for first-time mothers or 5 or higher for veterans. Version is the turning of the fetus to a better position by a physician for an easier or safer birth. A TOL is the observance of a woman and her fetus for several hours of active labor to assess the safety of vaginal birth. Two thirds of cases of induced labor are elective and not done for medical reasons

Which FHR finding is the most concerning to the nurse who is providing care to a laboring client? a.Accelerations with fetal movement b.Early decelerations c.Average FHR of 126 beats per minute d.Late decelerations

ANS: D Late decelerations are caused by uteroplacental insufficiency and are associated with fetal hypoxemia. Late FHR decelerations are considered ominous if they are persistent and left uncorrected. Accelerations with fetal movement are an indication of fetal well-being. Early decelerations in the FHR are associated with head compression as the fetus descends into the maternal pelvic outlet; they are not generally a concern during normal labor. An FHR finding of 126 beats per minute is normal and not a concern.

The nurse is using intermittent auscultation (IA) to locate the fetal heartbeat. Which statement regarding this method of surveillance is accurate? a.The nurse can be expected to cover only two or three clients when IA is the primary method of fetal assessment. b.The best course is to use the descriptive terms associated with EFM when documenting results. c.If the heartbeat cannot be immediately found, then a shift must be made to EFM. d.Ultrasound can be used to find the FHR and to reassure the mother if the initial difficulty is a factor.

ANS: D Locating fetal heartbeats often takes time. Mothers can be verbally reassured and reassured by viewing the ultrasound pictures if that device is used to help locate the heartbeat. When used as the primary method of fetal assessment, IA requires a nurse-to-client ratio of one to one. Documentation should use only terms that can be numerically defined; the usual visual descriptions of EFM are inappropriate

Nafarelin (Synarel) is used to treat mildtosevere endometriosis. What instruction or information should the nurse provide to a client regarding nafarelin administration? a. Nafarelin stimulates the secretion of gonadotropin-releasing hormone (GnRH), thereby stimulating ovarian activity. b. It should be administered by intramuscular (IM) injection. c. Nafarelin should be administered by a subcutaneous implant. d. It can cause the client to experience some hot flashes and vaginal dryness.

ANS: D Nafarelin is a GnRH agonist, and its side effects are similar to those of menopause. The hypoestrogenism effect results in hot flashes and vaginal dryness. Nafarelin is a GnRH agonist that suppresses the secretion of GnRH. Nafarelin is administered twice daily by nasal spray and can be intranasally administered. Leuprolide is given once per month by IM injection. Goserelin is administered by subcutaneous implant.

Which gastrointestinal alteration of pregnancy is a normal finding? a. Insufficient salivation (ptyalism) is caused by increases in estrogen. b. Acid indigestion (pyrosis) begins early but declines throughout pregnancy. c. Hyperthyroidism often develops (temporarily) because hormone production increases. d. Nausea and vomiting rarely have harmful effects on the fetus and may be beneficial.

ANS: D Normal nausea and vomiting rarely produce harmful effects and may be less likely to result in miscarriage or preterm labor. Ptyalism is excessive salivation that may be caused by a decrease in unconscious swallowing or by stimulation of the salivary glands. Pyrosis begins as early as the first trimester and intensifies through the third trimester. Increased hormone production does not lead to hyperthyroidism in pregnant women.

A healthy 60-year-old African-American woman regularly receives health care at her neighborhood clinic. She is due for a mammogram. At her first visit, her health care provider is concerned about the 3-week wait at the neighborhood clinic and made an appointment for her to have a mammogram at a teaching hospital across town. She did not keep her appointment and returned to the clinic today to have the nurse check her blood pressure. What is the most appropriate statement for the nurse to make to this client? a."Do you have transportation to the teaching hospital so that you can get your mammogram?" b."I'm concerned that you missed your appointment; let me make another one for you." c."It's very dangerous to skip your mammograms; your breasts need to be checked." d."Would you like me to make an appointment for you to have your mammogram here?"

ANS: D Offering to make an appointment for the client at the neighborhood location is nonjudgmental and gives her options as to where she may have her mammogram. Furthermore, it is an innocuous way to investigate the reasons the client missed her previous appointment. Mortality rates from breast cancer remain high for African-American women. Rather than reminding this woman that she has "missed her appointment," discussing the evidence behind the recommendations for a mammogram might be preferable for the nurse. The nurse can offer to reschedule should the client agree to return for the test. Telling the client that it is dangerous to skip mammograms can be perceived as judgmental and derogatory and may alienate and embarrass the client.

Which classification of placental separation is not recognized as an abnormal adherence pattern? a.Placenta accreta b.Placenta increta c.Placenta percreta d.Placenta abruptio

ANS: D Placenta abruptio is premature separation of the placenta as opposed to partial or complete adherence. This classification occurs between the 20th week of gestation and delivery in the area of the decidua basalis. Symptoms include localized pain and bleeding. Placenta accreta is a recognized degree of attachment. With placenta accreta, the trophoblast slightly penetrates into the myometrium. Placenta increta is a recognized degree of attachment that results in deep penetration of the myometrium. Placenta percreta is the most severe degree of placental penetration that results in deep penetration of the myometrium. Bleeding with complete placental attachment occurs only when separation of the placenta is attempted after delivery. Treatment includes blood component therapy and, in extreme cases, hysterectomy may be necessary.

Which physiologic alteration of pregnancy most significantly affects glucose metabolism? a. Pancreatic function in the islets of Langerhans is affected by pregnancy. b. Pregnant women use glucose at a more rapid rate than nonpregnant women. c. Pregnant women significantly increase their dietary intake. d. Placental hormones are antagonistic to insulin, thus resulting in insulin resistance.

ANS: D Placental hormones, estrogen, progesterone, and human placental lactogen (HPL) create insulin resistance. Insulin is also broken down more quickly by the enzyme placental insulinase. Pancreatic functioning is not affected by pregnancy. The glucose requirements differ because of the growing fetus. The pregnant woman should increase her intake by 200 calories a day.

Which assessment is least likely to be associated with a breech presentation? a.Meconium-stained amniotic fluid b.Fetal heart tones heard at or above the maternal umbilicus c.Preterm labor and birth d.Postterm gestation

ANS: D Postterm gestation is not likely to occur with a breech presentation. The presence of meconium in a breech presentation may be a result of pressure on the fetal wall as it traverses the birth canal. Fetal heart tones heard at the level of the umbilical level of the mother are a typical finding in a breech presentation because the fetal back would be located in the upper abdominal area. Breech presentations often occur in preterm births.

Which hormone is responsible for the maturation of mammary gland tissue? a. Estrogen b. Testosterone c. Prolactin d. Progesterone

ANS: D Progesterone causes maturation of the mammary gland tissue, specifically acinar structures of the lobules. Estrogen increases the vascularity of the breast tissue. Testosterone has no bearing on breast development. Prolactin is produced after birth and released from the pituitary gland; it is produced in response to infant suckling and an emptying of the breasts.

Which hormone is essential for maintaining pregnancy? a. Estrogen b. hCG c. Oxytocin d. Progesterone

ANS: D Progesterone is essential for maintaining pregnancy; it does so by relaxing smooth muscles, which reduces uterine activity and prevents miscarriage. Estrogen plays a vital role in pregnancy, but it is not the primary hormone for maintaining pregnancy. hCG levels rise at implantation but decline after 60 to 70 days. Oxytocin stimulates uterine contractions.

Which collection of risk factors will most likely result in damaging lacerations, including episiotomies? a. Dark-skinned woman who has had more than one pregnancy, who is going through prolonged second-stage labor, and who is attended by a midwife b. Reddish-haired mother of two who is going through a breech birth c. Dark-skinned first-time mother who is going through a long labor d. First-time mother with reddish hair whose rapid labor was overseen by an obstetrician

ANS: D Reddish-haired women have tissue that is less distensible than darker-skinned women and therefore may have less efficient healing. First-time mothers are also at greater risk, especially with breech births, long second-stage labors, or rapid labors during which the time for the perineum to stretch is insufficient. The rate of episiotomies is higher when obstetricians rather than midwives attend the births. The woman in the first scenario (a) is at low risk for either damaging lacerations or an episiotomy. She is multiparous, has dark skin, and is being attended by a midwife, who is less likely to perform an episiotomy. Reddish-haired women have tissue that is less distensible than that of darker-skinned women. Consequently, the client in the second scenario (b) is at increased risk for lacerations; however, she has had two previous deliveries, which result in a lower likelihood of an episiotomy. The fact that the woman in the third scenario (c) is experiencing a prolonged labor might increase her risk for lacerations. Fortunately, she is dark skinned, which indicates that her tissue is more distensible than that of fair-skinned women and therefore less susceptible to injury.

A 32-year-old primigravida is admitted with a diagnosis of ectopic pregnancy. Which information assists the nurse in developing the plan of care? a.Bed rest and analgesics are the recommended treatment. b.She will be unable to conceive in the future. c.A D&C will be performed to remove the products of conception. d.Hemorrhage is the primary concern

ANS: D Severe bleeding occurs if the fallopian tube ruptures. The recommended treatment is to remove the pregnancy before rupture to prevent hemorrhaging. If the tube must be removed, then the woman's fertility will decrease; however, she will not be infertile. A D&C is performed on the inside of the uterine cavity. The ectopic pregnancy is located within the tubes.

As part of the discharge teaching, the nurse can prepare the mother for her upcoming adjustment to her new role by instructing her regarding self-care activities to help prevent PPD. Which statement regarding this condition ismost helpful for the client? a. Stay home, and avoid outside activities to ensure adequate rest. b. Be certain that you are the only caregiver for your baby to facilitate infant attachment. c. Keep your feelings of sadness and adjustment to your new role to yourself. d. Realize that PPD is a common occurrence that affects many women.

ANS: D Should the new mother experience symptoms of the baby blues, it is important that she be aware that these symptoms are nothing to be ashamed of. As many as 10% to 15% of new mothers experience similar symptoms. Although obtaining enough rest is important for the mother, she should not distance herself from her family and friends. Her spouse or partner can communicate the best visiting times to enable the new mother to obtain adequate rest. It is also important that she not isolate herself at home by herself during this time of role adjustment. Even if breastfeeding, other family members can participate in the infants care. If depression occurs, then the symptoms will often interfere with mothering functions; therefore, family support is essential. The new mother should share her feelings with someone else and avoid overcommitting herself or feel as though she has to besuperwoman. A telephone call to the hospital warm line may provide reassurance with lactation issues and other infant care questions. Should symptoms continue, a referral to a professional therapist may be necessary.

Which sign or symptom is considered a first-trimester warning sign and should be immediately reported by the pregnant woman to her health care provider? a. Nausea with occasional vomiting b. Fatigue c. Urinary frequency d. Vaginal bleeding

ANS: D Signs and symptoms that must be reported include severe vomiting, fever and chills, burning on urination, diarrhea, abdominal cramping, and vaginal bleeding. These symptoms may be signs of complications of the pregnancy. Nausea with occasional vomiting is a normal first-trimester complaint. Although it may be worrisome or annoying to the mother, it is not usually an indication of a problem with the pregnancy. Fatigue is common during the first trimester. Because of physiologic changes that happen during pregnancy, clients should be taught that urinary frequency is normal.

Which risk factor would the nurse recognize as being frequently associated with osteoporosis? a. African-American race b. Lowprotein intake c. Obesity d. Cigarette smoking

ANS: D Smoking is associated with earlier and greater bone loss and decreased estrogen production. Women at risk for osteoporosis are likely to be Caucasian or Asian. Inadequate calcium intake is a risk factor for osteoporosis. Women at risk for osteoporosis are likely to be small boned and thin. Obese women have higher estrogen levels as a result of the conversion of androgens in the adipose tissue. Mechanical stress from extra weight also helps preserve bone mass

Which precaution should the nurse take while caring for a client who is undergoing internal radiation therapy for cervical cancer? a.Wear gloves when assessing the cervical intracavity implant. b.Instruct the client to urinate in the lead-lined bedpan or "hat" every 2 hours. c.Prepare the client for an enema before inserting the implant. d.Limit staff or visitor exposure to 30 minutes or less in an 8-hour period.

ANS: D Staff and visitor exposure should be limited to 30 minutes or less in an 8-hour period to reduce the risk of overexposure to radiation. Nurses need to protect themselves from overexposure to radiation. Wearing a shield is one method of protection. An indwelling catheter is inserted to prevent urinary distention that could dislodge the applicator. No bowel preparation is necessary.

Numerous changes in the integumentary system occur during pregnancy. Which change persists after birth? a. Epulis b. Chloasma c. Telangiectasia d. Striae gravidarum

ANS: D Striae gravidarum, or stretch marks, reflect a separation within the underlying connective tissue of the skin. They usually fade after birth, although they never completely disappear. An epulis is a red, raised nodule on the gums that easily bleeds; it disappears or shrinks after giving birth. Chloasma, or the mask of pregnancy, is a blotchy, brown hyperpigmentation of the skin over the cheeks, nose, and forehead, especially in dark-complexioned pregnant women. Chloasma usually fades after the birth. Telangiectasia, or vascular spiders, are tiny, star-shaped or branchlike, slightly raised, pulsating end-arterioles usually found on the neck, thorax, face, and arms. They occur as a result of elevated levels of circulating estrogen and usually disappear after birth.

Which woman is at the greatest risk for psychologic complications after hysterectomy? a.55-year-old woman who has been having abnormal bleeding and pain for 3 years b.46-year-old woman who has had three children and has just been promoted at work c.62-year-old widow who has three friends who have had uncomplicated hysterectomies d.19-year-old woman who had a ruptured uterus after giving birth to her first child

ANS: D The 19-year-old woman is still in her childbearing years. Often the uterus is related to self-concept in women in this age group, and they may feel that sexual functioning is related to having a uterus. The 55-year-old woman is past her childbearing years and has had bleeding and pain for 3 years. The hysterectomy may be well received as a method of pain relief. The 46-year-old woman has a family and positive events occurring in her life (job promotion). The 62-year-old woman is past her reproductive years and has relationships with others who have had positive outcomes.

The obstetric provider has informed the nurse that she will be performing an amniotomy on the client to induce labor. What is the nurse's highest priority intervention after the amniotomy is performed? a.Applying clean linens under the woman b.Taking the client's vital signs c.Performing a vaginal examination d.Assessing the fetal heart rate (FHR)

ANS: D The FHR is assessed before and immediately after the amniotomy to detect any changes that might indicate cord compression or prolapse. Providing comfort measures, such as clean linens, for the client is important but not the priority immediately after an amniotomy. The woman's temperature should be checked every 2 hours after the rupture of membranes but not the priority immediately after an amniotomy. The woman would have had a vaginal examination during the procedure. Unless cord prolapse is suspected, another vaginal examination is not warranted. Additionally, FHR assessment provides clinical cues to a prolapsed cord.

Some of the embryos intestines remain within the umbilical cord during the embryonic period. What is the rationale for this development of the gastrointestinal system? a. Umbilical cord is much larger at this time than it will be at the end of pregnancy. b. Intestines begin their development within the umbilical cord. c. Nutrient content of the blood is higher in this location. d. Abdomen is too small to contain all the organs while they are developing.

ANS: D The abdominal contents grow more rapidly than the abdominal cavity; therefore, part of their development takes place in the umbilical cord. By 10 weeks of gestation, the abdomen is large enough to contain them. Intestines begin their development within the umbilical cord but only because the liver and kidneys occupy most of the abdominal cavity. Blood supply is adequate in all areas.

A woman is using the basal body temperature (BBT) method of contraception. She calls the clinic and tells the nurse, My period is due in a few days, and my temperature has not gone up. What is the nurses mostappropriate response? a. This probably means that youre pregnant. b. Dont worry; its probably nothing. c. Have you been sick this month? d. You probably didnt ovulate during this cycle.

ANS: D The absence of a temperature decrease most likely is the result of a lack of ovulation. Pregnancy cannot occur without ovulation, which is being measured using the BBT method. A comment such as, Dont worry; its probably nothing, discredits the clients concerns. Illness is most likely the cause of an increase in BBT.

A client currently uses a diaphragm and spermicide for contraception. She asks the nurse to explain the major differences between the cervical cap and the diaphragm. What is the most appropriate response by the nurse? a. No spermicide is used with the cervical cap, so its less messy. b. The diaphragm can be left in place longer after intercourse. c. Repeated intercourse with the diaphragm is more convenient. d. The cervical cap can be safely used for repeated acts of intercourse without adding more spermicide later.

ANS: D The cervical cap can be inserted hours before sexual intercourse without the need for additional spermicide later. Spermicide should be used inside the cap as an additional chemical barrier. The cervical cap should remain in place for 6 hours after the last act of intercourse. Repeated intercourse with the cervical cap is more convenient because no additional spermicide is needed.

The Valsalva maneuver can be described as the process of making a forceful bearing-down attempt while holding one's breath with a closed glottis and a tightening of the abdominal muscles. When is it appropriate to instruct the client to use this maneuver? a. During the second stage to enhance the movement of the fetus b. During the third stage to help expel the placenta c. During the fourth stage to expel blood clots d. Not at all

ANS: D The client should not be instructed to use this maneuver. This process stimulates the parasympathetic division of the autonomic nervous system and produces a vagal response (decrease in heart rate and blood pressure.) An alternative method includes instructing the client to perform open-mouth and open-glottis breathing and pushing.

A blind woman has arrived for an examination. Her guide dog assists her to the examination room. She appears nervous and says, Ive never had a pelvic examination. What response from the nurse would be most appropriate? a. Dont worry. It will be over before you know it. b. Try to relax. Ill be very gentle, and I wont hurt you. c. Your anxiety is common. I was anxious when I first had a pelvic examination. d. Ill let you touch each instrument that Ill use during the examination as I tell you how it will be used.

ANS: D The client who is visually impaired needs to be oriented to the examination room and needs a full explanation of what the examination entails before the nurse proceeds. Telling the client that the examination will be over quickly diminishes the clients concerns. The nurse should openly and directly communicate with sensitivity. Women who have physical disabilities should be respected and involved in the assessment and physical examination to the full extent of their abilities. Telling the client that she will not be hurt does not reflect respect or sensitivity. Although anxiety may be common, the nurse should not discuss her own issues nor compare them to the clients concerns.

Which part of the menstrual cycle includes the stimulated release of gonadotropin-releasing hormone (GnRH) and follicle-stimulating hormone (FSH)? a. Menstrual phase b. Endometrial cycle c. Ovarian cycle d. Hypothalamic-pituitary cycle

ANS: D The cyclic release of hormones is the function of the hypothalamus and pituitary glands. The menstrual cycle is a complex interplay of events that simultaneously occur in the endometrium, hypothalamus, pituitary glands, and ovaries. The endometrial cycle consists of four phases: menstrual phase, proliferative phase, secretory phase, and ischemic phase. The ovarian cycle remains under the influence of FSH and estrogen.

Nurses should be cognizant of what information with regard to the noncontraceptive medical effects of combination oral contraceptives (COCs)? a. COCs can cause TSS if the prescription is wrong. b. Hormonal withdrawal bleeding is usually a little more profuse than in normal menstruation and lasts a week for those who use COCs. c. COCs increase the risk of endometrial and ovarian cancers. d. Effectiveness of COCs can be altered by some over-the-counter medications and herbal supplements.

ANS: D The effectiveness of COCs can be altered by some over-the-counter medications and herbal supplements. TSS can occur in some who use the diaphragm, but it is not a consequence of taking oral contraceptive pills. Hormonal withdrawal bleeding usually is lighter than in normal menstruation and lasts a couple of days. Oral contraceptive pills offer protection against the risk of endometrial and ovarian cancers.

A client has just returned from a uterine artery embolization (UAE) procedure. Before her discharge, a discussion concerning the symptoms that require a call to the provider postprocedure is very important. Which symptom would reassure the client that the procedure went well with no reason to call the provider? a.Temperature of 39° C b.Swelling or hematoma at the puncture site c.Abnormal vaginal discharge d.Urinary frequency

ANS: D The physician should be notified if the client is experiencing urinary retention. Urinary frequency is not a complication of UAE. A body temperature of 39° C or higher may indicate the presence of an infectious process, and the physician should be notified. A slight fever or pain may be experienced as a result of acute fibroid degradations. Swelling or hematoma at the puncture site may be an indication of bleeding into the groin. The client should not experience any abnormal vaginal discharge (e.g., foul odor, brown color, brown tissue).

Nutrition is an alterable and important preventive measure for a variety of potential problems such as low birth weight and prematurity. While completing the physical assessment of the pregnant client, the nurse is able to evaluate the clients nutritional status by observing a number of physical signs. Which physical sign indicates to the nurse that the client has unmet nutritional needs? a. Normal heart rate, rhythm, and blood pressure b. Bright, clear, and shiny eyes c. Alert and responsive with good endurance d. Edema, tender calves, and tingling

ANS: D The physiologic changes of pregnancy may complicate the interpretation of physical findings. Lower extremity edema often occurs when caloric and protein deficiencies are present; however, edema in the lower extremities may also be a common physical finding during the third trimester. Completing a thorough health history and physical assessment and requesting further laboratory testing, if indicated, are essential for the nurse. The malnourished pregnant client may display rapid heart rate, abnormal rhythm, enlarged heart, and elevated blood pressure. A client receiving adequate nutrition will have bright, shiny eyes with no sores and moist, pink membranes. Pale or red membranes, dryness, infection, dull appearance of the cornea, or blue sclerae are signs of poor nutrition. A client who is alert and responsive with good endurance is well nourished. A listless, cachectic, easily fatigued, and tired presentation would be an indication of a poor nutritional status.

In comparing the abdominal and transvaginal methods of ultrasound examination, which information should the nurse provide to the client? a. Both require the woman to have a full bladder. b. The abdominal examination is more useful in the first trimester. c. Initially, the transvaginal examination can be painful. d. The transvaginal examination allows pelvic anatomy to be evaluated in greater detail.

ANS: D The transvaginal examination allows pelvic anatomy to be evaluated in greater detail than the abdominal method and also allows intrauterine pregnancies to be diagnosed earlier. The abdominal examination requires a full bladder; the transvaginal examination requires an empty one. The transvaginal examination is more useful in the first trimester; the abdominal examination works better after the first trimester. Neither the abdominal nor the transvaginal method of ultrasound examination should be painful, although the woman will feel pressure as the probe is moved during the transvaginal examination.

What is the goal of a long-term treatment plan for an adolescent with an eating disorder? a. Managing the effects of malnutrition b. Establishing sufficient caloric intake c. Improving family dynamics d. Restructuring client perception of body image

ANS: D The treatment of eating disorders is initially focused on reestablishing physiologic homeostasis. Once body systems are stabilized, the next goal of treatment for eating disorders is maintaining adequate caloric intake. Although family therapy is indicated when dysfunctional family relationships exist, the primary focus of therapy for eating disorders is to help the adolescent cope with complex issues. The focus of treatment in individual therapy for an eating disorder involves restructuring cognitive perceptions about the individuals body image.

A multiparous woman has been in labor for 8 hours. Her membranes have just ruptured. What is the nurse's highest priority in this situation? a. Prepare the woman for imminent birth. b. Notify the woman's primary health care provider. c. Document the characteristics of the fluid. d. Assess the fetal heart rate (FHR) and pattern.

ANS: D The umbilical cord may prolapse when the membranes rupture. The FHR and pattern should be closely monitored for several minutes immediately after the ROM to ascertain fetal well-being, and the findings should be documented. The ROM may increase the intensity and frequency of the uterine contractions, but it does not indicate that birth is imminent. The nurse may notify the primary health care provider after ROM occurs and the fetal well-being and response to ROM have been assessed. The nurse's priority is to assess fetal well-being. The nurse should document the characteristics of the amniotic fluid, but the initial response is to assess fetal well-being and the response to ROM.

What represents a typical progression through the phases of a womans establishing a relationship with the fetus? a. Accepts the fetus as distinct from herselfaccepts the biologic fact of pregnancyhas feelings of caring and responsibility. b. Fantasizes about the childs gender and personalityviews the child as part of herselfbecomes introspective. c. Views the child as part of herselfhas feelings of well-beingaccepts the biologic fact of the pregnancy. d. I am pregnantI am going to have a babyI am going to be a mother.

ANS: D The woman first centers on herself as pregnant, then on the baby as an entity separate from herself, and then on her responsibilities as a mother. The expressions I am pregnant, I am going to have a baby, and I am going to be a mother sum up the progression through the three phases. In phase one, the woman views the child as part of herself and not as a separate being. This is only the first step of the progression through phases of attachment. Accepting the fetus as distinct from herself occurs during the second phase of emotional attachment. Fantasizing about the childs sex and personality based on fetal activity occurs during the third phase of attachment.

A woman arrives at the clinic seeking confirmation that she is pregnant. The following information is obtained: She is 24 years old with a body mass index (BMI) of 17.5. She admits to having used cocaine several times during the past year and occasionally drinks alcohol. Her blood pressure is 108/70 mm Hg. The family history is positive for diabetes mellitus and cancer. Her sister recently gave birth to an infant with a neural tube defect (NTD). Which characteristics places this client in a high-risk category? a. Blood pressure, age, BMI b. Drug and alcohol use, age, family history c. Family history, blood pressure (BP), BMI d. Family history, BMI, drug and alcohol abuse

ANS: D The womans family history of an NTD, her low BMI, and her drug and alcohol use abuse are high risk factors of pregnancy. The womans BP is normal, and her age does not put her at risk. Her BMI is low and may indicate poor nutritional status, which is a high risk.

A woman in the active phase of the first stage of labor is using a shallow pattern of breathing, which is approximately twice the normal adult breathing rate. She starts to complain about feeling lightheaded and dizzy and states that her fingers are tingling. Which intervention should the nurse immediately initiate? a.Contact the woman's physician. b.Tell the woman to slow her pace of her breathing. c.Administer oxygen via a mask or nasal cannula. d.Help her breathe into a paper bag.

ANS: D This woman is experiencing the side effects of hyperventilation, which include the symptoms of lightheadedness, dizziness, tingling of the fingers, or circumoral numbness. Having the woman breathe into a paper bag held tightly around her mouth and nose may eliminate respiratory alkalosis and enable her to rebreathe carbon dioxide and replace the bicarbonate ion.

A client is in early labor, and her nurse is discussing the pain relief options she is considering. The client states that she wants an epidural "no matter what!" What is the nurse's best response? a."I'll make sure you get your epidural." b."You may only have an epidural if your physician allows it." c."You may only have an epidural if you are going to deliver vaginally." d."The type of analgesia or anesthesia used is determined, in part, by the stage of your labor and the method of birth."

ANS: D To avoid suppressing the progress of labor, pharmacologic measures for pain relief are generally not implemented until labor has advanced to the active phase of the first stage and the cervix is dilated approximately 4 to 5 cm. A plan of care is developed for each woman that addresses her particular clinical and nursing problems. The nurse collaborates with the primary health care provider and the laboring woman in selecting features of care relevant to the woman and her family. The decision whether to use an epidural to relieve labor pain is multifactorial. The nurse should not make a blanket statement guaranteeing the client one pharmacologic option over another until a complete history and physical examination has been obtained. A physician's order is required for pharmacologic options for pain management. However, expressing this requirement is not the nurse's best response. An epidural is an effective pharmacologic pain management option for many laboring women. It can also be used for anesthesia control if the woman undergoes an operative delivery.

A woman who is 16 weeks pregnant has come in for a follow-up visit with her significant other. To reassure the client regarding fetal well-being, which is the highest priority action for the nurse to perform? a. Assess the fetal heart tones with a Doppler stethoscope. b. Measure the girth of the womans abdomen. c. Complete an ultrasound examination (sonogram). d. Offer the woman and her family the opportunity to listen to the fetal heart tones.

ANS: D To provide the parents with the greatest sense of reassurance, the nurse should offer to have the client and her significant other the chance to listen to their babys heartbeat. A fetoscope can detect the fetal heart rate around 20 weeks of gestation. Doppler can detect the fetal heart rate between 10 and 12 weeks and should be performed as part of routine fetal assessment. Abdominal girth is not a valid measure for determining fetal well-being. Fundal height is an important measure that should be determined with precision, with the same technique and positioning of the client consistently used at every prenatal visit. Routine ultrasound examinations are recommended in early pregnancy; they date the pregnancy and provide useful information about the health of the fetus. However, they are not necessary at each prenatal visit.

The nurse is evaluating the EFM tracing of the client who is in active labor. Suddenly, the FHR drops from its baseline of 125 down to 80 beats per minute. The mother is repositioned, and the nurse provides oxygen, increased IV fluids, and performs a vaginal examination. The cervix has not changed. Five minutes have passed, and the FHR remains in the 80s. What additional nursing measures should the nurse take next? a.Call for help. b.Insert a Foley catheter. c.Start administering Pitocin. d.Immediately notify the care provider.

ANS: D To relieve an FHR deceleration, the nurse can reposition the mother, increase IV fluids, and provide oxygen. If oxytocin is infusing, then it should be discontinued. If the FHR does not resolve, then the primary care provider should be immediately notified. Inserting a Foley catheter is an inappropriate nursing action. If the FHR were to continue in a nonreassuring pattern, then a cesarean section could be warranted, which would require a Foley catheter. However, the physician must make that determination. The administration of Pitocin may place additional stress on the fetus.

Maternity nurses often have to answer questions about the many, sometimes unusual, ways people have tried to make the birthing experience more comfortable. Which information regarding nonpharmacologic pain relief isaccurate? a.Music supplied by the support person has to be discouraged because it could disturb others or upset the hospital routine. b.Women in labor can benefit from sitting in a bathtub, but they must limit immersion to no longer than 15 minutes at a time. c.Effleurage is permissible, but counterpressure is almost always counterproductive. d.Electrodes attached to either side of the spine to provide high-intensity electrical impulses facilitate the release of endorphins.

ANS: D Transcutaneous electrical nerve stimulation (TENS) may help and is most useful for lower back pain that occurs during the first stage of labor. Music may be very helpful for reducing tension and certainly can be accommodated by the hospital. Women can stay in a bath as long as they want, although repeated baths with breaks might be more effective than one long bath. Counterpressure can help the woman cope with lower back pain.

Which characteristic of a uterine contraction is not routinely documented? a. Frequency: how often contractions occur b. Intensity: strength of the contraction at its peak c. Resting tone: tension in the uterine muscle d. Appearance: shape and height

ANS: D Uterine contractions are described in terms of frequency, intensity, duration, and resting tone. Appearance is not routinely charted.

Which statement best describes chronic hypertension? a. Chronic hypertension is defined as hypertension that begins during pregnancy and lasts for the duration of the pregnancy. b. Chronic hypertension is considered severe when the systolic BP is higher than 140 mm Hg or the diastolic BP is higher than 90 mm Hg. c. Chronic hypertension is general hypertension plus proteinuria. d. Chronic hypertension can occur independently of or simultaneously with preeclampsia.

ANS: D Women with chronic hypertension may develop superimposed preeclampsia, which increases the morbidity for both the mother and the fetus. Chronic hypertension is present before pregnancy or diagnosed before the 20 weeks of gestation and persists longer than 6 weeks postpartum. Chronic hypertension becomes severe with a diastolic BP of 110 mm Hg or higher. Proteinuria is an excessive concentration of protein in the urine and is a complication of hypertension, not a defining characteristic.

Which basic type of pelvis includes the correct description and percentage of occurrence in women? a. Gynecoid: classic female pelvis; heart shaped; 75% b. Android: resembling the male pelvis; wide oval; 15% c. Anthropoid: resembling the pelvis of the ape; narrow; 10% d. Platypelloid: flattened, wide, and shallow pelvis; 3%

ANS: D A platypelloid pelvis is flattened, wide, and shallow; approximately 3% of women have this shape. The gynecoid pelvis is the classic female shape, slightly ovoid and rounded; approximately 50% of women have this shape. An android or malelike pelvis is heart shaped; approximately 23% of women have this shape. An anthropoid or apelike pelvis is oval and wide; approximately 24% of women have this shape.

Since the gene for cystic fibrosis was identified in 1989, data can be collected for the purposes of genetic counseling for couples regarding carrier status. According to the most recent statistics, how often does cystic fibrosis occur in Caucasian live births? a. 1 in 100 b. 1 in 1000 c. 1 in 2000 d. 1 in 3200

ANS: D Cystic fibrosis occurs in approximately 1 in 3200 Caucasian live births. 1 in 100, 1 in 1000, and 1 in 2000 occurrences of cystic fibrosis in live births are all too frequent rates.

Certain changes stimulate chemoreceptors in the aorta and carotid bodies to prepare the fetus for initiating respirations immediately after birth. Which change in fetal physiologic activity is not part of this process? a. Fetal lung fluid is cleared from the air passages during labor and vaginal birth. b. Fetal partial pressure of oxygen (PO2) decreases. c. Fetal partial pressure of carbon dioxide in arterial blood (PaCO2) increases. d. Fetal respiratory movements increase during labor.

ANS: D Fetal respiratory movements actually decrease during labor. Fetal lung fluid is cleared from the air passages during labor and vaginal birth. Fetal PO2 decreases, and fetal PaCO2 increases.

What form of heart disease in women of childbearing years generally has a benign effect on pregnancy? a. Cardiomyopathy b. Rheumatic heart disease c. Congenital heart disease d. Mitral valve prolapse

ANS: D Mitral valve prolapse is a benign condition that is usually asymptomatic. Cardiomyopathy produces congestive heart failure during pregnancy. Rheumatic heart disease can lead to heart failure during pregnancy. Some congenital heart diseases produce pulmonary hypertension or endocarditis during pregnancy.

Another common pregnancy-specific condition is pruritic urticarial papules and plaques of pregnancy (PUPPP). A client asks the nurse why she has developed this condition and what can be done. What is the nurse's best response? a. PUPPP is associated with decreased maternal weight gain. b. The rate of hypertension decreases with PUPPP. c. This common pregnancy-specific condition is associated with a poor fetal outcome. d. The goal of therapy is to relieve discomfort.

ANS: D PUPPP is associated with increased maternal weight gain, increased rate of twin gestation, and hypertension. It is not, however, associated with poor maternal or fetal outcomes. The goal of therapy is simply to relieve discomfort. Antipruritic topical medications, topical steroids, and antihistamines usually provide relief. PUPPP usually resolves before childbirth or shortly thereafter.

Which statement regarding the care of a client in labor is correct and important to the nurse as he or she formulates the plan of care? a. The woman's blood pressure will increase during contractions and fall back to prelabor normal levels between contractions. b. The use of the Valsalva maneuver is encouraged during the second stage of labor to relieve fetal hypoxia. c. Having the woman point her toes will reduce leg cramps. d. Endogenous endorphins released during labor will raise the woman's pain threshold and produce sedation.

ANS: D The endogenous endorphins released during labor will raise the woman's pain threshold and produce sedation. In addition, physiologic anesthesia of the perineal tissues, caused by the pressure of the presenting part, decreases the mother's perception of pain. Blood pressure levels increase during contractions but remain somewhat elevated between them. The use of the Valsalva maneuver is discouraged during the second stage labor because of a number of unhealthy outcomes, including fetal hypoxia. Pointing the toes can cause leg cramps, as can the process of labor itself.

The nurse taught new parents the guidelines to follow regarding the bottle feeding of their newborn. They will be using formula from a can of concentrate. The parents would demonstrate an understanding of the nurse's instructions if they: a. Wash the top of can and can opener with soap and water before opening the can. b. Adjust the amount of water added according to weight gain pattern of the newborn. c.. Add some honey to sweeten the formula and make it more appealing to a fussy newborn. d. Warm formula in a microwave oven for a couple of minutes prior to feeding.

a. Wash the top of can and can opener with soap and water before opening the can.

A new father is ready to take his wife and newborn son home. He proudly tells the nurse who is discharging them that within the next week he plans to start feeding the infant cereal between breastfeeding sessions. Which information should the nurse provide regarding this feeding plan? a. "Feeding solid foods before your son is 4 to 6 months old may decrease your son's intake of sufficient calories." b. "Feeding solid foods between breastfeeding sessions before your son is 4 to 6 months old will lead to an early cessation of breastfeeding." c. "Your feeding plan will help your son sleep through the night." d. "Feeding solid foods before your son is 4 to 6 months old will limit his growth."

b. "Feeding solid foods between breastfeeding sessions before your son is 4 to 6 months old will lead to an early cessation of breastfeeding."

Following a vaginal delivery, the patient tells the nurse that she intends to breastfeed her infant but she is very concerned about returning to her prepregnancy weight. On the basis of this interaction, the nurse would advise the patient that: (Select all that apply.) a. She should join Weight Watchers as soon as possible to ensure adequate weight loss. b. Even though more calories are needed for lactation, typically women who breastfeed lose weight more rapidly than women who bottle feed in the postpartum period. c. Weight loss diets are not recommended for women who breastfeed. d. If breastfeeding, she should regulate her fluid consumption in response to her thirst level. e. If she decreases her calorie intake by 100-200 calories a day she will lose weight more quickly.

b. Even though more calories are needed for lactation, typically women who breastfeed lose weight more rapidly than women who bottle feed in the postpartum period. c. Weight loss diets are not recommended for women who breastfeed. d. If breastfeeding, she should regulate her fluid consumption in response to her thirst level.

A postpartum woman telephones the provider regarding her 5-day-old infant. The client is not scheduled for another weight check until the infant is 14 days old. The new mother is worried about whether breastfeeding is going well. Which statement indicates that breastfeeding is effective for meeting the infant's nutritional needs? a. Sleeps for 6 hours at a time between feedings b. Has at least one breast milk stool every 24 hours c. Gains 1 to 2 ounces per week d. Has at least six to eight wet diapers per day

d. Has at least six to eight wet diapers per day


Set pelajaran terkait

Pharm FINAL study guide questions

View Set

Theoritical Foundation of Nursing

View Set

Personal Finance: Chapter 15 Investing in Bonds

View Set

MARK 3000 Exam #1 Practice Questions

View Set

Maternity-Physiological Adaptation

View Set

Chapter 1 Introduction to Troubleshooting

View Set